NIOS Math Book (1) Tharanya Chenai
NIOS Math Book (1) Tharanya Chenai
Algebra
1 Notes
NUMBER SYSTEMS
From time immemorial human beings have been trying to have a count of their
belongings- goods, ornaments, jewels, animals, trees, sheeps/goats, etc. by using various
techniques
- putting scratches on the ground/stones
- by storing stones - one for each commodity kept/taken out.
This was the way of having a count of their belongings without having any knowledge of
counting.
One of the greatest inventions in the history of civilization is the creation of numbers. You
can imagine the confusion when there were no answers to questions of the type “How
many?”, “How much?” and the like in the absence of the knowledge of numbers. The
invention of number system including zero and the rules for combining them helped people
to reply questions of the type:
(i) How many apples are there in the basket?
(ii) How many speakers have been invited for addressing the meeting?
(iii) What is the number of toys on the table?
(iv) How many bags of wheat have been the yield from the field?
The answers to all these situations and many more involve the knowledge of numbers and
operations on them. This points out to the need of study of number system and its extensions
in the curriculum. In this lesson, we will present a brief review of natural numbers, whole
numbers and integers. We shall then introduce you about rational and irrational numbers in
detail. We shall end the lesson after discussing about real numbers.
OBJECTIVES
After studying this lesson, you will be able to
• illustrate the extension of system of numbers from natural numbers to real
(rationals and irrational) numbers
Recall that the counting numbers 1, 2, 3, ... constitute the system of natural numbers.
These are the numbers which we use in our day-to-day life.
Recall that there is no greatest natural number, for if 1 is added to any natural number, we
get the next higher natural number, called its successor.
We have also studied about four-fundamental operations on natural numbers. For, example,
4 + 2 = 6, again a natural number;
6 + 21 = 27, again a natural number;
22 – 6 = 16, again a natural number, but
2 – 6 is not defined in natural numbers.
Similarly, 4 × 3 = 12, again a natural number
12 × 3 = 36, again a natural number
12 6
= 6 is a natural number but is not defined in natural numbers. Thus, we can say that
2 4
i) a) addition and multiplication of natural numbers again yield a natural number but
Notes
b) subtraction and division of two natural numbers may or may not yield a natural
number
ii) The natural numbers can be represented on a number line as shown below.
• • • • • • • • •
1 2 3 4 5 6 7 8 9 ...........
iii) Two natural numbers can be added and multiplied in any order and the result obtained
is always same. This does not hold for subtraction and division of natural numbers.
1.2.2 Whole Numbers
(i) When a natural number is subtracted from itself we can not say what is the left out
number. To remove this difficulty, the natural numbers were extended by the number
zero (0), to get what is called the system of whole numbers
Thus, the whole numbers are
0, 1, 2, 3, ...........
Again, like before, there is no greatest whole number.
(ii) The number 0 has the following properties:
a+0=a=0+a
a – 0 = a but (0 – a) is not defined in whole numbers
a×0=0=0×a
Division by zero (0) is not defined.
(iii) Four fundamental operations can be performed on whole numbers also as in the case
of natural numbers (with restrictions for subtraction and division).
(iv) Whole numbers can also be represented on the number line as follows:
• • • • • • • • • •
0 1 2 3 4 5 6 7 8 9 ...........
1.2.3 Integers
While dealing with natural numbers and whole numbers we found that it is not always
possible to subtract a number from another.
We extend the number line used for representing whole numbers to the left of zero and
mark points – 1, – 2, – 3, – 4, ... such that 1 and – 1, 2 and – 2, 3 and – 3 are equidistant
from zero and are in opposite directions of zero. Thus, we have the integer number line as
follows:
• • • • • • • • •
.......... –4 –3 –2 –1 0 1 2 3 4.......
We can now easily represent integers on the number line. For example, let us represent
– 5, 7, – 2, – 3, 4 on the number line. In the figure, the points A, B, C, D and E respectively
represent – 5, 7, – 2, – 3 and 4.
A D C E B
• • • • • • • • • • • • • • • •
–7 –6 –5 –4 –3 –2 –1 0 1 2 3 4 5 6 7 8
We note here that if an integer a > b, then ‘a’ will always be to the right of ‘b’, otherwise
vise-versa.
For example, in the above figure 7 > 4, therefore B lies to the right of E. Similarly,
– 2 > – 5, therefore C (– 2) lies to the right of A (–5).
Conversely, as 4 < 7, therefore 4 lies to the left of 7 which is shown in the figure as E is to
the left of B
∴ For finding the greater (or smaller) of the two integers a and b, we follow the following
rule:
i) a > b, if a is to the right of b
ii) a < b, if a is to the left of b
Example 1.1: Identify natural numbers, whole numbers and integers from the following:-
15, 22, – 6, 7, – 13, 0, 12, – 12, 13, – 31
Solution: Natural numbers are: 7, 12, 13, 15 and 22
whole numbers are: 0, 7, 12, 13, 15 and 22
Integers are: – 31, – 13, – 12, – 6, 0, 7, 12, 13, 15 and 22
A represents 9 on the number line. Going 5 units to the left of A, we reach the point
B, which represents 4.
∴ 9 + (–5) = 4
Starting from zero (0) and going three units to the left of zero, we reach the point
Notes A, which represents – 3. From A going 7 units to the left of A, we reach the point
B which represents – 10.
∴ (–3) + (–7) = – 10
a
Suppose a = mb, where m is a natural number or integer, then =m
b
(ii) When a is not a multiple of b
a
In this case is not an integer, and hence is a new type of number. Such a number is
b
called a rational number.
p
Thus, a number which can be put in the form q , where p and q are integers and q ≠ 0, is
called a rational number
2 5 6 11
Thus, − , , , are all rational numbers.
3 −8 2 7
p
(i) A rational number q is said to be a positive rational number if p and q are both
positive or both negative integers
3 5 − 3 − 8 − 12
Thus , , , , are all positive rationals.
4 6 − 2 − 6 − 57
p
(ii) If the integes p and q are of different signs, then q is said to be a negaive rational
number.
− 7 6 − 12 16
Thus, , , , are all negaive rationals.
2 −5 4 −3
−p p −p p
, , and
q −q −q q
−p ⎛ p ⎞ − p − (− p ) p p (− p ) = − p ,
= −⎜⎜ ⎟⎟ , = = , =
q ⎝ q ⎠ − q − (− q ) q − q − (− q ) q
p
A rational number , where p and q are integers and q ≠ 0, in which q is positive (or
q
made positive) and p and q are co-prime (i.e. when they do not have a common factor
other than 1 and –1) is said to be in standard form.
2 −2 −5 −3
Thus the standard form of the rational number is . Similarly, and are
−3 3 6 5
rational numbers in standard form.
Note: “A rational number in standard form is also referred to as “a rational number in its
lowest form”. In this lesson, we will be using these two terms interchangably.
18 2
For example, rational number can be written as in the standard form (or the lowest
27 3
form) .
25 −5
Similarly, , in standard form (or in lowest form) can be written as (cancelling out
− 35 7
5 from both numerator and denominator).
Example 1.5: Which of the following are rational numbers and which are not?
5 15 18 7
− 2, ,−17, , ,−
3 7 5 6
Notes
Solution:
−2 p
(i) –2 can be written as , which is of the form q , q ≠ 0. Therefore, –2 is a rational
1
number.
5 p
(ii) is a rational number, as it is of the form q , q ≠ 0
3
− 17
(iii) –17 is also a rational number as it is of the form
1
15 18 −7
(iv) Similarly, , and are all rational numbers according to the same argument
7 5 6
Example 1.6: Write the following rational numbers in their lowest terms:
− 24 12 − 21
(i) (ii) (iii)
192 168 49
Solution:
− 24 − 3× 8 −1
(i) = =
192 3× 8× 8 8
1 − 24
− is the lowest form of the rational number
8 192
12 12 1
(ii) = =
168 12 × 14 14
1 12
∴ is the lowest form of the rational number
14 168
− 21 − 3× 7 − 3
(iii) = =
49 7×7 7
−3 − 21
∴ is the lowest form of the rational number
7 49
2 2× 2 4 2 2× 4 8 2 × 8 16
= = , = = , =
3 3× 2 6 3 3 × 4 12 3 × 8 24
4 8 16 2
∴ , , etc. are equivalent forms of the rational number
6 12 24 3
Similarly
3 6 21 27
= = = = ...
8 16 56 72
4 8 12 28
and = = = = ...
7 14 21 49
3 4
are equivalent forms of and respectively.
8 7
Example 1.7: Write five equivalent forms of the following rational numbers:
3 −5
(i) (ii)
17 9
Solution:
3 3× 2 6 3 3 × 4 12 3 × (− 3) −9
= = , = = , =
17 17 × 2 34 17 17 × 4 68 17 × (− 3) − 51
(i)
3× 8 24 3 7 21
= , × =
17 × 8 136 17 7 119
3
∴ Five equivalent forms of are
17
6 12 − 9 24 21
, , , ,
34 68 − 51 136 119
−5
(ii) As in part (i), five equivalent forms of are
9
Notes − 10 − 15 − 20 − 60 − 35
, , , ,
18 27 36 108 63
A’ R O P A
–2 –1 0 1 1 2 3
2
4
Similarly, can be represented on the number line as below:
3
C’ B’ A’ O A P B C D
–3 –2 –1 0 1 2 3 4
4/3
4 4
As 1 < < 2, therefore lies between 1 and 2. Divide the distance AB in three equal
3 3
parts. Let one of this part be AP
4 1
Now = 1 + = OA + AP = OP
3 3
4
The point P represents on the number line.
3
In order to compare two rational numbers, we follow any of the following methods:
(i) If two rational numbers, to be compared, have the same denominator, compare their
numerators. The number having the greater numerator is the greater rational number.
5 9
Thus for the two rational numbers and , with the same positive denominator
17 17
9 5
17, > as 9 > 5
17 17
9 5
∴ >
17 17
(ii) If two rational numbers are having different denominators, make their denominators
equal by taking their equivalent form and then compare the numerators of the resulting
rational numbers. The number having a greater numerator is greater rational number.
3 6
For example, to compare two rational numbers and , we first make their
7 11
denominators same in the following manner:
3 × 11 33 9 × 7 42
= and =
7 × 11 77 11× 7 77
42 33 6 3
As 42 > 33, > or >
77 77 11 7
(iii) By plotting two given rational numbers on the number line we see that the rational
number to the right of the other rational number is greater.
2 3
For example, take and , we plot these numbers on the number line as below:
3 4
2
A (3) B
–2 –1 0 (3) 1 2 3 4
4
2 3 2 3
0< < 1 and 0 < < 1. It means and both lie between 0 and 1. By the method
3 4 3 4
2 3
Notes of dividing a line into equal number of parts, A represents and B represents
3 4
3 2 2 3
As B is to the right of A, > or <
4 3 3 4
2 3 3
∴ Out of and , is the greater number.
3 4 4
7 −3 5 3 4
− ,16, ,−15,0, , ,−
4 7 17 − 4 3
3. By making the following rational numbers with same denominator, simplify the following
and specify whether the result in each case is a natural number, whole number, integer
or a rational number:
7 10
(i) 3 + (ii) − 3 + (iii) − 8 − 13 (iv) 12 − 12
3 4
9 1 5
(v) − (vi) 2 × (vii) 8 ÷ 3
2 2 7
8 5 − 3 − 6 2 3 15
, , , , ,
12 7 12 7 27 24
6. Which of the following rational numbers are integers?
Notes
15 − 5 13 27 7 × 3 − 6
− 10, , , , , ,
5 15 5 9 14 − 2
7. Write 3 rational numbers equivalent to given rational numbers:
2 − 5 17
, ,
5 6 3
8. Represent the following rational numbers on the number line.
2 3 1
, ,
5 4 2
9. Compare the following rational numbers by (i) changing them to rational numbers in
equivalent forms (ii) using number line:
2 3 3 7 –2 −1
(a) and (b) and (c) and
3 4 5 9 3 2
3 5 −7 3
(d) and (e) and
7 11 6 2
p r p+r
+ =
q q q
For example
2 5 2+5 7
(i) + = =
3 3 3 3
3 9 3 + 9 12
(ii) + = =
17 17 17 17
14 ⎛ − 5 ⎞ 14 − 5 9
and (iii) +⎜ ⎟= = =3
3 ⎝ 3 ⎠ 3 3
p r
(b) Consider the two rational numbers and .
q s
Notes p r ps rq ps + rq
+ = + =
q s qs sq qs
For example,
3 2 3 × 3 + 4 × 2 9 + 8 17
(i) + = = =
4 3 4×3 12 12
4 7 − 4 × 8 + 5 × 7 35 − 32 3
(ii) − + = = =
5 8 5×8 40 40
2 6 4 −3 5 −3
(i) and (ii) and (iii) − and
7 7 17 17 11 11
2 6 2+6 8
Solution: (i) + = =
7 7 7 7
2 6 8
∴ + =
7 7 7
4 (− 3) 4 + (− 3) 4 − 3 1
(ii) + = = =
17 17 17 17 17
4 (− 3) 1
∴ + =
17 17 17
⎛ 5 ⎞ ⎛ − 3 ⎞ (− 5) + (− 3) − 5 − 3 − 8
(iii) ⎜ − ⎟ + ⎜ ⎟= = =
⎝ 11 ⎠ ⎝ 11 ⎠ 11 11 11
⎛ 5⎞ ⎛ 3⎞ 8
∴⎜ − ⎟ + ⎜ − ⎟ = − Notes
⎝ 11 ⎠ ⎝ 11 ⎠ 11
3 × 7 1× 4
= +
4× 7 7× 4
21 4 21 + 4
= + =
28 28 28
25
=
28
3 1 25 ⎡ 3 × 7 + 4 × 1 21 + 4 25 ⎤
∴ + = or = = ⎥
4 7 28 ⎢⎣ 4 × 7 28 28 ⎦
2 3
(ii) +
7 5
2 × 5 3× 7
= +
7 × 5 5× 7
10 21
= +
35 35
10 + 21 31
= =
35 35
2 3 31 ⎡ 2 × 5 + 3× 7 10 + 21 31⎤
∴ + = or ⎢ = = ⎥
7 5 35 ⎣ 35 35 35⎦
5 (− 4 )
(iii) +
9 15
5 × 15 (− 4 )× 9
= +
9 × 15 15 × 9
75 (− 36 )
= +
135 135
Mathematics Secondary Course 17
MODULE - 1 Number Systems
Algebra
75 − 36 39 3 × 13 13
= = = =
135 135 3 × 45 45
5 (− 4 ) 13 ⎡ 5 × 15 + 9 × (− 4 ) 75 − 36 39 13 ⎤
∴ + = or ⎢ = = =
135 45 ⎥⎦
Notes
9 15 45 ⎣ 9 × 15 135
1.7.2 Subtraction of Rational Numbers
p r p−r
(a) − =
q q q
p r ps − qr
(b) − =
q s qs
⎛ p⎞ ⎛r⎞ p ⎛s⎞
In other words ⎜⎜ ⎟⎟ ÷ ⎜ ⎟ = × ⎜ ⎟
⎝ q⎠ ⎝s⎠ q ⎝r⎠
Or (First rational number) × (Reciprocal of the second rational number)
Notes
Let us consider some examples.
Example 1.11: Multiply the following rational numbers:
3 2 5 ⎛−2⎞ 7 ⎛−2⎞
(i) and (ii) and ⎜ ⎟ (iii) and ⎜ ⎟
7 9 6 ⎝ 19 ⎠ 13 ⎝ −5⎠
3 2 3× 2 3× 2 2
Solution: (i) × = = =
7 9 7 × 9 7 × 3 × 3 21
⎛3⎞ ⎛2⎞ 2
∴⎜ ⎟ × ⎜ ⎟ =
⎝ 7 ⎠ ⎝ 9 ⎠ 21
5 ⎛ − 2 ⎞ 5 × (− 2 )
(ii) ×⎜ ⎟=
6 ⎝ 19 ⎠ 6 × 19
2×5 5
=− =−
2 × 3 × 19 57
⎛5⎞ ⎛ 2 ⎞ 5
∴⎜ ⎟ × ⎜ − ⎟ = −
⎝ 6 ⎠ ⎝ 19 ⎠ 57
7 ⎛ − 2 ⎞ ⎛ 7 ⎞⎛ − (− 2 ) ⎞
(iii) ×⎜ ⎟ = ⎜ ⎟⎜ ⎟
13 ⎝ − 5 ⎠ ⎝ 13 ⎠⎝ 5 ⎠
7 2 7 × 2 14
= × = =
13 5 13 × 5 65
⎛ 7 ⎞ ⎛ − 2 ⎞ 14
∴⎜ ⎟ × ⎜ ⎟=
⎝ 13 ⎠ ⎝ − 5 ⎠ 65
⎛3⎞ ⎛ 7 ⎞ 9 ⎛ 105 ⎞ ⎛ 87 ⎞ ⎛ 29 ⎞
(i) ⎜ ⎟ ÷ ⎜ ⎟ (ii) ÷⎜− ⎟ (iii) ⎜ ⎟ ÷ ⎜ ⎟
⎝ 4 ⎠ ⎝ 12 ⎠ 16 ⎝ 12 ⎠ ⎝ 27 ⎠ ⎝ 18 ⎠
⎛3⎞ ⎛ 7 ⎞
Solution: (i) ⎜ ⎟÷⎜ ⎟
⎝ 4 ⎠ ⎝ 12 ⎠
Notes ⎛ 3 ⎞ ⎛ 12 ⎞ ⎡ 7 12 ⎤
= ⎜ ⎟×⎜ ⎟ ⎢⎣Reciprocal of 12 is 7 ⎥⎦
⎝4⎠ ⎝ 7 ⎠
3 × 12 3 × 3 × 4 9
= = =
4× 7 7×4 7
⎛3⎞ ⎛ 7 ⎞ 9
∴⎜ ⎟ ÷ ⎜ ⎟ =
⎝ 4 ⎠ ⎝ 12 ⎠ 7
⎛ 9 ⎞ ⎛ − 105 ⎞
(ii) ⎜ ⎟÷⎜ ⎟
⎝ 16 ⎠ ⎝ 2 ⎠
⎛9⎞ ⎛ 2 ⎞ ⎡ - 105 2 ⎤
⎜ ⎟×⎜ ⎟ ⎢⎣Reciprocal of 2 is - 105 ⎥⎦
⎝ 16 ⎠ ⎝ − 105 ⎠
9× 2 3× 3× 2
=− =−
2 × 8 × 3 × 35 2 × 8 × 3 × 35
−3 −3
= =
8 × 35 280
⎛ 9 ⎞ ⎛ − 105 ⎞ − 3
∴⎜ ⎟ ÷ ⎜ ⎟=
⎝ 16 ⎠ ⎝ 2 ⎠ 280
⎛ 87 ⎞ ⎛ 29 ⎞
(iii) ⎜ ⎟ ÷⎜ ⎟
⎝ 27 ⎠ ⎝ 18 ⎠
⎛ 87 ⎞ ⎛ 18 ⎞ 87 18 29 × 3 × 2 × 9 2
= ⎜ ⎟×⎜ ⎟ = × = =
⎝ 27 ⎠ ⎝ 29 ⎠ 27 29 9 × 3 × 29 1
⎛ 87 ⎞ ⎛ 29 ⎞ 2
∴⎜ ⎟ ÷ ⎜ ⎟ =
⎝ 27 ⎠ ⎝ 18 ⎠ 1
⎛ 7 −5⎞ 3 ⎛ 7 3⎞ ⎛ 3⎞
(i) ⎜ − + ⎟+ (ii) ⎜ + ⎟ + ⎜ − ⎟
⎝ 8 12 ⎠ 16 ⎝ 3 4⎠ ⎝ 5⎠
4. Subtract:-
7 13 7 5 3 9
(i) from (ii) from − (iii) from
15 15 3 3 7 24
5. Simplify:-
⎛ 1 7 1⎞ 5 13 3
(i) ⎜ 3 + − 2 ⎟ (ii) + −6
⎝ 5 5 6⎠ 2 4 4
6. Multiply:-
2 5 3 − 33 − 11 − 27
(i) by (ii) − by (iii) by
11 6 11 35 3 77
7. Divide:
1 1 −7 −4 35 −7
(i) by (ii) by (iii) by
2 4 4 5 33 22
8. Simplify the following:
⎛ 2 7 ⎞ 8 37 ⎡⎛ 3 2 ⎞ 1 ⎤
(i)⎜ + ⎟ × ÷ (ii) ⎢⎜ − ⎟ ÷ ⎥ × 21
⎝ 3 8 ⎠ 25 15 ⎣⎝ 4 3 ⎠ 4 ⎦
16 −3
9. Divide the sum of and by their difference.
7 14
13 39
10. A number when multiplied by gives . Find the number.
3 12
12 − 27 13
(i) (ii) (iii)
5 25 16
Solution: i) Using long division, we get
2 .4
5 12.0 12
10 Hence, = 2.4
2.0 5
2.0
×
ii) 25 − 27 (–1.08
25 – 27
200 Hence, = –1.08
25
200
×
0.8125
iii) 16 13.0000
128
20
16 13
Hence, = 0.8125
40 16
32
80
80
×
From the above examples, it can be seen that the division process stops after a finite
number of steps, when the remainder becomes zero and the resulting decimal number has
a finite number of decimal places. Such decimals are known as terminating decimals.
Note: Note that in the above division, the denominators of the rational numbers had only
2 or 5 or both as the only prime factors.
12 12 × 2 24
Alternatively, we could have written as = = 2.4 and similarly for the others
5 5× 2 10
7 2 5
(a ) (b ) (c ) Notes
3 7 11
2.33
Solution: (a) 3 7.00 Here the remainder 1 repeats.
6 ∴ The decimal is not a terminating decimal
1.0
9 7
= 2.333... or 2.3
1.0 3
9
1.00
0.28571428
(b) 7 2.000
Here when the remainder is 3, the digit after
14
that start repeating
60
56 2
= 0.285714
40 7
35
50 Note: A bar over a digit or a group of digits
implies that digit or that group of digits starts
49
repeating itself indefinitely.
10
7
30
28
20
14
60
56
4
0.454
(c) 11 5.00
Here again when the remainder is 5, the digits
44
after 5 start repeating
60
55 5
∴ = 0.45
50 11
44
50...
Notes Thus, we see from examples 1.13 and 1.14 that the decimal representation of a rational
number is
(i) either a terminating decimal (and the remainder is zero after a finite number of steps)
(ii) or a non-terminating repeating decimal (where the division will never end)
∴ Thus, a rational number is either a terminating decimal or a non-terminating repeating
decimal
p
Example 1.15: Express (i) 0.48 and (ii) 0.1357 in form
q
48 12
Solution: (i) 0.48 = =
100 25
1375 55 11
(ii) 0.1375 = = =
10000 400 80
p
Example 1.16: Express (i) 0.666... (ii) 0.374374... in form
q
Solution: (i) Let x = 0.666... (A)
∴ 10 x = 6.666... (B)
2
(B) – (A) gives 9 x = 6 or x =
3
374
or x =
999
374
∴ 0.374374374 ... =
999
Note: The non- terminating recurring decimals like 0.374374374... are written as 0.374 .
The bar on the group of digits 374 indicate that the group of digits repeats again and again.
2 5 25
(i) (ii) (iii)
3 7 11
p
3. Represent the following decimals in the form .
q
(a) (i) 2.3 (ii) – 3.12 (iii) –0.715 (iv) 8.146
(b) (i) 0.333 (ii) 3.42 (iii) – 0.315315315...
3 6
Example 1.17: Find a rational number between and
4 5
1⎛3 6⎞
Solution: Let us try to find the number ⎜ + ⎟
2⎝4 5⎠
1 ⎛ 15 + 24 ⎞ 39
= ⎜ ⎟=
2 ⎝ 20 ⎠ 40
Notes 3 3 × 10 30
Now = =
4 4 × 10 40
6 6 × 8 48
and = =
5 5 × 8 40
30 39 48
Obviously < <
40 40 40
39 3 6
i.e. is a rational number between the rational numbers and .
40 4 5
3 39 6
Note: = 0.75, = 0.975 and = 1.2
4 40 5
3 39 6
or < <
4 40 5
∴ This can be done by either way:
(i) reducing each of the given rational number with a common base and then taking
their average
or (ii) by finding the decimal expansions of the two given rational numbers and then
taking their average.
The question now arises, “How many rationals can be found between two given rationals?
Consider the following examples.
1 3
Example 1.18: Find 3 rational numbers between and .
2 4
1 1× 8 8
Solution: = =
2 2 × 8 16
3 3 × 4 12
and = =
4 4 × 4 16
8 9 10 11 12
As < < < <
16 16 16 16 16
9 10 11 1 3
, and between and
16 16 16 2 4
Notes
In fact, we can find any number of rationals between two given numbers.
1 50 50
Again = =
2 2 × 50 100
3 3 × 25 75
= =
4 4 × 25 100
50 51 52 53 72 73 74 75
As < < < < ..... < < < < < .... (i)
100 100 100 100 100 100 100 100
1 3
∴ we have been able to find 24 rational numbers between and as given in
2 4
(i) above.
We can continue in this way further.
Note: From the above it is clear that between any two rationals an infinite number of
rationals can be found.
3 4 3 1
(i) and (ii) 5 and 6 (iii) − and
4 3 4 3
2 1 2 1
(i) – and (ii) − and −
3 2 3 4
one unit
It can be proved that 2 is not a rational number, as there
is no rational, whose square is 2, [Proof is beyond the scope
of this lesson]. D one unit C
We conclude that we can not exactly measure the lengths of all line-segments using rationals,
in terms of a given unit of length. Thus, the rational numbers are inadequate to measure all
lengths in terms of a given unit. This inadequacy necessitates the extension of rational
numbers to irrationals (which are not rational)
We have also read that corresponding to every rational number, there corresponds a point
on the number line. Consider the converse of this statement:
Given a point on the number line, will it always correspond to a rational number? The
answer to this question is also “No”. For clarifying this, we take the following example.
On the number line take points O, A, B, C and D representing rational 0, 1, 2, –1 and –2
respectively. At A draw AA′⊥ to OA such that AA′ = 1 unit
D C O A B
P Notes
–2 –1 0 1 2
As 2 is irrational, we conclude that there are points on the number line (like P) which
are not represented by a rational number. Similarly, we can show that we can have points
like 3 , 2 3 , 5 2 etc, which are not represented by rationals.
∴ The number line, consisting of points corresponding to rational numbers, has gaps on it.
Therefore, the number line consists of points corresponding to rational numbers and irrational
numbers both.
We have thus extended the system of rational numbers to include irrational numbers also.
The system containing rationals and irrationals both is called the Real Number System.
The system of numbers consisting of all rational and irrational numbers is called the system
of real numbers.
1. Write the first three digits of the decimal representation of the following:
2, 3, 5
2 3
(i) (ii) 1 + 2 (iii)
2 2
3+2
Solution: Consider the number
2
3 1.732
= 1+ ≈ 1+ = 1.866
2 2
3+2
∴ ≈ 1.866 lies betwen 3 (≈ 1.732) and 2
2
3+2
∴ The required irrational number is
2
2. Can you state the number of irrationals between 1 and 2? Illustrate with three examples.
Solution: We look up at the third place after the decimal point. In this case it 8, which is
more than 5. So the approximate value of 2.71832, upto two places of decimal is 2.72.
Example 1.22: Find the approximate value of 12.78962 correct upto 3 places of decimals.
LET US SUM UP
• Recall of natural numbers, whole numbers, integers with four fundamental operations
is done.
• Representation of above on the number line.
• Extension of integers to rational numbers - A rational number is a number which can
be put in the form p/q, where p and q are integers and q ≠ 0.
• When q is made positive and p and q have no other common factor, then a rational
number is said to be in standard form or lowest form.
• Two rational numbers are said to be the equivalent form of the number if standard
forms of the two are same.
• The rational numbers can be represented on the number line.
• Corresponding to a rational number, there exists a unique point on the number line.
• The rational numbers can be compared by
• reducing them with the same denominator and comparing their numerators.
• when represented on the number line, the greater rational number lies to the right
of the other.
TERMINAL EXERCISE
1. From the following pick out:
(i) natural numbers
(ii) integers which are not natural numbers
(iii) rationals which are not natural numbers
(iv) irrational numbers
6 −3 3 11
− 3,17 , , ,0, − 32 , , , 2 , 2 + 3
7 8 14 6
2. Write the following integers as rational numbers:
(i) – 14 (ii) 13 (iii) 0 (iv) 2
(v) 1 (vi) –1 (vii) –25
3. Express the following rationals in lowest terms:
6 14 − 17 13
, , ,
8 21 153 273
4. Express the following rationals in decimal form:
(i ) 11 (ii)
8
(iii)
14
(iv)
15
(v)
98
80 25 8 6 35
15 7
(vi) (vii) − (viii )115 (ix ) − 17 (x)
126
7 6 11 13 36
p
5. Represent the following decimals in form: Notes
q
(v) 0.415415415...
6. Find a rational number betwen the following rational numbers:
3 −3
(i) and (ii) 0.27 and 0.28 (iii) 1.32 and 1.34
4 4
8. Write the rational numbers corresponding to the points O, P, Q, R, S and T on the
number line in the following figure:
R S O P Q T
–5 –4 –3 –2 –1 0 1 2 3 4 5 6 7 8 9
3 −7 7 5 3 7 9 2 18 7
(i) , (ii) − , (iii) , (iv) , (v) ,−
5 5 9 9 5 3 5 3 7 6
10. Find the product of the following rationals:
3 7 19 2 15 − 14
(i) , (ii) , (iii) ,
5 3 5 3 7 5
11. Write an irrational number between the following pairs of numbers:
12. How many rational numbers and irrational numbers lie between the numbers 2 and 7?
13. Find the approximate value of the following numbers correct to 2 places of decimals:
(i) 0.338 (ii) 3.924 (iii) 3.14159 (iv) 3.1428
3
(i) (ii) 2 + 2 (iii) 1.7326 (iv) 0.9999...
4
Notes
15. Simplify the following as irrational numbers. The first one is done for you.
(i) 12 3 + 5 3 − 7 3 = 3 [12 + 5 − 7 ] = 10 3
(ii) 3 2 − 2 8 + 7 2
(iii) 3 2 × 2 3 × 5 6
( )
(iv) [ 8 × 3 2 × 6 2 ] ÷ 36 2
−3 5 12 − 3 15
Rational Numbers: 4, , ,−36, , , , −6
4 6 7 8 7
7 3 5 3 4
2. (i) − ,− ,−15,0. ,− ,−
4 7 17 4 3
7 3 5 3 4
(ii) − ,− ,−15, ,− ,−
4 7 17 4 3
7 3 5 3 4
(iii) − ,− , ,− ,−
4 7 17 4 3
(iv) All are rational numbers.
16 1
3. (i) , rational (ii) − , rational (iii) –21, integer and rational
3 2
(iv) zero, whole number, integer and rational (v) 4, All
10 8
(vi) , rational (vii) , rational
7 3
4. (i) 2 (ii) – 8 (iii) 1
5 −6 2 3
5. , ,
7 7 27
15 27 − 6 Notes
6. − 10, , ,
5 9 −2
2 4 6 8 5 10 15 20 17 34 51 68
7. (i) = = = (ii) − = − = − = − (iii) = = =
5 10 15 20 6 12 18 24 3 6 9 12
8. (i) (ii) 0 3/4 1
2/5
–1 0 1 2
(iii) 1
0 1
2
3 2 7 3 −1 − 2 5 3
9. (a) > (b) > (c) > (d) >
4 3 9 5 2 3 11 7
3 7
(e) >−
2 6
1.2
9 4 1 1
1. (i) (ii) − (iii) (iv)
7 15 2 2
19 188 11
2. (i) (ii) (iii) −
6 63 35
53 149
3. (i) − (ii)
48 60
2 −3
4. (i) (ii) – 4 (iii)
5 56
73
5. (i) (ii) – 1
30
5 9 9
6. (i) (ii) (iii)
33 35 7
35 10
7. (i) 2 (ii) (iii) −
16 3
Notes 1
8. (i) (ii) 7
5
29
9.
35
3
10.
4
1.3
1. (i) 0.3875 (ii) 0.48 (iii) 1.5 (iv) 6.25 (v) 1.4
23 78 143 4073
3. (a) (i) (ii) − (iii) − (iv)
10 25 200 500
1 113 35
(b) (i) (ii) (iii) –
3 33 111
1.4
25 −5
1. (i) (ii) 5.5 (iii)
24 24
2. (i) 0.2 and 0.3 (ii) – 0.30, – 0.35
3. (i) 0.271, 0.275, 0, 281, 0.285, 0.291
(ii) 7.315, 7.320 7.325, 7.330, 7.331
(iii) 21.75, 22.75, 23.75, 24.75, 25.75
(iv) 1.0011, 1.0012, 1.0013, 1.0014, 1.0015
Note: Can be other answers as well.
1.5
1. 1.414, 1.732, 2.236
3/2
(iii)
0 1 2
1.6
2+ 3
1. (i) 5 (ii) 3 + 1 (iii)
2
2. Infinitely many:
1.0001, 1.0002, ....., 1.0010, 1.0011,....., 1.0020, 1.0021, .....
1.7
1. (i) 0.778 (ii) 7.326 (iii) 1.012 (iv) 3.143 (v) 1.141
6 −3 3 11
Rationals but not natural numbers: − 3, , ,0,−32, ,
7 8 14 6
14 13 0 2
2. (i) − (ii) (iii) (iv)
1 1 1 1
1 −1 − 25
(v) (vi) (vii)
1 1 1
3 2 1 1
3. , ,− ,
4 3 9 21
4. (i) 0.1375 (ii) 0.32 (iii) 1.75 (iv) 2.5 (v) 2.8
(vi) 2.142857 (vii) − 1.166 (viii) 10.45 (ix) − 1.307692 (x) 3.5
13
6. (i) (ii) – 2.5 (iii) zero
16
7. (i) 0.50, 0.25, 0.00 (ii) 0.271, 0.274, 0.277 (iii) 1.325. 1.33, 1.335
8. (i) R: – 3.8 (ii) S: – 0.5 (iii) O: 0.00 (iv) S: – 0.33 (v) Q: 3.5
(vi) T: 7.66
4 2 44 37 59
9. (i) − (ii) − (iii) (iv) (v)
5 9 15 15 42
7 38
10. (i) (ii) (iii) – 6
5 15
2
11. (i) 3 (ii) 1 + 3 (iii) 3 (iv)
2
12. Infinitely many
13. (i) 0.34 (ii) 3.92 (iii) 3.14 (iv) 3.14
14. (i) 0.75 (ii) 3.414 (iii) 1.733 (iv) 1.000
Notes
2
EXPONENTS AND RADICALS
We have learnt about multiplication of two or more real numbers in the earlier lesson. You
can very easily write the following
4 × 4 × 4 = 64,11 × 11 × 11 × 11 = 14641 and
2 × 2 × 2 × 2 × 2 × 2 × 2 × 2 = 256
Think of the situation when 13 is to be multiplied 15 times. How difficult is it to write?
13 × 13 × 13 ×.................15 times?
This difficulty can be overcome by the introduction of exponential notation. In this lesson,
we shall explain the meaning of this notation, state and prove the laws of exponents and
learn to apply these. We shall also learn to express real numbers as product of powers of
prime numbers.
In the next part of this lesson, we shall give a meaning to the number a1/q as qth root of a.
We shall introduce you to radicals, index, radicand etc. Again, we shall learn the laws of
radicals and find the simplest form of a radical. We shall learn the meaning of the term
“rationalising factor’ and rationalise the denominators of given radicals.
OBJECTIVES
After studying this lesson, you will be able to
• write a repeated multiplication in exponential notation and vice-versa;
• identify the base and exponent of a number written in exponential notation;
• express a natural number as a product of powers of prime numbers uniquely;
• state the laws of exponents;
p
• explain the meaning of a0, a–m and a q ;
1 1
• rationalise the denominator of a given surd of the form a + b x and x+ y ,
where x and y are natural numbers and a and b are integers;
• simplify expressions involving surds.
(i ) ⎛⎜ 2 ⎞⎟ ⎛ 3⎞
(ii) ⎜ − ⎟
⎝7⎠ ⎝ 5⎠
⎛ 2 ⎞ 2 2 2 (2 )
3 3
8
Solution: (i) ⎜ ⎟ = × × = 3=
⎝ 7 ⎠ 7 7 7 (7 ) 343
⎛ 3 ⎞ ⎛ 3 ⎞⎛ 3 ⎞⎛ 3 ⎞⎛ 3 ⎞ (− 3)
4 4
81
(ii) ⎜ − ⎟ = ⎜ − ⎟⎜ − ⎟⎜ − ⎟⎜ − ⎟ = =
⎝ 5 ⎠ ⎝ 5 ⎠⎝ 5 ⎠⎝ 5 ⎠⎝ 5 ⎠ (5)
5
625
Example 2.2: Write the following in exponential form:
(i) (–5) × (–5) × (–5) × (–5) × (–5) × (–5) × (–5)
Example 2.3: Express each of the following in exponential notation and write the base
and exponent in each case.
Notes 125
(i) 4096 (ii) (iii) – 512
729
Solution: (i) 4096 = 4 × 4 × 4 × 4 × 4 × 4 Alternatively 4096 = (2)12
= (4)6 Base = 2, exponent =12
Here, base = 4 and exponent = 6
3
125 5 5 5 ⎛5⎞
(ii) = × × =⎜ ⎟
729 9 9 9 ⎝ 9 ⎠
⎛5⎞
Here, base = ⎜ ⎟ and exponent = 3
⎝9⎠
(iii) 512 = 2 × 2 × 2 × 2 × 2 × 2 × 2 × 2 × 2 = 29
Here, base = 2 and exponent = 9
Example 2.4: Simplify the following:
3 4
⎛3⎞ ⎛4⎞
⎜ ⎟ ×⎜ ⎟
⎝2⎠ ⎝3⎠
3
⎛3⎞ 3 3 3 33
Solution: ⎜ ⎟ = × × = 3
⎝2⎠ 2 2 2 2
4
⎛4⎞ 44
Similarly ⎜ ⎟ = 4
⎝3⎠ 3
3 4
⎛3⎞ ⎛4⎞ 33 4 4
×
⎜ ⎟ ⎜ ⎟ = 3× 4
⎝2⎠ ⎝3⎠ 2 3
33 16 ×16 32
= × =
8 34 3
Example 2.5: Write the reciprocal of each of the following and express them in exponential
form:
2 9
⎛3⎞ ⎛ 5⎞
5
(i) 3 (ii) ⎜ ⎟ (iii) ⎜ − ⎟
⎝4⎠ ⎝ 6⎠
2
⎛3⎞ 32
(ii) ⎜ ⎟ = 2
⎝4⎠ 4
2 2
⎛3⎞ 42 ⎛ 4 ⎞
∴ Reciprocal of ⎜ ⎟ = 2 = ⎜ ⎟
⎝4⎠ 3 ⎝3⎠
(− 5)9
9
⎛ 5⎞
(iii) ⎜− ⎟ =
⎝ 6⎠ 69
9 9
⎛ 5⎞ −6 ⎛−6⎞
9
∴ Reciprocal of ⎜ − ⎟ = 9 = ⎜ ⎟
⎝ 6⎠ 5 ⎝ 5 ⎠
p
From the above example, we can say that if is any non-zero rational number and m is
q
m m
⎛ p⎞ ⎛q⎞
any positive integer, then the reciprocal of ⎜⎜ ⎟⎟ is ⎜⎜ ⎟⎟ .
⎝q⎠ ⎝ p⎠
⎛3⎞ ⎛3⎞
(ii) ⎜ ⎟ × ⎜ ⎟ × .... 10 times
⎝4⎠ ⎝4⎠
⎛ 5⎞ ⎛ 5⎞
(iii) ⎜ − ⎟ × ⎜ − ⎟ × .... 20 times
⎝ 7⎠ ⎝ 7⎠
8
⎛ 2⎞
(i) (–3) 5 4
(ii) (7) (iii) ⎜ − ⎟
⎝ 11 ⎠
Notes 3. Evaluate each of the following
4 4 3
⎛3⎞ ⎛ –2⎞ ⎛ 3⎞
(i) ⎜ ⎟ (ii) ⎜ ⎟ (iii) ⎜ – ⎟
⎝7⎠ ⎝ 9 ⎠ ⎝ 4⎠
5 6
⎛7⎞ ⎛3⎞
(i) ⎜ ⎟ × ⎜ ⎟
⎝3⎠ ⎝7⎠
2 2
⎛ 5⎞ ⎛ 3⎞
(ii) ⎜ − ⎟ ÷ ⎜ − ⎟
⎝ 6⎠ ⎝ 5⎠
4
⎛ 3⎞
5
(i) 3 (ii) (–7) 4
(iii) ⎜ − ⎟
⎝ 5⎠
1331 243
(iv) (v) −
4096 32
(i) 32 × 33 = (3 × 3) × (3 × 3 × 3) = (3 × 3 × 3 × 3 × 3)
= 35 = 32 + 3
(ii) (–7)2 × (–7)4 = [(–7) × (–7)] × [(–7) × (–7) × (–7) × (–7)]
= [ (–7) × (–7) × (–7) × (–7) × (–7) × (–7)]
= (–7)6 = (–7)2+4
3 4
⎛3⎞ ⎛3⎞ ⎛3 3 3⎞ ⎛3 3 3 3⎞
(iii) ⎜ ⎟ × ⎜ ⎟ = ⎜ × × ⎟ × ⎜ × × × ⎟
⎝4⎠ ⎝4⎠ ⎝4 4 4⎠ ⎝4 4 4 4⎠
⎛3 3 3 3 3 3 3⎞
=⎜ × × × × × × ⎟
⎝4 4 4 4 4 4 4⎠
Notes 7 3+ 4
⎛3⎞ ⎛3⎞
= ⎜ ⎟ =⎜ ⎟
⎝4⎠ ⎝4⎠
(iv) a3 × a4 = (a × a × a) × (a × a × a × a) = a7 = a3+4
From the above examples, we observe that
Law 1: If a is any non-zero rational number and m and n are two positive integers, then
am × an = am+n
3 5
⎛ 3⎞ ⎛ 3⎞
Example 2.8: Evaluate ⎜ − ⎟ × ⎜ − ⎟ .
⎝ 2⎠ ⎝ 2⎠
3
Solution: Here a = − , m = 3 and n = 5.
2
3 5 3+ 5 8
⎛ 3⎞ ⎛ 3⎞ ⎛ 3⎞ ⎛ 3 ⎞ 6561
∴ ⎜− ⎟ ×⎜− ⎟ = ⎜− ⎟ = ⎜− ⎟ =
⎝ 2⎠ ⎝ 2⎠ ⎝ 2⎠ ⎝ 2⎠ 256
75 7 × 7 × 7 × 7 × 7
(i) 7 ÷ 7 = 3 =
5 3 = 7 × 7 = 7 2 = 7 5−3
7 7×7×7
1
am ÷ an = m−n
a
6 9
⎛3⎞ ⎛3⎞
Example 2.11: Find the value of ⎜ ⎟ ÷ ⎜ ⎟
⎝7⎠ ⎝7⎠
3
Solution: Here a = , m = 6 and n = 9.
7
6 9 1
⎛ 3 ⎞ ⎛ 3 ⎞ ⎛ 3 ⎞ 9−6
∴ ⎜ ⎟ ÷⎜ ⎟ = ⎜ ⎟
⎝7⎠ ⎝7⎠ ⎝7⎠
7 3 343
= =
33 27
Let us consider the following:
(i) (3 )
3 2
= 33 × 33 = 33+3 = 36 = 33×2
5
⎡⎛ 3 ⎞ 2 ⎤ ⎛ 3 ⎞ 2 ⎛ 3 ⎞ 2 ⎛ 3 ⎞ 2 ⎛ 3 ⎞ 2 ⎛ 3 ⎞ 2
(ii) ⎢⎜ ⎟ ⎥ = ⎜ ⎟ × ⎜ ⎟ × ⎜ ⎟ × ⎜ ⎟ × ⎜ ⎟
⎢⎣⎝ 7 ⎠ ⎥⎦ ⎝ 7 ⎠ ⎝ 7 ⎠ ⎝ 7 ⎠ ⎝ 7 ⎠ ⎝ 7 ⎠
2+ 2+ 2+ 2+ 2 10 2×5
⎛3⎞ ⎛3⎞ ⎛3⎞
⎜ ⎟ =⎜ ⎟ =⎜ ⎟
⎝7⎠ ⎝7⎠ ⎝7⎠
Notes
From the above two cases, we can infer the following:
Law 4: If a is any non-zero rational number and m and n are two positive integers, then
(a )
m n
= a mn
Let us consider an example.
3
⎡⎛ 2 ⎞ 2 ⎤
Example 2.12: Find the value of ⎢⎜ ⎟ ⎥
⎣⎢⎝ 5 ⎠ ⎦⎥
3
⎡⎛ 2 ⎞ 2 ⎤ ⎡ 2 ⎤ 2×3 ⎛ 2 ⎞6 64
Solution: ⎢⎜ ⎟ ⎥ = ⎢ ⎥ = ⎜ ⎟ =
⎢⎣⎝ ⎠ ⎥⎦ ⎣ 5 ⎦
5 ⎝ 5 ⎠ 15625
1
= n−m , if n > m
a
Let us consider the case, when m = n
∴ a m ÷ a m = a m−m
am
⇒ m = a0
a
⇒ 1 = a0
0
⎛ −3⎞
(ii) Again using a = 1, we get ⎜0 ⎟ = 1.
⎝ 4 ⎠
Notes
3 2 1 2 3
⎛3⎞ ⎛3⎞ ⎛ 7⎞ ⎛ 7⎞ ⎛ 7⎞
2
(i) (7) ×(7) 3
(ii) ⎜ ⎟ × ⎜ ⎟ (iii) ⎜ − ⎟ × ⎜ − ⎟ × ⎜ − ⎟
⎝4⎠ ⎝4⎠ ⎝ 8⎠ ⎝ 8⎠ ⎝ 8⎠
8 2 18 3
⎛3⎞ ⎛3⎞ ⎛−7⎞ ⎛−7⎞
(i) (− 7 ) ÷ (− 7 )
9 7
(ii) ⎜ ⎟ ÷ ⎜ ⎟ (iii) ⎜ ⎟ ÷⎜ ⎟
⎝4⎠ ⎝4⎠ ⎝ 3 ⎠ ⎝ 3 ⎠
3. Simplify and express the result in exponential form:
2 5
⎡⎛ 3 ⎞3 ⎤ ⎡⎛ 5 ⎞3 ⎤
( )
(i) 26 3
(ii) ⎢⎜ ⎟ ⎥
⎢⎣⎝ 4 ⎠ ⎥⎦
(iii) ⎢⎜ − ⎟ ⎥
⎢⎣⎝ 9 ⎠ ⎥⎦
5 0 0 3
⎛ 11 ⎞ ⎛ 15 ⎞ ⎛ 7⎞ ⎛ 7⎞
(iv) ⎜ ⎟ × ⎜ ⎟ (v) ⎜ − ⎟ × ⎜ − ⎟
⎝3⎠ ⎝7⎠ ⎝ 11 ⎠ ⎝ 11 ⎠
4. Which of the following statements are true?
5 2 7
⎛3⎞ ⎛3⎞ ⎛3⎞
3
(i) 7 × 7 = 7 3 6
(ii) ⎜ ⎟ × ⎜ ⎟ = ⎜ ⎟
⎝ 11 ⎠ ⎝ 11 ⎠ ⎝ 11 ⎠
4 2
⎡⎛ 4 ⎞5 ⎤ ⎛ 4 ⎞9 ⎡⎛ 3 ⎞ 6 ⎤ ⎛ 3 ⎞8
(iii) ⎢⎜ ⎟ ⎥ = ⎜ ⎟ (iv) ⎢⎜ ⎟ ⎥ = ⎜ ⎟
⎢⎣⎝ 9 ⎠ ⎥⎦ ⎝ 9 ⎠ ⎢⎣⎝ 19 ⎠ ⎥⎦ ⎝ 19 ⎠
0 2
⎛3⎞ ⎛ 3⎞ 9
(v) ⎜ ⎟ = 0 (vi) ⎜ − ⎟ = −
⎝ 11 ⎠ ⎝ 2⎠ 4
5 0 5
⎛ 8 ⎞ ⎛7⎞ ⎛ 8 ⎞
(vii) ⎜ ⎟ × ⎜ ⎟ = ⎜ ⎟
⎝ 15 ⎠ ⎝ 6 ⎠ ⎝ 15 ⎠
1
Notes i) We know that the reciprocal of 5 is . We write it as 5–1 and read it as 5 raised to
5
power –1.
1
ii) The reciprocal of (–7) is − . We write it as (–7)–1 and read it as (–7) raised to the
7
power –1.
1
iii) The reciprocal of 52 = . We write it as 5–2 and read it as ‘5 raised to the power (–2)’.
52
From the above all, we get
If a is any non-zero rational number and m is any positive integer, then the reciprocal of am
⎛ 1 ⎞
⎜ i.e. m ⎟ is written as a–m and is read as ‘a raised to the power (–m)’. Therefore,
⎝ a ⎠
1
m
= a −m
a
Let us consider an example.
Example 2.14: Rewrite each of the following with a positive exponent:
−2 −7
⎛3⎞ ⎛ 4⎞
(i) ⎜ ⎟ (ii) ⎜ − ⎟
⎝8⎠ ⎝ 7⎠
Solution:
−2 2
⎛ 3⎞ 1 1 82 ⎛ 8 ⎞
(i) ⎜ ⎟ = 2
= = =⎜ ⎟
⎝8⎠ ⎛ 3⎞ 32 32 ⎝ 3 ⎠
⎜ ⎟
⎝8⎠ 82
−7 7
⎛ 4⎞ 1 77 ⎛ 7⎞
(ii) ⎜ − ⎟ = = = ⎜− ⎟
⎝ 7⎠ ⎛ 4⎞
7
(− 4) ⎝ 4 ⎠
7
⎜− ⎟
⎝ 7⎠
From the above example, we get the following result:
p
If is any non-zero rational number and m is any positive integer, then
q
−m m
⎛ p⎞ qm ⎛ q ⎞
⎜⎜ ⎟⎟ = m = ⎜⎜ ⎟⎟ .
⎝q⎠ p ⎝ p⎠
−4 3 3 3– 4
⎛3⎞ ⎛3⎞ 1 ⎛3⎞ 3
(i) ⎜ ⎟ × ⎜ ⎟ = 4
×⎜ ⎟ =
⎝5⎠ ⎝5⎠ ⎛3⎞ ⎝5⎠ 5
⎜ ⎟
⎝5⎠
−2 −3 − 2 −3
⎛ 2⎞ ⎛ 2⎞ 1 1 1 ⎛ 2⎞
(ii) ⎜ − ⎟ × ⎜ − ⎟ = 2
× 3
= 2+3
= ⎜− ⎟
⎝ 3⎠ ⎝ 3⎠ ⎛ 2⎞ ⎛ 2⎞ ⎛ 2⎞ ⎝ 3⎠
⎜− ⎟ ⎜− ⎟ ⎜− ⎟
⎝ 3⎠ ⎝ 3⎠ ⎝ 3⎠
−3 −7 7 7 −3
⎛ 3⎞ ⎛ 3⎞ 1 1 1 ⎛ 3⎞ ⎛ 3⎞
(iii) ⎜ − ⎟ ÷ ⎜ − ⎟ = 3
÷ 7
= 3
×⎜− ⎟ = ⎜− ⎟
⎝ 4⎠ ⎝ 4⎠ ⎛ 3⎞ ⎛ 3⎞ ⎛ 3⎞ ⎝ 4⎠ ⎝ 4⎠
⎜− ⎟ ⎜− ⎟ ⎜− ⎟
⎝ 4⎠ ⎝ 4⎠ ⎝ 4⎠
3 3
⎛ ⎛ 2 ⎞ − 2 ⎞ ⎡⎛ 7 ⎞ 2 ⎤ ⎛ 7 ⎞ 6 ⎛ 2 ⎞ −6 ⎛ 2 ⎞ − 2×3
(iv) ⎜ ⎜ ⎟ ⎟ = ⎢⎜ ⎟ ⎥ = ⎜ ⎟ = ⎜ ⎟ = ⎜ ⎟
⎜ ⎝ 7 ⎠ ⎟ ⎢⎝ 2 ⎠ ⎥ ⎝ 2 ⎠ ⎝ 7 ⎠ ⎝7⎠
⎝ ⎠ ⎣ ⎦
Thus, from the above results, we find that laws 1 to 5 hold good for negative exponents
also.
∴ For any non-zero rational numbers a and b and any integers m and n,
1. am × an = am+n
2. am ÷ an = am–n if m > n
= an–m if n > m
3. (am)n = amn
4. (a × b)m = am × bm
4
⎛3⎞
−4 ⎡⎛ 3 ⎞ −3 ⎤
Notes (i) ⎜ ⎟ (ii) 125 × 12 −3 (iii) ⎢⎜ ⎟ ⎥
⎝7⎠ ⎢⎣⎝ 13 ⎠ ⎥⎦
5
⎛3⎞
4 ⎡⎛ 3 ⎞ 2 ⎤
(i) ⎜ ⎟
⎝7⎠
[ ]
(ii) (7 )2
5
(iii) ⎢⎜ − ⎟ ⎥
⎣⎢⎝ 4 ⎠ ⎦⎥
4. Simplify:
−3 7 −3 4 −4 −7
⎛3⎞ ⎛3⎞ ⎛ 2⎞ ⎛ 2⎞ ⎛ 7⎞ ⎛ 7⎞
(i) ⎜ ⎟ × ⎜ ⎟ (ii) ⎜ − ⎟ × ⎜ − ⎟ (iii) ⎜ − ⎟ ÷ ⎜ − ⎟
⎝2⎠ ⎝2⎠ ⎝ 3⎠ ⎝ 3⎠ ⎝ 5⎠ ⎝ 5⎠
m
⎛a⎞
(iv) a ÷ b = ⎜ ⎟
m m
⎝b⎠
(v) a–m × ao = am
q times
q
= aq = a
1
In other words, the qth power of a q = a or
1
in other words a q is the qth root of a and is written as q a . Notes
For example,
1 1 1 1 1 1 1 1 4
+ + +
74 ×74 ×74 ×74 = 74 4 4 4
= 7 4 = 71 = 7
1
or 7 4 is the fourth root of 7 and is written as 4 7 ,
Let us now define rational powers of a
If a is a positive real number, p is an integer and q is a natural number, then
p
q
aq = ap
We can see that
p p p p p p p p
+ + + .....q times .q
a × a × a ........× a = a
q q q q q q q
=a q
= ap
q times
p
q
∴a = a pq
Solution:
1 1
(625)4 = (5 × 5 × 5 × 5) 4 = (54 )4 = 5
1 1 4×
(i) 4
=5
Notes
2 2
( )
2 2 5×
(ii) (243)5 = (3 × 3 × 3 × 3 × 3) 5 = 35 5 =3 5
= 32 = 9
−3 −3
⎛ 16 ⎞ 4 ⎛ 2 × 2 × 2 × 2 ⎞ 4
(iii) ⎜ ⎟ = ⎜ ⎟
⎝ 81 ⎠ ⎝ 3× 3× 3× 3 ⎠
−3
⎛ −3 ⎞
⎡⎛ 2 ⎞ 4 ⎤ 4 ⎛ 2 ⎞ 4×⎜⎝ 4 ⎟⎠ ⎛ 2 ⎞ −3 ⎛ 3 ⎞ 3 27
= ⎢⎜ ⎟ ⎥ = ⎜ ⎟ =⎜ ⎟ =⎜ ⎟ =
⎣⎢⎝ 3 ⎠ ⎦⎥ ⎝3⎠ ⎝3⎠ ⎝2⎠ 8
1 1 3
−
⎛ 7 ⎞ 4 ⎛ 7 ⎞2 ⎛ 7 ⎞4
(ii) ⎜ ⎟ × ⎜ ⎟ × ⎜ ⎟
⎝8⎠ ⎝8⎠ ⎝8⎠
3 1 3
−
⎛ 13 ⎞ 4 ⎛ 13 ⎞ 4 ⎛ 13 ⎞ 2
(iii) ⎜ ⎟ × ⎜ ⎟ × ⎜ ⎟
⎝ 16 ⎠ ⎝ 16 ⎠ ⎝ 16 ⎠
2.7 SURDS
We have read in first lesson that numbers of the type 2 , 3 and 5 are all irrational
numbers. We shall now study irrational numbers of a particular type called radicals or
surds.
A surd is defined as a positive irrational number of the type n x , where it is not possible to
find exactly the nth root of x, where x is a positive rational number.
The number n
x is a surd if and only if Notes
In the surd n x , the symbol is called a radical sign. The index ‘n’ is called the order
of the surd and x is called the radicand.
Note: i) When order of the surd is not mentioned, it is taken as 2. For example, order
of ( )
7 = 2 7 is 2.
ii) 3
8 is not a surd as its value can be determined as 2 which is a rational.
ii) A surd, having rational factor other than 1 alongwith the irrational factor, is called a
mixed surd.
∴ 49 is not a surd.
Notes
(ii) 96 = 4 × 4 × 6 = 4 6
∴ 96 is an irrational number.
⇒ 96 is a surd.
(iii) 3
81 = 3 3 × 3 × 3 × 3 = 33 3 , which is irrational
∴ 3
81 is a surd.
(iv) 3
256 = 3 4 × 4 × 4 × 4 = 43 4
∴ 3 256 is irrational.
⇒ 3
256 is a surd
∴ (ii), (iii) and (iv) are surds.
Example 2.17: Find “index” and “radicand” in each of the following:
(i) [ a] = a
n
n
(ii) n
a n b = n ab
n
a n a
(iii) n
= Notes
b b
(i) 5 × 80 (ii) 2 15 ÷ 4 10
(iii) 3 4 × 3 16 (iv) 32 ÷ 27
∴ 5 × 80 is not a surd.
2 15 15
(ii) 2 15 ÷ 4 10 = =
4 10 2 10
15 15 3
= = = , which is irrational.
2 × 2 ×10 40 8
∴ 2 15 ÷ 4 10 is a surd.
(iii) 3
4 × 3 16 = 3 64 = 4 ⇒ It is not a surd.
32 32
(iv) 32 ÷ 27 = = , which is irrational
27 27
∴ 32 ÷ 27 is a surd.
1 1 1
(i) n
x. n y = xy or x .y = ( xy )n
n n n
1 1
n
x x xn ⎛ x ⎞n
(ii) =n or 1 = ⎜⎜ ⎟⎟
yn ⎝ ⎠
n y y y
1 1
⎛ 1 ⎞m 1
⎛ 1 ⎞n
(iii)
m n
x = mn x = n m
x or ⎜⎜ x n ⎟⎟ = x mn = ⎜⎜ x m ⎟⎟
⎝ ⎠ ⎝ ⎠
m m
( )
1
(iv) n
x m = x n or x m n = xn
p pn
( ) ( )
1 1
(v) n
x p = mn x pn or x p m = x m = x mn = x pn mn
Here, x and y are positive rational numbers and m, n and p are positive integers.
Let us illustrate these laws by examples:
1 1 1
(i) 3
3 3
8 = 3 × 8 = (24)3 = 3 24 = 3 3 × 8
3 3
1 1
(5)3 ⎛ 5 ⎞3
=⎜ ⎟ =3
5
(ii) 1
(9)3 ⎝9⎠ 9
1
3 ⎛ 1 ⎞3
1 1
(iii)
3 2
7 = 7 = ⎜⎜ 7 2 ⎟⎟ = 7 6 = 6 7 = 2×3 7 = 2
2 3
7
⎝ ⎠
Notes
3 9
( )
1
(iv) 5
43 = 4 3 5
= 4 = 4 = 15 49 = 3×5 43×3
5 15
For example 3
2 = 6 22 = 6 4
and 4
3 = 8 32 = 8 9
3
(i) 2 7 (ii) 44 7 (iii) 32
4
Solution:
3 9
(iii) 32 = 32 × = 18 , which is a pure surd.
4 16
Example 2.21: Express as a mixed surd in the simplest form:
Solution:
(i) 128 = 64 × 2 = 8 2 ,
(ii) 6
320 = 6 2 × 2 × 2 × 2 × 2 × 2 × 5
(iii) 3
250 = 3 5 × 5 × 5 × 2 = 53 2 , which is a mixed surd.
As in rational numbers, surds are added and subtracted in the same way.
For example, 5 3 + 17 3 = (5 + 17 ) 3 = 22 3
and 12 5 − 7 5 = [12 − 7 ] 5 = 5 5
Notes
For adding and subtracting surds, we first change them to similar surds and then perform
the operations.
= 5 × 5 × 2 + 12 ×12 × 2
= 5 2 + 12 2 = 2 (5 + 12 ) = 17 2
ii) 98 − 18
= 7 × 7 × 2 − 3× 3× 2
= 7 2 − 3 2 = (7 − 3) 2 = 4 2
(i) 4 6 + 2 54
(ii) 45 6 − 3 216
Solution: (i) 4 6 + 2 54
= 4 6 + 2 3× 3× 6
= 4 6 + 6 6 = 10 6
(ii) 45 6 − 3 216
= 45 6 − 3 6 × 6 × 6
= 45 6 − 18 6
= 27 6
24 45 − 16 20 + 245 − 47 5 = 0
Solution: 24 45 − 16 20 + 245 − 47 5
= 24 3 × 3 × 5 − 16 2 × 2 × 5 + 7 × 7 × 5 − 47 5
= 72 5 − 32 5 + 7 5 − 47 5
Notes
= 5 [72 − 32 + 7 − 47 ]
= 5 × 0 = 0 = RHS
83 128 = 83 4 × 4 × 4 × 2 = 323 2
33 54 = 33 3 × 3 × 3 × 2 = 93 2
4
32 = 24 2
∴ Required expression
= 403 2 + 323 2 − 93 2 + 24 2
= (40 + 32 − 9)3 2 + 24 2
= 633 2 + 24 2
Two surds can be multiplied or divided if they are of the same order. We have read that the
order of a surd can be changed by multiplying or dividing the index of the surd and index
Notes
of the radicand by the same positive number. Before multiplying or dividing, we change
them to the surds of the same order.
Let us take some examples:
3 = 6 33 = 6 27
3
2 =6 4
∴ 3 × 3 2 = 6 27 × 6 4 = 6 108
3 6 27 6 27
and 3
= =
2 64 4
Let us consider an example:
Example 2.25:(i) Multiply 53 16 and 113 40 .
= 5 ×11× 3 2 × 2 × 2 × 2 × 3 2 × 2 × 2 × 5
= 55 × 2 × 23 2 3
5
= 220 3 10
2 50 × 32 × 2 72
Solution: 2 50 = 2 5 × 5 × 2 = 10 2
32 = 2 × 2 × 2 × 2 × 2 = 4 2
Notes
2 72 = 2 × 6 2 = 12 2
∴ Given expression
= 10 2 × 4 2 × 12 2
= 960 2
1 3
1
Example 2.27: Which is greater or ?
4 3
3
1 6 ⎛1⎞ 1
Solution: = ⎜ ⎟ =6
4 ⎝4⎠ 64
1 6 1
3 =
3 9
1 1 1 1 1 1
> ⇒6 >6 ⇒3 >
9 64 9 64 3 4
∴ 3 2 = 6 22 = 6 4
3 = 6 33 = 6 27
6
5=6 5
Now 6
4 < 6 5 < 6 27
⇒3 2<6 5< 3
2. Multipliy 3 and 3 5 .
3. Divide 3 135 by 3
5.
4. Divide 2 24 by 3
320 .
5. Which is greater 4 5 or 3 4 ?
6. Which in smaller: 5 10 or 4 9 ?
7 4
(ii) 511 × 511 = 5
1 3
(iii) 7 4 × 7 4 = 7
In each of the above three multiplications, we see that on multiplying two surds, we get the
result as rational number. In such cases, each surd is called the rationalising factor of the
other surd.
(ii) 11
54 is a rationalising factor of 11 57 and vice-versa.
(iii) 4
7 is a rationalising factor of 4 73 and vice-versa.
In other words, the process of converting surds to rational numbers is called rationalisation
and two numbers which on multiplication give the rational number is called the rationalisation
factor of the other.
Notes
For example, the rationalising factor of x is x , of 3 + 2 is 3 − 2 .
Note:
(i) The quantities x − y and x + y are called conjugate surds. Their sum and product
are always rational.
(ii) Rationalisation is usually done of the denominator of an expression involving irrational
surds.
Let us consider some examples.
Solution: 18 = 3× 3× 2 = 3 2
∴ Rationalising factor is 2 .
12 = 2× 2×3 = 2 3 .
∴ Rationalising factor is 3 .
2+ 5
Example 2.30: Rationalise the denominator of .
2− 5
2+ 5 ( 2+ 5 )( 2+ 5
=
) ( 2+ 5 )
2
Solution:
2− 5
=
( 2− 5 )( 2+ 5 ) −3
7 + 2 10 7 2
=− =− − 10
3 3 3
4+3 5
Example 2.31: Rationalise the denominator of .
4−3 5
4+3 5 (
4+3 5 4+3 5 )( )
Solution:
4−3 5
=
(
4−3 5 4+3 5 )( )
16 + 45 + 24 5 61 24
= =− − 5
16 − 45 29 29
1
Example 2.32: Rationalise the denominator of .
3 − 2 +1
1 ( 3 − 2 −1 ) Notes
Solution:
3 − 2 +1
=
[( 3 − 2 +1 ) ][( 3 − 2 −1 ) ]
3 − 2 −1 3 − 2 −1
=
=
( )
3 − 2 −1
2
4−2 6
3 − 2 −1 4 + 2 6
= ×
4−2 6 4+2 6
4 3 −4 2 −4+6 2 −4 3 −2 6
=
16 − 24
2 −2− 6 6− 2+2
=− =
4 4
3+ 2 2
Example 2.33: If = a + b 2 , find the values of a and b.
3− 2
3+ 2 2 3+ 2 2 3+ 2 9+ 4+9 2
Solution: = × =
3− 2 3− 2 3+ 2 9−2
13 + 9 2 13 9
= = + 2 =a+b 2
7 7 7
13 9
⇒a= , b=
7 7
2+ 3 2− 3
3. Simplify: +
2− 3 2+ 3
Notes 1
4. Rationalise the denominator of
3 − 2 −1
1
5. If a = 3 + 2 2 . Find a + .
a
2+5 7
6. If = x + 7 y , find x and y.
2−5 7
______________________________________________________________
LET US SUM UP
• a × a × a × ..... m times = am is the exponential form, where a is the base and m is the
exponent.
• Laws of exponent are:
m
⎛a⎞ am
m
(i) a × a = a n m+n
(ii) a ÷ a = a
m n m–n
(iii) (ab) = a b m m m
(iv) ⎜ ⎟ = m
⎝b⎠ b
1
(v) a m ( ) n
= a mn (vi) ao = 1 (vii) a
−m
=
am
p
• a = ap
q q
• In n
x , n is called index and x is called radicand.
• A surd with rational co-efficient (other than 1) is called a mixed surd.
• The order of the surd is the number that indicates the root.
• The order of n
x is n
• Laws of radicals (a > 0, b > 0)
n
[ a] = a
a n a
(i) n
n
(ii) a × b = ab
n n n (iii) n
=
b b
m a an
(x ) ( ) ( )
1 1 1
m n
=x ; n m
x = a mn
x an
or x a m
=x =x m mn
= x an mn
TERMINAL EXERCISE
1. Express the following in exponential form:
(i) 5 × 3 × 5 × 3 × 7 × 7 × 7 × 9 × 9
20 20
⎛ 37 ⎞ ⎛ 37 ⎞
(ii) ⎜ − ⎟ ÷ ⎜ − ⎟
⎝ 19 ⎠ ⎝ 19 ⎠
Notes 5
⎡⎛ 3 ⎞3 ⎤
(iii) ⎢⎜ ⎟ ⎥
⎢⎣⎝ 13 ⎠ ⎥⎦
4. Simplify each of the following:
(i) 3o + 7o + 37o – 3 (ii) ( 7o + 3o) ( 7o – 3o)
5. Simplify the following:
−3 5
⎛ 2⎞ ⎛ 2⎞
(i) (32 ) ÷ (32 )
12 −6
(ii) (111) × (111)6 −5
(iii) ⎜ − ⎟ × ⎜ − ⎟
⎝ 9⎠ ⎝ 9⎠
−3 11 x
⎛3⎞ ⎛3⎞ ⎛3⎞
6. Find x so that ⎜ ⎟ × ⎜ ⎟ = ⎜ ⎟
⎝7⎠ ⎝7⎠ ⎝7⎠
−2 −9 2 x +1
⎛3⎞ ⎛3⎞ ⎛3⎞
7. Find x so that ⎜ ⎟ × ⎜ ⎟ = ⎜ ⎟
⎝ 13 ⎠ ⎝ 13 ⎠ ⎝ 13 ⎠
8. Express as a product of primes and write the answers of each of the following in
exponential form:
(i) 6480000 (ii) 172872 (iii) 11863800
9. The star sirus is about 8.1 × 1013 km from the earth. Assuming that the light travels at
3.0 × 105 km per second, find how long light from sirus takes to reach earth.
10. State which of the following are surds:
36
(i) (ii) 9 729 (iii) 3 5 +1 (iv) 4 3125
289
11. Express as a pure surd:
5 1
(i) 4 48 − +6 3
2 3 Notes
(ii) 63 + 28 − 175
(iii) 8 + 128 − 50
15. Which is greater?
(i) 2 or 3
3 (ii) 3 6 or 4 8
(i) 3 , 3 4 , 4 5 (ii) 2 , 3 , 3 4
3 12 5 −2
(i) (ii) (iii)
6− 7 7− 3 5+2
1 1
(i) (ii)
1+ 2 − 3 7 + 5 − 12
5+ 2 3
20. If = a + b 3, find the values of a and b, where a and b are rational numbers.
7+4 3
1
21. If x = 7 + 4 3 , find the value of x + .
x
2. Base Exponent
(i) – 3 5
Notes (ii) 7 4
2
(iii) − 8
11
81 16 27
3. (i) (ii) (iii) −
2401 6561 64
3 625
4. (i) (ii)
7 324
5 4 4
⎛1⎞ ⎛ 1⎞ ⎛ 5⎞
5. (i) ⎜ ⎟ (ii) ⎜ − ⎟ (iii) ⎜ − ⎟
⎝3⎠ ⎝ 7⎠ ⎝ 3⎠
2.2
1. (i) 31 × 111 × 131 (ii) 23 × 34 (iii) 23 × 33 × 71
2. (i) 36 (ii) 29 (iii) 25 × 34
113 (− 7 )3
(iv) 12 (v)
2 25
2.3
5 6
⎛3⎞ ⎛ 7⎞
1. (i) (7) 5
(ii) ⎜ ⎟ (iii) ⎜ − ⎟
⎝4⎠ ⎝ 8⎠
6 15
⎛3⎞ ⎛ 7⎞
2. (i) (–7) 2
(ii) ⎜ ⎟ (iii) ⎜ − ⎟
⎝4⎠ ⎝ 8⎠
6 15
⎛3⎞ ⎛ 5⎞
3. (i) 2 18
(ii) ⎜ ⎟ (iii) ⎜ − ⎟
⎝4⎠ ⎝ 9⎠
5 3
⎛ 11 ⎞ ⎛ 7⎞
(iv) ⎜ ⎟ (v) ⎜ − ⎟
⎝3⎠ ⎝ 11 ⎠
4. True: (i), (ii), (vii)
False: (iii), (iv), (v), (vi)
49
1.
9
Notes
4 12
⎛7⎞ ⎛ 13 ⎞
2. (i) ⎜ ⎟ (ii) 12 2
(iii) ⎜ ⎟
⎝3⎠ ⎝3⎠
−4 −10 −10
⎛7⎞ ⎛1⎞ ⎛ 4⎞
3. (i) ⎜ ⎟ (ii) ⎜ ⎟ (iii) ⎜ − ⎟
⎝3⎠ ⎝7⎠ ⎝ 3⎠
81 2 343
4. (i) (ii) − (iii) −
16 3 125
5. True: (ii), (iii), (iv)
2.5
25
1. (i) 8 (ii)
9
7 13
2. (i) 1 (ii) (iii)
8 16
2.6
1. (i) 4, 64 (ii) 6, 343 (iii) 2, 119
2. (iii), (iv)
3. Pure: (i), (iv)
Mixed: (ii), (iii)
2.7
1. (i), (iii)
75
2. (i) 147 (ii) 3 432 (iii)
8
2.8
1. 9 7 2. 22 2 3. 27 2 4. 3
5. − 3 3 6. 303 2 7. 51 2 + 36 5 − 42 3
2.9
Notes
216
1. 203 2 2. 33 5 3. 3 4. 6
25
5. 3
4 6. 4
9 7. 6
3, 3 2 , 3 4 8. 3
4 , 4 3, 3 2
2.10
12 2 51 8 55
2. (i) 5 (ii) (iii) − (iv) 2 + 3
5 17 3 3
3. 14
4. −
1
4
[
2+ 6 + 2 ]
5. 6
179 20 7
6. − −
171 171
4
⎛ 7⎞
1. (i) 52 × 32 × 73 × 92 (ii) ⎜ − ⎟
⎝ 9⎠
5 1
2. (i) − (ii)
56 105
15
⎛3⎞
4
3. (i) 2 × 3 × 5 2 4
(ii) 1 (iii) ⎜ ⎟
⎝ 13 ⎠
4. (i) zero (ii) zero
2
⎛2⎞
5. (i) (32) 18
(ii) 111 (iii) ⎜ ⎟
⎝9⎠
127
14. (i) 3 (ii) zero (iii) 5 2
6
17. 3 16 , 6 320 , 12
(
18. (i) − 3 6 + 7 ) (
(ii) 3 7 + 3 ) (ii) 9 − 4 5
2+ 2 + 6 7 5 + 5 7 + 2 105
19. (i) (ii)
4 70
20. a = 11, b = –6
21. 14
Notes
3
ALGEBRAIC EXPRESSIONS AND
POLYNOMIALS
So far, you had been using arithmetical numbers, which included natural numbers, whole
numbers, fractional numbers, etc. and fundamental operations on those numbers. In this
lesson, we shall introduce algebraic numbers and some other basic concepts of algebra
like constants, variables, algebraic expressions, special algebraic expressions, called
polynomials and four fundamental operations on them.
OBJECTIVES
After studying this lesson, you will be able to
• identify variables and constants in an expression;
• cite examples of algebraic expressions and their terms;
• understand and identify a polynomial as a special case of an algebraic expression;
• cite examples of types of polynomials in one and two variables;
• identify like and unlike terms of polynomials;
• determine degree of a polynomial;
• find the value of a polynomial for given value(s) of variable(s), including zeros
of a polynomial;
• perform four fundamental operations on polynomials.
3 4 21
4, − 14, 2 , , − , 3x, y, 2z
2 15 8
3 4
You know that 4, − 14, 2 , , and − are real numbers, each of which has a fixed
2 15
21
value while 3x, y and 2z contain unknown x, y and z respectively and therefore do
8
not have fixed values like 4, –14, etc. Their values depend on x, y and z respectively.
Therefore, x, y and z are variables.
Thus, a variable is literal number which can have different values whereas a constant
has a fixed value.
In algebra, we usually denote constants by a, b, c and variables x, y, z. However, the
context will make it clear whether a literal number has denoted a constant or a variable.
x
x2 +8 is a polynomial in one variable x and 2x2 + y3 is a polynomial in two variables x and
y. In this lesson, we shall restrict our discussion of polynomials including two variables
only.
General form of a polynomial in one variable x is:
a0 + a1x + a2x2 + ....+anxn
where coefficients a0, a1, a2, ....an are real numbers, x is a variable and n is a whole
number. a0, a1x, a2x2, ...., anxn are (n + 1) terms of the polynomial.
An algebraic expression or a polynomial, consisting of only one term, is called a monomial.
1 2 3
Thus, –2, 3y, –5x2, xy, x y are all monomials.
2
An algebraic expression or a polynomial, consisting of only two terms, is called a binomial.
Thus, 5 + x, y2 – 8x, x3 – 1 are all bionomials.
5 1
3x 2 y − x− y+2
2 3
Solution: The terms of the given expression are
5 1
3x2y, − x, − y , 2
2 3
1 1
(i) + x 3 − 2x 2 + 6 x (ii) x +
2 x
Notes
2 2 3
(iii) 2x + 3x –5 x + 6 (iv) 5 – x – x – x
1
In (ii), second term is = x −1 . Since second term contains negative exponent
x
of the variable, the expression is not a polynomial.
1
In (iii) , third term is − 5 x = −5x 2 . Since third term contains fractional exponent
of the variable, the expression is not a polynomial.
Example 3.5: Write like terms, if any, in each of the following expressions:
1 2
(ii) x − 2 y − x + 3y − 8
2
(i) x + y + 2
2
2 1 5 1
(iii) 1 – 2xy + 2x2y – 2xy2 + 5x2y2 (iv) y− z+ y+
3 3 3 3
1 2
(ii) x2 and − x are like terms, also –2y and 3 y are like terms
2
(iii) There are no like terms in the expression.
2
(iv) y and 5 y are like terms
3 3
2 5 1 1
(iv) xy + (v) 2x2 + y2 – 8 (vi) x +
5 2 x
1 3 2
(iv) x y
8
5. Identify like terms, if any, in each of the following expressions:
1 2 1 2
(i) – xy2 + x2y + y2 + yx (ii) 6a + 6b – 3ab + a b + ab
3 4
1 2
(iii) ax2 + by2 + 2c – a2x – b2y – c
3
6. Which of the following algebraic expressions are polynomials?
1 3
(i) x +1 (ii) 52 – y2 – 2 (iii) 4x–3 + 3y
3
1
(iv) 5 x + y + 6 (v) 3x2 – 2y
2
(vi) y2 – +4
y2
7. Identify each of the following as a monomial, binomial or a trinomial:
1 3 3
(i) x3 + 3 (ii) xy (iii) 2y2 + 3yz + z2
3
(iv) 5 – xy – 3x2y2 (v) 7 – 4x2y2 (vi) – 8x3y3
9 1
= =4
2 2
(i) 5x6y4 + 1 (ii) 105 + xy3 (iii) x2 + y2 (iv) x2y + xy2 – 3xy + 4
4. Evaluate each of the following polynomials for the indicated value of the variable:
2 3 4 2 7
(iii) x + x − for x = – 1 (iv) 2x3 – 3x2 – 3x + 12 for x = – 2
3 5 5
⎛ 3⎞ ⎛ 7⎞
∴ ⎜ 5x + 3y − ⎟ + ⎜ − 2x + y + ⎟ = 3x + 4y + 1
⎝ 4⎠ ⎝ 4⎠
3 3 x3 Notes
Example 3.10: Add x + x 2 + x + 1 and x 4 − − 3x + 1
2 2
3 2
Solution: x + x2 + x + 1
2
1
+ x4 − x3 – 3x + 1
2
⎛3 1⎞
x 4 + ⎜ − ⎟ x 3 + x 2 + (1 − 3)x + (1 + 1)
⎝2 2⎠
= x 4 + x3 + x 2 − 2x + 2
⎛3 2 ⎞ ⎛ 4 x ⎞
3
∴ ⎜ x + x + x + 1⎟ + ⎜ x − − 3x + 1⎟⎟ = x 4 + x 3 + x 2 − 2 x + 2
2
⎜
⎝2 ⎠ ⎝ 2 ⎠
In order to subtract one polynomial from another polynomial, we go through the following
three steps:
Step 1: Arrange the given polynomials in columns so that like terms are in one column.
Step 2: Change the sign (from + to – and from – to +) of each term of the polynomial to
be subtracted.
Step 3: Add the like terms of each column separately.
Let us understand the procedure by means of some examples.
2 2
Example 3.11: Subtract − 4x + 3x + from 9x 2 − 3x − .
2
3 7
2
Solution: 9x 2 − 3x −
7
2
− 4x 2 + 3x +
3
+ – –
(9 + 4)x 2 + (− 3 − 3)x + ⎛⎜ − 2 − 2 ⎞⎟
⎝ 7 3⎠
20
= 13 x − 6 x −
2
21
⎛ 2⎞ ⎛ 2⎞ 20
∴ ⎜ 9x 2 − 3x − ⎟ − ⎜ − 4x 2 + 3x + ⎟ = 13 x 2 − 6 x −
⎝ 7⎠ ⎝ 3⎠ 21
Notes Example 3.12: Subtract 3x – 5x2 + 7 + 3x3 from 2x2 –5 + 11x – x3.
Solution: – x3 + 2x2 + 11x – 5
3x3 – 5x2 + 3x + 7
– + – –
(–1–3)x3 + (2 + 5)x2 + (11 – 3)x + (–5 – 7)
= – 4x3 + 7x2 + 8x – 12
∴ (2x2 –5 + 11x – x3) – (3x – 5x2 + 7 + 3x3 ) = – 4x3 + 7x2 + 8x – 12
Example 3.13: Subtract 12xy – 5y2 – 9x2 from 15xy + 6y2 + 7x2.
Solution: 15xy + 6y2 + 7x2
12xy – 5y2 – 9x2
– + +
3xy + 11y2 + 16x2
Thus, (15xy + 6y2 + 7x2) – (12xy – 5y2 – 9x2 ) = 3xy + 11y2 + 16x2
We can also directly subtract without arranging expressions in columns as follows:
(15xy + 6y2 + 7x2) – (12xy – 5y2 – 9x2 )
= 15xy + 6y2 + 7x2 – 12xy + 5y2 + 9x2
= 3xy + 11y2 + 16x2
In the same manner, we can add more than two polynomials.
Example 3.14: Add polynomials 3x + 4y – 5x2, 5y + 9x and 4x – 17y – 5x2.
Solution: 3x + 4y – 5x2
9x + 5y
4x – 17y – 5x2
16x – 8y – 10x2
Example 3.15: Subtract x2 – x – 1 from the sum of 3x2 – 8x + 11, – 2x2 + 12x and
– 4x2 + 17.
1 2 7 2 1
(ii) x + x − 5, x 2 + 5 + x and − x 2 − x
3 8 3 8
(iii) a2 – b2 + ab, b2 – c2 + bc and c2 – a2 + ca
(iv) 2a2 + 3b2, 5a2 – 2b2 + ab and – 6a2 – 5ab + b2
3. Subtract:
(i) 7x3 – 3x2 + 2 from x2 – 5x + 2
(ii) 3y – 5y2 + 7 + 3y3 from 2y2 – 5 + 11y – y3
1 2 ⎛ 1⎞ ⎛ 1 ⎞⎛ 1 ⎞ 1
− y z × ⎜ − ⎟ yz = ⎜ − ⎟⎜ − ⎟ y 2+1 z1+1 = y 3 z 2
2 ⎝ 3⎠ ⎝ 2 ⎠⎝ 3 ⎠ 6
= 2n3 – 3n2 – n + 12
Example 3.16: Find the product of (0.2x2 + 0.7 x + 3) and (0.5 x2 – 3x)
(i) (15x 3
)
− 3 x 2 + 18 x ÷ 3 x =
15 x 3 3 x 2 18 x
3x
− +
3x 3x
= 5x2 – x + 6
− 8x 2 10x
(ii) (− 8x 2
)
+ 10x ÷ (− 2x ) = +
− 2x − 2x
⎛ − 8 ⎞⎛ x ⎞ 10 x
2
=⎜ ⎟⎜⎜ ⎟⎟ + ×
⎝ − 2 ⎠⎝ x ⎠ (− 2) x
= 4x – 5
6 Quotient
Notes Divisor 3 20 Dividend
18
2 Remainder
The steps involved in the process of division of a polynomial by another polynomial are
explained below with the help of an example.
Let us divide 2x2 + 5x + 3 by 2x + 3.
Step 1: Arrange the terms of both the polynomials in 2x + 3 2x 2 + 5x + 3
decreasing powers of the variable common to both
the polynomials.
x
Step 2: Divide the first term of the dividend by the first term 2x + 3 2 x 2 + 5x + 3
of the divisor to obtain the first term of the quotient.
Step 3: Multiply all the terms of the divisor by the first term
x
of the quotient and subtract the result from the
dividend, to obtain a remainder (as next dividend) 2x + 3 2 x + 5x + 3
2
Step 4: Divide the first term of the resulting dividend by the 2x2 + 3x
– –
first term of the divisor and write the result as the
2x + 3
second term of the quotient.
Step 5: Multiply all the terms of the divisor by the second
term of the quotient and subtract the result from x +1
the resulting dividend of Step 4. 2x + 3 2 x 2 + 5x + 3
Step 6: Repeat the process of Steps 4 and 5, till you get 2x2 + 3x
either the remainder zero or a polynomial having – –
the highest exponent of the variable lower than that 2x + 3
of the divisor. 2x +3
In the above example, we got the quotient x + 1 – –
and remainder 0. 0
7 25
x2 − x −
2 4
So,
2x − 5 2x 3 − 12x 2 + 5x – 11
2x3 – 5x2
– +
– 7 x2 + 5x – 11
35
– 7 x2 + x
2
+ –
25
– x – 11
2
25 125
– x +
2 4
+ –
169
–
4
7 25 169
We get quotient x − x − and remainder –
2
.
2 4 4
LET US SUM UP
• A literal number (unknown quantity), which can have various values, is called a variable.
• A constant has a fixed value.
• An algebraic expression is a combination of numbers, variables and arithmetical
operations. It has one or more terms joined by the signs + or –.
• Numerical coefficient of a term, say, 2xy is 2. Coefficient of x is 2y and that of y is 2x.
• Numerical coefficient of non-negative x is + 1 and that of – x is – 1.
• An algebraic expression, in which variable(s) does (do) not occur in the denominator,
exponents of variables are whole numbers and numerical coefficients of various terms
are real numbers, is called a polynomial.
• The standard form of a polynomial in one variable x is:
a0 + a1x + a2x2+ ....+ anxn (or written in reverse order) where a0, a1, a2, .... an are real
numbers and n, n–1, n–2, ...., 3, 2, 1 are whole numbers.
• An algebraic expression or a polynomial having one term is called a monomial, that
having two terms a bionomial and the one having three terms a trinomial.
• The terms of an algebraic expression or a polynomial having the same variable(s) and
same exponent(s) of variable(s) are called like terms. The terms, which are not like,
are called unlike terms.
• The sum of the exponents of variables in a term is called the degree of that term.
• The degree of a polynomial is the same as the degree of its term or terms having the
highest degree and non-zero numerical coefficient.
• The degree of a non-zero constant polynomial is zero.
• The process of substituting a numerical value for the variable(s) in an algebraic expression
(or a polynomial) is called evaluation of the algebraic expression (or polynomial).
• The value(s) of variable(s), for which the value of a polynomial is zero, is (are) called
zero(s) of the polynomial.
• The sum of two like terms is a like term whose numerical coefficient is the sum of the
numerical coefficients of the two like terms.
TERMINAL EXERCISE
1. Mark a tick () against the correct alternative:
(i) The coefficient of x4 in 6x4y2 is
(A) 6 (B) y2 (C) 6y2 (D) 4
(ii) Numerical coefficient of the monomial –x2y4 is
(A) 2 (B) 6 (C) 1 (D) –1
(iii) Which of the following algebraic expressions is a polynomial?
1 2 1
(A) x − 8 + 3.7x (B) 2x + −4
2 2x
( ) (
(C) x 2 − 2y 2 ÷ x 2 + y 2 ) (D) 6 + x − x − 15x 2
(iv) How many terms does the expression 1 − 2a 2 b 3 − (7a )(2b ) + 3b 2 contain?
(A) 5 (B) 4 (C) 3 (D) 2
(v) Which of the following expressions is a binomial?
(A) 2x2y2 (B) x2 + y2 – 2xy
(C) 2 + x2 + y2 + 2x2y2 (D) 1 – 3xy3
(vi) Which of the following pairs of terms is a pair of like terms?
(A) 2a, 2b (B) 2xy3, 2x3y
1
(C) 3x2y, yx 2 (D) 8, 16 a
2
(vii)A zero of the polynomial x2 – 2x – 15 is
(A) x = – 5 (B) x = – 3
(C) x = 0 (D) x = 3
(viii) The degree of the polynomial x3y4 + 9x6 – 8y5 + 17 is
(A) 7 (B) 17
Notes
(C) 5 (D) 6
2. Using variables and operation symbols, express each of the following verbal statements
as algebraic statement:
(i) A number added to itself gives six.
(ii) Four subtracted from three times a number is eleven.
(iii) The product of two successive odd numbers is thirty-five.
(iv) One-third of a number exceeds one-fifth of the number by two.
3. Determine the degree of each of the following polynomials:
(i) 327 (ii) x + 7x2y2 – 6xy5 – 18 (iii) a4x + bx3 where a and b are constants
(iv) c6 – a3x2y2 – b2x3y Where a, b and c are constants.
4. Determine whether given value is a zero of the polynomial:
(i) x2 + 3x – 40; x = 8
(ii) x6 – 1; x=–1
5. Evaluate each of the following polynomials for the indicated value of the variable:
3 2 4 5 1
(i) 2x − x + x + 7x at x =
3
2 5 2
4 3 1 2
(ii) y + y − 6y − 65 at y = −5
5 5
1 2 1
6. Find the value of n + n for n = 10 and verify that the result is equal to the sum of
2 2
first 10 natural numbers.
7. Add:
7 3 2 2 7 2 3 3
(i) x + x − 3x + and x 3 + x 2 − 3x +
3 5 5 3 5 5
(ii) x2 + y2 + 4xy and 2y2 – 4xy
(iii) x3 + 6x2 + 4xy and 7x2 + 8x3 + y2 + y3
2 2
(iv) 2x + 3x + and − 3x 5 + x − 3
5
3 5
2 5 7 2 2 5
(vii) x + x + by x − x + x − 3 by 3x 2 + 4x + 1
2
(viii)
3 6 4 3 4
15. Subtract the product of (x2 – xy + y2) and (x + y) from the product of (x2 + xy + y2)
and (x – y).
16.Divide
(i) 8x3 + y3 by 2x + y (ii) 7x3 + 18x2 + 18x – 5 by 3x + 5
(iii) 20x2 – 15x3y6 by 5x2 (iv) 35a3 – 21a4b by (–7a3)
(v) x3 – 3x2 + 5x – 8 by x – 2 (vi) 8y2 + 38y + 35 by 2y + 7
In each case, write the quotient and remainder.
2 1
(iv) x, y; , (v) x, y; 2, –8 (vi) x; None
5 2
1
4. (i) 2, abc (ii) a,b,c, 2 (iii) x2y, – 2xy2, −
2
1 3 2
(iv) xy
8
1 2
5. (i) –xy2, + yx (ii) –3ab, + ab (iii) No like terms
3
6. (i) , (ii) and (v) 7. Monomials (ii) and (vi);
Binomials: (i) and (v); Trinomials : (iii) and (iv)
3.2
1. (i) 7 (ii) 3 (iii) 1 (iv) 0
2 2
2. 2.5, 9x, x , –25x3, – 3x6
9
3. (i) 10 (ii) 4 (iii) 2 (iv) 3
19
4. (i) 0 (ii) –7 (iii) − (iv) 6
15
3.3
23 2 5 7 3
1. (i) x + x+6 (ii) x + x2 + x − 2
11 4 5
19
(iii) 3x + 12x − 7x +
3 2
y (iv) 9x3 + 5x2y – 8xy
3
2. (i) –x2 + 4x + 17 (ii) 0
(iii) ab + bc +ca (iv) a2 + 2b2 – 4ab
3. (i) – 7x3 + 4x2 – 5x (ii) –4y3 + 7y2 + 8y – 12
(iii) 3z3 + 2z2 – 2z + 5 (iv) – 7x3 + 10x2 – 9x – 17
4. a – ab – 3
12 3 24 12 3 4
15. –2y3
16. (i) 4x2 –2xy + y2; 0 (ii) 9x2 – 9x + 21; –110
(iii) 4 – 3xy6; 0 (iv) – 5 + 3ab; 0
(iv) x2 – x + 3; – 2 (v) 4y + 5; 0
Notes
4
SPECIAL PRODUCTS AND
FACTORIZATION
OBJECTIVES
After studying this lesson, you will be able to
• write formulae for special products (a ± b)2, (a + b) (a –b), (x + a) (x +b),
(a + b) (a2 – ab + b2), (a – b) (a2 + ab + b2), (a ± b)3 and (ax + b) (cx +d);
• calculate squares and cubes of numbers using formulae;
• factorise given polynomials including expressions of the forms a2 – b2, a3 ± b3;
• factorise polynomials of the form ax2 + bx + c (a ≠ 0) by splitting the middle
term;
• determine HCF and LCM of polynomials by factorization;
a–b
(a + b) (a – b) = Area of Rectangle ABCD
= Area of Rectangle AEFD +
area of rectangle EBCF a–b F
E G
= Area of Rectangle AEFD + b b
2
⎛3 ⎞
(i) (2a + 3b)2
(ii) ⎜ a − 6b ⎟
⎝2 ⎠
(iii) (3x + y) (3x – y) (iv) (x + 9) (x + 3)
(v) (a + 15) ( a – 7) (vi) (5x – 8) (5x – 6)
(vii) (7x – 2a) (7x + 3a) (viii) (2x + 5) (3x + 4)
Solution:
(i) Here, in place of a, we have 2a and in place of b, we have 3b.
(2a + 3b)2 = (2a)2 + 2(2a) (3b) + (3b)2
= 4a2 + 12ab + 9b2
(ii) Using special product (2), we get
2 2
⎛3 ⎞ ⎛3 ⎞ ⎛3 ⎞
⎜ a − 6b ⎟ = ⎜ a ⎟ − 2⎜ a ⎟(6b ) + (6b )
2
⎝2 ⎠ ⎝2 ⎠ ⎝2 ⎠
9 2
= a − 18ab + 36b 2
4
(iii) (3x + y) (3x – y) = (3x)2 – y2 [using speical product (3)]
= 9x2 – y2
(iv) (x + 9) (x + 3) = x2 + (9 + 3)x + 9 × 3 [using speical product (4)]
⎛ 4 ⎞⎛ 3⎞
(vii) (a2 + 5) (a2 – 5) (viii) (xy – 1) (xy + 1) (ix) ⎜ x + ⎟⎜ x + ⎟
⎝ 3 ⎠⎝ 4⎠
⎛2 2 ⎞⎛ 2 2 1 ⎞
(x) ⎜ x − 3 ⎟⎜ x + ⎟ (xi) (2x + 3y) (3x + 2y) (xii) (7x + 5y) (3x – y)
⎝3 ⎠⎝ 3 3⎠
2. Simplify:
(i) (2x2 + 5)2 – (2x2 – 5)2 (ii) (a2 + 3)2 + (a2 – 3)2
(iii) (ax + by)2 + (ax – by)2 (iv) (p2 + 8q2)2 – (p2 – 8q2)2
Solution: (i) (7x + 9y)3 = (7x)3 + 3(7x) (9y) (7x + 9y) + (9y)3
= 343 x3 + 189 xy (7x + 9y) + 729y3
= 343x3 + 1323x2y + 1701xy2 + 729y3
(ii) (px – yz)3 = (px)3 – 3(px) (yz) (px – yz) – (yz)3
= p3x3 – 3pxyz (px – yz) – y3z3
= p3x3 – 3p2x2yz + 3pxy2z2 – y3z3
(iii) (x – 4y2)3 = x3 – 3x (4y2) (x – 4y2) – (4y2)3
= x3 – 12xy2 (x – 4y2) – 64y6
= x3 – 12x2y2 + 48xy4 – 64y6
(iv) (2a2 + 3b2)3= (2a2)3 + 3(2a2)(3b2) (2a2 + 3b2) + (3b2)3
= 8a6 + 18a2b2 (2a2 + 3b2) + 27b6
= 8a6 + 36a4b2 + 54a2b4 + 27b6
3 3 3
⎛2 5 ⎞ ⎛2 ⎞ ⎛ 2 ⎞⎛ 5 ⎞⎛ 2 5 ⎞ ⎛5 ⎞
(v) ⎜ a − b ⎟ = ⎜ a ⎟ − 3⎜ a ⎟⎜ b ⎟⎜ a − b ⎟ − ⎜ b ⎟
⎝3 3 ⎠ ⎝3 ⎠ ⎝ 3 ⎠⎝ 3 ⎠⎝ 3 3 ⎠ ⎝3 ⎠
8 3 10 ⎛ 2 5 ⎞ 125 3
= a − ab⎜ a − b ⎟ − b
27 3 ⎝3 3 ⎠ 27
8 3 20 2 50 125 3
= a − a b + ab 2 − b
27 9 9 27
3 3
⎛ 4 ⎞ ⎛ 4 ⎞⎛ 4 ⎞ ⎛ 4 ⎞
(vi) ⎜1 + c ⎟ = (1) + 3(1)⎜ c ⎟⎜1 + c ⎟ + ⎜ c ⎟
3
⎝ 3 ⎠ ⎝ 3 ⎠⎝ 3 ⎠ ⎝ 3 ⎠
⎛ 4 ⎞ 64 3
= 1 + 4c⎜1 + c ⎟ + c
⎝ 3 ⎠ 27
16 2 64 3
= 1 + 4c + c + c
3 27
= (3a)3 – (2b)3
= 27a3 – 8b3
Example 4.6: Simplify:
Notes
(i) (3x – 2y)3 + 3 (3x – 2y)2 (3x + 2y) + 3(3x – 2y) (3x + 2y)2 + (3x + 2y)3
(ii) (2a – b)3 + 3 (2a – b) (2b – a) (a + b) + (2b – a)3
Solution: (i) Put 3x – 2y = a and 3x + 2y = b
The given expression becomes
a3 + 3a2b + 3ab2 + b3
= (a + b)3
= (3x – 2y + 3x + 2y)3
= (6x)3
= 216x3
(ii) Put 2a – b = x and 2b –a = y so that a + b = x + y
The given expression becomes
x3 + 3xy (x + y) + y3
= (x + y)3
= (a + b)3
= a3 + 3a2b + 3ab2 + b3
Example 4.7: Simplify:
857 3 − 537 3
857 2 + 857 × 537 + 537 2
Let 857 = a and 537 = b, then the expression becomes
a 3 − b3
=
( )
(a − b ) a 2 + ab + b 2 = a − b
a 2 + ab + b 2 a 2 + ab + b 2
3 3 3
⎛a ⎞ ⎛1 2 2 2⎞ ⎛1 2 3 3 2⎞
(iv) ⎜ − b ⎟ (v) ⎜ a + b ⎟ (vi) ⎜ a x − 2b y ⎟
⎝3 ⎠ ⎝2 3 ⎠ ⎝3 ⎠
2. Using special products, find the cube of each of the following:
(i) 8 (ii) 12 (iii) 18 (iv) 23
(v) 53 (vi) 48 (vii) 71 (viii) 69
(ix) 97 (x) 99
3. Without actual multiplication, find each of the following products:
(i) (2x + y) (4x2 – 2xy + y2) (ii) (x – 2) ( x2 + 2x + 4)
(iii) (1 + x) ( (1 – x + x2) (iv) (2y – 3z2) (4y2 + 6yz2 + 9z4)
⎛ 1 ⎞⎛ 2 3 1 2⎞
(v) (4x + 3y) (16x2 – 12xy + 9y2) (vi) ⎜ 3x − y ⎟⎜ 9x + xy + y ⎟
⎝ 7 ⎠⎝ 7 49 ⎠
4. Find the value of:
(i) a3 + 8b3 if a + 2b = 10 and ab = 15
[Hint: (a + 2b)3 = a3 + 8b3 + 6ab (a + 2b) ⇒ a3 + 8b3 = (a + 2b)3 – 6ab (a + 2b)]
(ii) x3 – y3 when x – y = 5 and xy = 66
Notes p
be expressed in the form where p and q (≠0) are integers, is called a rational number,
q
P
an algebraic expression, which can be expressed in the from , where P and Q (non-zero
Q
polynomials) are polynomials, is called a rational expression. Thus, each of the expressions
1 2 5
a + b2 −
x + 1 x 2 − 3x + 5 2 6 , x + 2y
2 2
, ,
x −1 x2 − 5 a+b 3x − y
2
x2 +1
(1) The polynomial ‘x + 1’ is a rational expresion since it can be written as and
1
you have learnt that the constant 1 in the denominator is a polynomial of degree zero.
7
(2) The polynomial 7 is a rational expresion since it can be written as where both 7 and
1
1 are polynomials of degree zero.
(3) Obvioulsy a rational expression need not be a polynomial. For example rational
expression
1
x
( )
= x −1 is not a polynomial. On the contrary every polynomial is also a
rational expression.
2
1
a3 −
None of the expressions x + 2 , x 2 + 2 x + 3, b is a rational expression.
1− x a + ab + b 2
2
2x − 3 8
(i) (ii)
4x − 1 x + y2
2
2 3x 2 + 5 2x 2 − x + 3
(iii) (iv)
7 6x
⎛ 1⎞ 3
1 Notes
(v) 200 + 11 (vi) ⎜ a + ⎟ ÷ b
⎝ b⎠
20 + 9
=
24
29
=
24
3x 2 + 4x − 1
=
x 2 −1
x −1 3x − 2
Example 4.20: Subtract from
x +1 3x + 1
3x − 2 x − 1 (x + 1)(3x − 2) − (x − 1)(3x + 1)
−
Solution:
3x + 1 x + 1
=
(3x + 1)(x + 1)
=
(
3x 2 + x − 2 − 3x 2 − 2x − 1) )
3x 2 + 4x + 1
3x − 1
=
3x + 4x + 1
2
Note: Observe that the sum and difference of two rational expressions are also rational
expressions.
Since the sum and difference of two rational expressions are rational expressions,
1
x+ (x ≠ 0) and x – 1 (x ≠ 0) are both rational expressions as x and 1 are both rational
x x x
1 3 1 2 1 3 1
expressions. Similarly, each of x + , x + 3 , x − 2 , x − 3 , etc. is a rational
2
x2 x x x
1 1
expression. These expresions create interest as for given value of x + or x – , we
x x
1 2 1 3 1 3 1
can determine values of x + , x − 2 , x + 3 , x − 3 etc. and in some case vice
2
x2 x x x
versa also. Let us concentrate on the following example.
Example 4.21:Find the value of
1 1 1 1
(i) x + if x − = 1 (ii) x + if x + = 4
2 4
2 4
x x x x
1 1 1 1
(iii) x – if x 4 + 4 = 119 (iv) x +
3
3
if x + = 3
x x x x
1 1
(v) x − if x – = 5
3
3
x x
1
Solution: (i) We have x − =1 Notes
x
2
⎛ 1⎞
∴ ⎜ x − ⎟ = (1)
2
⎝ x⎠
1 1
⇒ x2 + 2
− 2× x × = 1
x x
1
⇒ x2 + 2 − 2 = 1
x
1
Hence, x 2 + 2 = 3
x
1
(ii) x + = 4
x
2
⎛ 1⎞
⇒ ⎜ x + ⎟ = (4 )
2
⎝ x⎠
1
⇒ x2 + + 2 = 16
x2
1
⇒ x2 + = 14
x2
2
⎛ 1 ⎞
⇒ ⎜ x 2 + 2 ⎟ = (14 )
2
⎝ x ⎠
1
⇒ x4 + + 2 = 196
x4
1
So, x 4 + = 194
x4
1
(iii) We have x + = 119
4
x4
2
( )
2 ⎛ 1 ⎞
⇒ x 2 + ⎜ 2 ⎟ + 2 = 119 + 2 = 121
⎝x ⎠
2
⎛ 1 ⎞
⇒ ⎜ x 2 + 2 ⎟ = (11)
2
⎝ x ⎠
Notes 1 1
⇒ x2 + = 11 [since both x2 and are positive]
x2 x2
1
⇒ x2 + −2=9
x2
2
⎛ 1⎞
⇒ ⎜ x − ⎟ = (3)
2
⎝ x⎠
1
∴x − = ±3
x
1
(iv) We have x + =3
x
3
⎛ 1⎞
∴ ⎜ x + ⎟ = (3)
3
⎝ x⎠
1 1⎛ 1⎞
⇒ x3 + 3
+ 3 × x × ⎜ x + ⎟ = 27
x x⎝ x⎠
1
⇒ x3 + + 3(3) = 27
x3
1
∴ x3 + = 18
x3
1
(v) We have x − =5
x
3
⎛ 1⎞
∴ ⎜ x − ⎟ = (5)
3
⎝ x⎠
1 1⎛ 1⎞
⇒ x3 − 3
− 3 × x × ⎜ x − ⎟ = 125
x x⎝ x⎠
1
⇒ x3 − − 3(5) = 125
x3
1
∴ x3 − = 140
x3
x2 +1 x 2 −1 x+2 x −1
(i) and (ii) and
x−2 x−2 x+3 x−2
x +1 1 3x + 2 x –5
and (iv) x 2 − 16 and
(iii)
(x – 1)2
x +1 (x + 4)2
x–2 x+2 x+2 x–2
(v) and (vi) and
x +3 x +3 x–2 x+2
x +1 x2 –1 3 2x + 1 – 2 2x + 1
(vii) and 2 (vii) and
x+2 x +1 3x 2
2x 2
2. Subtract
x –1 x+4 2x – 1 2x + 1
(i) from (ii) from
x−2 x+2 2x + 1 2x − 1
1 2 x +1
(iii) from x (iv) from 2
x x x −1
x2 +1 2x 2 + 3 1 2x 3 + x 2 + 3
from
(v)
x–4
from
x–4
(vi) x 2 + 2
(
x2 + 2 )
2
x+2 x–2 x +1 4x
(vii) 2(x 2 − 9 ) from from 2
(x + 3)2 (vii)
x −1 x −1
3. Find the value of
1 1 1 1
(i) a + when a + = 2 (ii) a + when a – = 2
2 2
2 2
a a a a
1 1 1 1
(iii) a + when a + = 2 (iv) a + when a + = 5
3 3
3 3
a a a a
1 1 1 1
3
(v) a – 3
when a – = 5 (vi) 8a +
3
3
when 2a + =5
a a 27a 3a
1 1 1 1
(vii) a + when a + = 3 (viii) a + when a 2 + 2 = 7, a > 0
3 3
3 3
a a a a
1 1 1 1
(ix) a – when a 4 + 4 = 727 3
(x) a – 3
when a 4 + 4 = 34, a > 0
a a a a
2 5
You know that the product of two rational numbers, say, and is given as
3 7
2 5 2 × 5 10 P R
× = = . Similarly, the product of two rational expressions, say, and
3 7 3 × 7 21 Q S
P R PR
where P, Q, R, S (Q, S ≠ 0) are polynomials is given by × = . You may observe
Q S QS
that the product of two rational expressions is again a rational expression.
Example 4.22: Find the product:
5x + 3 2x − 1 2x + 1 x − 1
(i) × (ii) ×
5x − 1 x + 1 x −1 x + 3
x 2 – 7x + 10 x 2 − 7x + 12
(iii) ×
(x − 4)2 x –5
5x + 3 2x − 1 (5x + 3)(2x − 1)
×
Solution: (i)
5x − 1 x + 1
=
(5x − 1)(x + 1)
10x 2 + x − 3
=
5x 2 + 4x − 1
2x + 1 x − 1 (2x + 1)(x − 1)
×
(ii)
x −1 x + 3
=
(x − 1)(x + 3)
2x + 1
= [Cancelling common factor (x –1) from
x +3
numerator and denominator]
x 2 – 7x + 10 x 2 − 7x + 12
×
(x 2
)(
– 7x + 10 x 2 − 7x + 12 )
(iii) =
(x − 4)2 x –5 (x − 4)2 (x – 5)
(x − 2)(x − 5)(x − 3)(x − 4)
=
(x − 4)2 (x − 5)
(x − 2)(x − 3)
=
(x − 4)
[Cancelling common factor (x –4) (x – 5) from numerator and denominator] Notes
x 2 − 5x + 6
=
x−4
Note: The result (product) obtained after cancelling the HCF from its numerator and
denominator is called the result (product) in lowest terms or in lowest form.
2
You are also familiar with the division of a rational number, say, by a rational number,
3
5 2 5 2 7 7 5
say, is given as ÷ = × where is the reciprocal of . Similarly, division of a
7 3 7 3 5 5 7
P R P R P S
rational expression by a non-zero rational expression is given by ÷ = ×
Q S Q S Q R
S R
where P, Q, R, S are polynomials and is the reciprocal expression of .
R S
Example 4.23: Find the reciprocal of each of the following rational expressions:
x 2 + 20 2y
(i) (ii) − (iii) x3 + 8
x 3 + 5x + 6 y −5
2
x 2 + 20 x 3 + 5x + 6
Solution: (i) Reciprocal of is
x 3 + 5x + 6 x 2 + 20
2y y2 − 5 5 − y2
(ii) Reciprocal of − 2 is − =
y −5 2y 2y
x3 + 8 1
(iii) Since x3 + 8 = , the reciprocal of x3 + 8 is 3
1 x +8
Example 4.24: Divide:
x2 +1 x −1
(i) by
x −1 x+2
x2 –1 x 2 − 4x − 5
(ii) by and express the result in lowest form.
x 2 − 25 x 2 + 4x − 5
x2 +1 x −1 x 2 +1 x + 2
Solution: (i) ÷ = ×
x −1 x + 2 x −1 x −1
Notes (x 2
)
+ 1 (x + 2 ) x 3 + 2x 2 + x + 2
=
=
(x − 1)2 x 2 − 2x + 1
x2 –1
÷
x 2 − 4x − 5
=
(
x 2 – 1 x 2 + 4x − 5 )( )
(ii)
x 2 − 25 (
x 2 + 4x − 5 x 2 − 25 x 2 − 4x − 5 )( )
(x − 1)(x + 1)(x + 5)(x − 1)
=
(x − 5)(x + 5)(x + 1)(x − 5)
(x − 1)(x − 1)
=
(x − 5)(x − 5)
[Cancelling HCF (x+1)(x+5)]
x 2 − 2x + 1
=
x 2 − 10x + 25
x 2 − 2x + 1
The result is in lowest form.
x 2 − 10x + 25
7x + 2 x +1 x3 +1 x3 −1
(i) × 2 (ii) ×
2x + 3x + 1 7x − 5x − 2
2
x 4 +1 x 4 −1
3x 2 − 15x + 18 17x + 3 5x − 3 x + 2
(iii) × 2 (iv) ×
2x − 4 x − 6x + 9 5x + 2 x + 6
x2 +1 x +1 x3 + 1 x −1
(v) × 2 (vi) ×
x −1 x − x + 1 x − 1 2x
x − 3 x 2 − 5x + 4 x 2 − 7x + 12 x 2 − 2x − 24
(vii) × (viii) ×
x − 4 x 2 − 2x − 3 x 2 − 2x − 3 x 2 − 16
2. Find the reciprocal of each of the following rational expressions:
x2 + 2 3a
(i) (ii) −
x −1 1− a
7
(iii) − (iv) x4 +1
1 − 2x − x 2
3. Divide and express the result as a rational expression in lowest terms:
Notes
x + 11x + 18 x + 7x + 10
2 2
6x + x – 1 4x + 4x + 1
2 2
(i) ÷ (ii) ÷
x 2 − 4x − 117 x 2 − 12x − 13 2x 2 − 7 x − 15 4x 2 − 9
x2 + x +1 x3 – 1 x 2 + 2 x – 24 x 2 – x – 6
(iii) ÷ (iv) ÷
x2 − 9 x 2 − 4x + 3 x 2 − x − 12 x2 − 9
3x 2 + 14 x – 5 3x 2 + 2 x – 1 2x 2 + x – 3 2x 2 + 5x + 3
(v) 2 ÷ (vi) ÷
x − 3x + 2 3x 2 − 3x − 2 (x – 1)2 x 2 −1
LET US SUM UP
• Special products, given below, occur very frequently in algebra:
(i) (x + y)2 = x2 + 2xy + y2 (ii) (x – y)2 = x2 – 2xy + y2
(iii) (x + y) (x – y) = x2 – y2 (iv) (x + a) (x + b) = x2 + (a + b)x + ab
(v) (ax + b) (cx + d) = acx2 + (ad + bc) x + bd
(vi) (x + y)3 = x3 + 3xy(x + y) + y3 (vii) (x – y)3 = x3 – 3xy(x – y) – y3
(viii) (x + y) (x2 – xy + y2) = x3 + y3 (ix) (x – y) (x2 + xy + y2) = x3 – y3
• Factorization of a polynomial is a process of writing the polynomial as a product of
two (or more) polynomials. Each polynomial in the product is called a factor of the
given polynomial.
• A polynomial is said to be completely factorised if it is expressed as a product of
factors, which have no factor other than itself, its negative, 1 or –1.
• Apart from the factorization based on the above mentioned special products, we can
factorise a polynomial by taking monomial factor out which is common to some or all
of the terms of the polynomial using distributive laws.
• HCF of two or more given polynomials is the product of the polynomial of the highest
degree and greatest numerical coefficient each of which is a factor of each of the given
polynomials.
• LCM of two or more given polynomials is the product of the polynomial of the lowest
degree and the smallest numerical coefficient which are multiples of corresponding
elements of each of the given polynomials.
P
• An algebraic expression, which can be expressed in the form where P and Q are
Q
polynomials, Q being a non-zero polynomial, is called a rational expression.
Notes
• Operations on rational expressions are performed in the way, they are performed in
case of rational numbers. Sum, Difference, Product and Quotient of two rational
expressions are also rational expressions.
• Expressing a rational expression into lowest terms means cancellation of common
factor, if any, from the numerator and denominator of the rational exprssion.
TERMINAL EXERCISE
1. Mark a tick against the correct alternative:
(i) If 1202 – 202 = 25p, then p is equal to
(A) 16 (B) 140 (C) 560 (D) 14000
(ii) (2a2 + 3)2 – (2a2 – 3)2 is equal to
(A) 24a2 (B) 24a4 (C) 72a2 (D) 72a4
(iii) (a2 + b2)2 + (a2 – b2)2 is equal to
(A) 2(a2 + b2) (B) 4(a2 + b2)
(C) 4(a4 + b4) (D) 2(a4 + b4)
1 1
(iv) If m − = − 3 , then m 3 − 3 is equal to
m m
(A) 0 (B) 6 3 (C) − 6 3 (D) − 3 3
x− 3
(C) 8 x + 6 y (D)
x+ 3
2. Find each of the following products:
(i) (am + an)(am – an) (ii) (x + y + 2)(x – y + 2)
(iii) (2x + 3y) (2x + 3y) (iv) (3a – 5b)(3a – 5b)
(v) (5x + 2y) ( 25x2 – 10xy + 4y2) (vi) (2x – 5y) (4x2 + 10xy + 25y2)
⎛ 5 ⎞⎛ 4⎞
(vii) ⎜ a + ⎟⎜ a + ⎟ (viii) (2z2 + 3)(2z2 – 5)
⎝ 4 ⎠⎝ 5⎠
(ix) 99 × 99 × 99 (x) 103 × 103 × 103
(xi) (a + b – 5) (a + b – 6) (xii) (2x + 7z) (2x + 5z)
3. If x = a – b and y = b –c, show that
(a – c) (a + c – 2b) = x2 – y2
4. Find the value of 64x3 – 125z3 if 4x – 5z = 16 and xz = 12.
5. Factorise:
(i) x7 y6 + x22y20 (ii) 3a5b – 243ab5
(iii) 3a6 + 12 a4b2 + 12 a2b4 (iv) a4 – 8a2b3 + 16 b6
(v) 3x4 + 12y4 (vi) x8 + 14 x4 + 81
(vii) x2 + 16x + 63 (viii) x2 – 12x + 27
(ix) 7x2 + xy – 6y2 (x) 5x2 – 8x – 4
(xi) x6 – 729y6 (xii) 125a6 + 64b6
6. Find the HCF of
(i) x3 – x5 and x4 – x7
x +1 1
+
(i)
(x − 1) x + 1
2
2x 2 + 2x − 7 x − 1
(ii) −
x2 + x − 6 x−2
x − 1 3x + 1
(iii) ×
x − 2 x2 − 4
x 2 − 1 x 2 − 4x − 5
(iv) 2 ÷
x − 25 x 2 + 4x − 5
2 2 4 8
9. Simpify: − − 2 − 4
a −1 a +1 a +1 a +1
2 2 4
[Hint : − = 2 ; now combine next term and so on]
a −1 a +1 a −1
x +1 x −1
10. If m = and n = , find m2 + n2 – mn.
x −1 x +1
x2 2 z2 1 1
(iv) 4x2 – 20xy + 5y2 (v) + x +1 (vi) − z+
9 3 4 3 9
25
(ix) x + x +1
2
(vii) a4 – 25 (viii) x2y2 – 1
12
4 4 25 2
(x) x − x −1 (xi) 6x2 + 13xy + 6y2 (xii) 21x2 + 8xy – 5y2
9 9
2. (i) 40x2 (ii) 2a6 + 18 (iii) 2(a2x2 + b2y2) (iv) 32p2q2
Notes
3. (i) 10404 (ii) 11664 (iii) 4761 (iv) 996004
(v) 6384 (vi) 22451 (vii) 89964 (viii) 249936
(ix) 11445 (x) 5621 (xi) 8930 (xii) 989028
4.2
1. (i) 27x3 + 36x2y + 36xy2 + 64y3 (ii) p3 – 3p2qr + 3pq2r2 – q3r3
ab 2 b 3 a 3 a 2b
(iii) a + a b + + − + ab 2 − b 3
3 2
(iv)
3 27 27 3
a6 1 4 2 2 2 4 8 6 a6x9 2 4 3 6 2
(v) + a b + a b + b (vi) − a b x y + 4a 2 b 6 x 3 y 4 − 8b 9 y 6
8 2 3 27 27 3
2. (i) 512 (ii) 1728 (iii) 5832 (iv) 12167 (v) 148877
(vi) 110592(vii) 357911 (viii) 328509 (ix) 912663 (x) 970299
3. (i) 8x3 + y3 (ii) x3 – 8 (iii) x3 +1
1 3
(vi) 27x −
3
(iv) 8y3 – 27z6 (v) 64x3 + 27y3 y
343
4. (i) 100 (ii) 1115
27027
5. (i) 15616 (ii)
125
6. (i) 120x2 + 250 (ii) 1000y3 (iii) 19x3 – 19y3 (iv) – 117x3 – 126
7. (i) 1000 (ii) 444
4.3
1. 5x(2y – 3z) 2. abc (c – b)
3. 3p(2p – 5q +9) 4. (b – c) (a2 – b)
5. (4x – y)2 (8ax – 2ay – b) 6. x (x + y) (x2 – xy + y2)
7. 25(2 + 5p) (2 – 5p) 8. (1 + 16y4) (1 + 4y2) (1 + 2y) (1 – 2y)
9. (5x + 1) (1 – x) 10. (a2 + bc + ab + ac) (a2 + bc – ab – ac)
4.8 Notes
2x 2 2x 2 + 2x − 7 2x 2 + 2
1. (i) (ii) 2 (iii) 3
x−2 x + x −6 x − x2 − x +1
4x 2 + 5x + 28 2x 2x 2 + 8
(iv) 3 (v) (vi) 2
x + 4x 2 − 16x + 64 x +3 x −4
2x 3 + 3x 2 − 1 5
(vii) (viii)
x 3 + 2x 2 + x + 2 6x 2
x–6 8x x2 –1
2. (i) 2 (ii) 2 (iii)
x −4 4x − 1 x
2–x x2 + 2 2x 3 + 1
(iv) 2
x −x
(v)
x–4
(vi)
(x 2
+2 )
2
x 2 − 15x + 16 1- x
(vii)
(
2 x 3 + 3x 2 − 9x - 27 ) (viii)
1+ x
4.9
1 x4 + x2 +1 51x + 9
1. (i) (ii) (iii)
2x − x − 1
2
x6 + x4 + x2 +1 2x − 6
5x 2 + 7x − 6 x3 + x2 + x +1 x3 +1
(iv) (v) (vi)
5x 2 + 32x + 12 x 3 − 2x 2 + 2x − 1 2x
x−1 x− 6
(vii) (viii)
x+1 x+1
x −1 a −1 x 2 + 2x − 1 1
2. (i) 2 (ii) (iii) (iv)
x +2 3a 7 x +1
4
x +1 6x 2 − 11x + 3 1
3. (i) (ii) (iii)
x+5 2x 2 − 9x − 5 x +3
41
(vii) a + a +1
2
(viii) 4z4 – 4z2 – 15 (ix) 970299
20
(x) 1092727 (xi) a2 + 2ab – 11a + 30 (xii) 4x2 + 24xz + 35z2
4. 15616
5. (i) x7y6(1 + x15y14) (ii) 3ab(a – 3b) (a + 3b) (a2 + 9b2)
(iii) 3a2(a2 + 2b2)2 (iv) (a2 – 4b3)2
(v) 3(x2 + 2xy + 2y2) (vi) (x4 – 2x2 + 9)(x4 + 2x2 + 9)
(vii) (x +9)(x + 7) (viii) (x – 3)(x – 9)
(ix) (x + y)(7x – 6y) (x) (x – 2) (5x + 2)
(xi) (x – 3y) (x + 3y)(x2 – 3xy + 9y2) (x2 + 3xy + 9y2)
(xii) (5a2 + 4b2)(25a4 – 20a2b2 + 16b4)
6. (i) x3(1 – x) (ii) 10(x – 1)
7. (i) (x2 – y2) (x2 – xy + y2 (ii) x4 + x2y2 + y4
2x 2 + 2 x+ 2
8. (i) (ii)
x3 − x2 − x +1 x+ 3
3x 2 − 2x − 1 x 2 − 2x + 1
(iii) 3 (iv) 2
x + 2x 2 − 4x − 8 x − 10x + 25
16
9.
a8 −1
x 4 + 14x 2 + 1
10.
x 4 − 2x 2 + 1
138 Mathematics Secondary Course
Linear Equations MODULE - 1
Algebra
5 Notes
LINEAR EQUATIONS
You have learnt about basic concept of a variable and a constant. You have also learnt
about algebraic exprssions, polynomials and their zeroes. We come across many situations
such as six added to twice a number is 20. To find the number, we have to assume the
number as x and formulate a relationship through which we can find the number. We shall
see that the formulation of such expression leads to an equation involving variables and
constants. In this lesson, you will study about linear equations in one and two variables.
You will learn how to formulate linear equations in one variable and solve them algebraically.
You will also learn to solve linear equations in two variables using graphical as well as
algebraic methods.
OBJECTIVES
After studying this lesson, you will be able to
• identify linear equations from a given collection of equations;
• cite examples of linear equations;
• write a linear equation in one variable and also give its solution;
• cite examples and write linear equations in two variables;
• draw graph of a linear equation in two variables;
• find the solution of a linear equation in two variables;
• find the solution of a system of two linear equations graphically as well as
algebraically;
• Translate real life problems in terms of linear equations in one or two variables
and then solve the same.
(i) 2x + 5 = 8 Notes
(ii) 3y – z = y + 5
(iii) x2 – 2x = x + 3
(iv) 3x – 7 = 2x +3
(v) 2 + 4 = 5 + 1
Solution:
(i) It is a linear equation in x as the exponent of x is 1. LHS = 2x + 5 and RHS = 8
(ii) It is not a linear equation in one variable as it contains two variables y and z. Here,
LHS = 3y – z and RHS = y + 5
(iii) It is not a linear equation as highest exponent of x is 2. Here, LHS = x2 – 2x and RHS
= x +3.
(iv) It is a linear equation in x as the exponent of x in both LHS and RHS is one.
LHS = 3x – 7, RHS = 2x + 3
(v) It is not a linear equation as it does not contain any variable. Here LHS = 2 + 4 and
RHS = 5 + 1.
Example 5.2: Which of the following are linear equations in two variables.
(i) 2x + z = 5
(ii) 3y – 2 = x + 3
(iii) 3t + 6 = t – 1
Solution:
(i) It is a linear equation in two variables x and z.
(ii) It is a linear equation in two variables y and x.
(iii) It is not a linear equation in two variables as it contains only one variable t.
(iii) 5 – 4 = 1
(iv) y2 = 2y – 1
2. Which of the following are linear equations in two variables:
Notes (i) 3y – 5 = x + 2
(ii) x2 + y = 2y – 3
(iii) x + 5 = 2x – 3
y
In (ii), the equation is = 2.
7
In (iii), You can assume the quantity to be found out as a variable say x, i.e., let Reena has
x apples. She gave 5 apples to her sister, hence she is left with x – 5 apples. Hence, the
required equation can be written as x – 5 = 3, or x = 8.
In (iv), Let the digit in the unit place be x. Therefore, the digit in the tens place should be
2x. Hence, the number is
10 (2x) + x = 20x + x = 21x
When the digit are reversed, the tens place becomes x and unit place becomes 2x. Therefore,
the number is 10x + 2x = 12x. Since original number is 18 more than the new number, the
equation becomes
21x – 12x = 18
or 9x = 18
3y
Example 5.6: Solve: −3 = 9
2
Solution: Adding 3 to both sides of the equation, we get
Notes
3y
−3+3 = 9+3
2
3y
or = 12
2
3y
or × 2 = 12 × 2 (Multiplying both sides by 2)
2
or 3y = 24
3y 24
or = (Dividing both sides by 3)
3 3
or y=8
Hence, y = 8 is the solution.
Example 5.7: Solve the equation 2(x + 3) = 3(2x – 7)
Solution: The equation can be written as
2x + 6 = 6x – 21
or 6x – 21 = 2x + 6 [Interchanging LHS and RHS]
or 6x – 21 + 21 = 2x + 6 + 21 [Adding 21 on both sides]
or 6x = 2x +27
or 6x – 2x = 2x +27 – 2x [Subtracting 2x from both sides]
or 4x = 27
27
or x=
4
27
Thus, x = is the solution of the equation.
4
Note:
1. It is not necessary to write the details of what we are adding, subtracting, multiplying
or dividing each time.
2. The process of taking a term from LHS to RHS or RHS to LHS, is called transposing.
3. When we transpose a term from one side to other side, sign ‘+’ changes to ‘–’,
‘–’ to ‘+’.
1
4. y + 9 = 12
3
5. 5(x – 3) = x + 5
or x = 15
Therefore, the present age of Jacob is 15 years and the present age of his father = 3x
= 3 × 15 = 45 years.
Check: After 5 years, age of Jacob = 15 + 5 = 20 years
After 5 years, age of his father = 45 + 5 =50 years
Difference of their ages = 50 – 20 = 30 years
Example 5.10 : The sum of three consecutive even integers is 36. Find the integers.
Solution: Let the smallest integer be x.
Therefore, other two integers are x + 2 and x + 4.
Since, their sum is 36, we have
x + (x + 2) + (x + 4) = 36
or 3x + 6 = 36
or 3x = 36 – 6 = 30
or x = 10
Therefore, the required integers are 10, 12 and 14.
Example 5.11: The length of a rectangle is 3 cm more than its breadth. If its perimeter is
34 cm find its length and breadth.
Solution: Let the breadth of rectangle be x cm
Therefore, its length = x + 3
Now, since perimeter = 34 cm
We have 2(x + 3 + x) = 34
or 2x + 6 + 2x = 34
or 4x = 34 – 6
or 4x = 28
or x=7
Therefore, breadth = 7 cm, and length = 7 + 3 = 10 cm.
2
4. In a class, the number of boys is of the number of girls. Find the number of girls in
5
the class, if the number of boys is 10.
b c
or x = − y−
a a
Now, for each value of y, we get a unique value of x. Thus, a linear equation in two
variables will have infinitely many solutions.
Note: A linear equation ax + c = 0, a ≠ 0, can be considered as a linear equation in two
variables by expressing it as
ax + 0y + c = 0
i.e., by taking the coefficient of y as zero. It still has many solutions such as
c c
x = − , y = 0; x = − , y = 1 etc.
a a
c
i.e., for each value of y, the value of x will be equal to − .
a
Example 5.12: The sum of two integers is 15. Form a linear equation in two variables.
Solution: Let the two integers be x and y. Therefore, their sum = x + y. It is given that the
sum is 15.
Example 5.13: For the equation 4x – 5y = 2, verify whether (i) x = 3, y = 2 and (ii) x =
4, y = 1 are solutions or not.
Solution: (i) We have 4x – 5y = 2
When x = 3, y = 2, LHS = 4x – 5y = 4 × 3 – 5 × 2
= 12 – 10 = 2
= RHS
Therefore, x = 3, y = 2 is a solution of the given equation.
(ii) When x = 4, y = 1, LHS = 4 × 4 – 5 × 1 = 16 – 5 = 11
But RHS = 2. Therefore, LHS ≠ RHS
Hence, x = 4, y = 1 is not a solution.
12 − 3y 12 − 2x
x= or y =
2 3
Now, for each value of y or for each value of x, we get a unique corresponding value of x
or y. We make the following table for the values of x and y which satisfy the equation:
2x + 3y = 12
x 0 6 3 9 –3
y 4 0 2 –2 6
Thus, x = 0, y = 4; x = 6, y = 0; x = 3, y = 2; x = 9, y = –2; x = –3, y = 6 are all solutions
of the given equation.
We write these solutions as order pairs (0, 4), (6, 0), (3, 2), (9, – 2) and (– 3, 6).
Here, first entry gives the value of x and the corresponding second entry gives the value of
y. We will now learn to draw the graph of this equation by plotting these ordered pairs in
a plane and then join them. In the graph of 2x + 3y = 12, the points representing the
solutions will be on a line and a point which is not a solution, will not lie on this line. Each
point also called orderd pair, which lies on the line will give a solution and a point which
does not lie on the line will not be a solution of the equation.
To draw the graph of a linear equation in two variables, we will first plot these points in a
plane. We proceed as follows:
Step 1: We take two perpendicular lines X'OX and YOY' intersecting at O. Mark the real
numbers on X'OX and YOY' by Notes
Y
considering them as number lines with the
point O as the real number 0 as shown in
Fig 5.2. These two lines divide the plane 4
into four parts, called first quadrant, 3
second quadrant, third quadrant and 2
fourth quadrant. The number line X'OX
1
is called x-axis and the line Y'OY is X' X
called y-axis. Since, we have taken x-
–4 –3 –2 –1 O 1 2 3 4
axis and y-axis, perpendicular to each –1
other in a plane, we call the plane as
–2
coordinate plane or cartesian plane in
the honour of French mathematician –3
Descartes who invented this system to –4
plot a point in the plane.
Y'
Step 2: To plot a point say (3, 2), take
the point 3 on x-axis and through this point, Fig 5.2
draw a line 'l' perpendicular to x-axis
(i.e. parallel to y-axis). Now take the point 2 on y-axis and through 2, draw a line 'm'
perpendicular to y-axis (i.e. parallel to x-axis) to meet l at P. The point P represents the
point (3, 2) on the plane.
Y'
3
P
2
(3, 2)
1
X' X
O
–2 –1 1 2 3 4
Fig. 5.3
Note 1: It may be noted that, for the ordered pair (a, b), a is called x-coordinate and b
is called y-coordinate.
Note 2: Every point on x-axis can be written as (a, 0) i.e. its y-coordinate is zero and
every point on y-axis is of the form (0, b) i.e., its x-coordinate is zero. The coordinates of
the point O are (0, 0).
Notes Note 3: In the first quadrant, both x and y coordinates are positive, in the second quadrant,
x coordinate is negative and y coordinate is positive, in the third quadrant both x and
y coordinates are negative and in the fourth quadrant, x-coordinate is positive and
y-coordinate is negative.
Example 5.14: Represent the point (–2, 3) in the coordinate plane.
Solution: Draw x-axis and y-axis on the plane
Y
and mark the points on them. Take the point – l
2 on x-axis and draw the line l parallel to y-
axis. Now take the point 3 on y-axis and draw 4
the line 'm' parallel to x-axis to meet l at P. 3 m
P
The point P represent (–2, 3), we say (–2 , 3) 2
are coordinates of the point P.
1
You will now learn to draw the graph of a linear X' X
equation in two variables. It should be noted –3 –2 –1 O 1 2 3
that the graph of linear equation in two variables
Y'
is a line and the coordinates of every point on
Fig 5.4
the line satisfies the equation. If a point does
not lie on the graph then its coordinates will
not safisfy the equation. You also know that from two given points, one and only one line
can be drawn. Therefore, it is sufficient to take any two points, i.e., values of the variables
x and y which satisfy the equation. However, it is suggested that you should take three
points to avoid any chance of a mistake occurring.
Example 5.15: Draw the graph of the equation 2x – 3y = 6.
Solution: Now choose values of x and y which satisfy the equation 2x – 3y = 6. It will be
easy to write the equation by transforming it in any of the following form
2x = 3y + 6 or 3y = 2x – 6
3y + 6 2x − 6
⇒ x= or y =
2 3
Now by taking different values of x or y, you find the corresponding values of y or x. If we
2x − 6
take different values of x in y = , we get corresponding values of y. If x = 0, we get
3
y = – 2, x = 3 gives y = 0 and x = – 3 gives y = – 4.
You can represent these values in the following tabular form:
x 0 3 –3
y –2 0 –4
The corresponding points in the plane are (0, –2), (3, 0) and (–3, –4). You can now plot Notes
these points and join them to get the line which represents the graph of the linear equation
as shown here.
Note that all the three points must lie on the line.
Y
4
3
2 6
y=
–3
1 2x
X' X
–4 –3 –2 –1 O 1 2 3 4
B (3, 0)
–1
–2 A (0, –2)
–3
4)
,– –4
(–3
C
Y'
Fig 5.5
Example 5.16: Draw the graph of the equation x = 3.
Solution: It appears that it is a linear equation in one variable x. You can easily convert it
into linear equation in two variables by writing it as
x+0y=3
Now you can have the following table for values of x and y.
x 3 3 3
y 3 0 1
Observe that for each value of y, the value of x is always 3. Thus, required points can be
taken as (3, 3), (3, 0), (3, 1). The graph is shown in Fig. 5.6.
C(3, 3,)
3
Notes
2
x=3
1 B(3, 1)
A(3, 0) X
X'
1 2 3 4
Y'
Fig. 5.6
2. Draw the graph of each of the following linear equations in two variables:
(i) x + y = 5 (ii) 3x + 2y = 6
(iii) 2x + y = 6 (iv) 5x + 3y = 4
2x + 3y = 19
3x + 2y = 16
These two equations taken together are called system of linear equations in two variables
and the values of x and y which satisfy both equations simultaneously is called the solution. Notes
There are different methods for solving such equation. These are graphical method and
algebraic method. You will first learn about graphical method and then algebraic method
for solving such equations.
In this method, you have to draw the graphs of both linear equations on the same graph
sheet. The graphs of the equations may be
(i) Intersecting lines: In this case, the point of intersection will be common solution of
both simultaneous equations. The x-coordinate will give the value of x and y-coordinate
will given value of y. In this case system will have a unique solution.
(ii) Concident lines: In this case each point on the common line will give the solution.
Hence, system of equations will have infinitely many solutions.
(iii) Parallel lines: In this case, no point will be common to both equations. Hence, system
of equations will have no solution.
Example 5.17: Solve the following system of equations:
x – 2y = 0 ...(1)
3x + 4y = 20 ...(2)
Solution: Let us draw the graphs of these equations. For this, you need atleast two solutions
of each equation. We give these values in the following tables.
x – 2y =0 3x + 4y = 20
x 0 2 –2 x 0 4 6
y 0 1 –1 y 5 2 1/2
Now plot these points on the same graph sheet as given below:
The two graphs intersect at the point P whose coordinates are (4, 2). Thus x = 4, y = 2 is
the solution.
You can verify that x = 4, y = 2 satisfies both the equations.
7
Notes
6
(0, 5)
5
3x
4 +
4y
=
20
3
P
2
(2, 1) (4, 2)
1 1
(6, )
2
X' X
–3 –2 –1 O 1 2 3 4 5 6 7
=0 –1
2y )
x– , –1
(–2
Y'
Fig. 5.7
Example 5.18: Solve the following system of equations:
x +y=8 ...(1)
2x –y = 1 ...(2)
Solution: To draw the graph of these equation, make the following by selecting some
solutions of each of the equation.
x+y=8 2x – y = 1
x 3 4 5 x 0 1 2
y 5 4 3 y –1 1 3
Now, plot the points (3, 5), (4, 4) and (5, 3) to get the graph of x + y = 8 and (0, –1),
(1, 1) and (2, 3) to get the graph of 2x – y = 1 on the same graph sheet. The two lines
intersect at the point P whose coordinates are (3, 5). Thus x = 3, y = 5, is the solution of
the system of equations. You can verify that x = 3, y = 5 satisfies both equations
simultaneously.
(3, 5)
5
P
(4, 4)
4 Notes
x
(5, 3)
+
3
y=
y
(2, 3)
=
8
–
2
2x
1 (1, 1)
O
X' –2 –1 1 2 3 4 5 X
–1
(0, –1)
–2
T'
Fig. 5.8
Example 5.19: Solve the following system of equations:
x+y=2 ...(1)
2x + 2y = 4 ...(2)
Solution: First make tables for some solutions of each of the equation.
x + y =2 2x + 2y = 4
x 0 2 1 x 0 2 1
y 2 0 1 y 2 0 1
Fig. 5.9
6
=
2
2y
–
1
4x
X'
X
–2 –1 O 1 2 3
–1
–2
4
y=
–3
–
2x
–4
Y'
Fig. 5.10
There are several methods of solving system of two linear equations in two variables. You
have learnt one method which is known as graphical method. We shall now discuss here
two more methods, called algebraic methods. They are
(i) Substitution Method.
(ii) Elimination method.
Note: These methods are useful in case the system of equations has a unique solution.
Substitution Method: In this method, we find the value of one of the variable from one
equation and substitute it in the second equation. This way, the second equation will be
reduced to linear equaion in one variable which we have already solved. We explain this
method through some examples.
Example 5.21: Solve the following system of equations by substitution method.
5x + 2y = 8 ...(1)
3x – 5y = 11 ...(2)
Solution: From (1), we get
2y = 8 – 5x
1
or y= (8 – 5x) ...(3)
2
Substituting the value of y in (2), we get
5
3x − (8 − 5x ) = 11
2
or 6x – 5(8 – 5x) = 22 [multiplying both sides by 2]
or 6x – 40 + 25x = 22
or 31x = 40 + 22
62
or x= =2
31
1 1
y= (8 – 5× 2) = (8 – 10)
2 2
Notes
2
or y= − =–1
2
So, the solution to the system of equations is x = 2, y = – 1.
Example 5.22: Solve the following system of equations by substitution method:
2x + 3y = 7 ...(1)
3x + y = 14 ...(2)
Solution: From equation (2), we get
y = 14 – 3x ...(3)
Substituting the value of y in (1), we get
2x + 3 (14 – 3x) = 7
or 2x + 42 – 9x = 7
or 2x – 9x = 7 – 42
or – 7x = – 35
− 35
Therefore x= =5
−7
Elimination Method: In this method, we eliminate one of the variable by multiplying both Notes
equations by suitable non-zero contants to make the coefficients of one of the variable
numerically equal. Then we add or subtract one equation to or from the other so that one
variable gets eliminated and we get an equation in one variable. We now consider some
examples to illustrate this method.
Example 5.23: Solve the following system of equations using elimination method.
3x – 5y = 4 ...(1)
9x – 2y = 7 ...(2)
Solution: To elminate x, multiply equation (1) by 3 to make coefficient of x equal. You get
the equations.
9x – 15y = 12 ...(3)
9x – 2y = 7 ...(4)
Subtracting (4) from (3), we get
9x – 15y – (9x – 2y) = 12 – 7
or 9x – 15y – 9x + 2y = 5
or – 13 y = 5
5
or y= −
13
5
Substituting y = − in equation (1), we get
13
⎛ 5⎞
3x – 5 × ⎜ − ⎟ = 4
⎝ 13 ⎠
25
or 3x + =4
13
25 27
or 3x = 4 − =
13 13
9
or x=
13
9 5
Therefore, x = and y = − is the required solution of the given system of equations.
13 13
Example 5.24: Solve the following system of equations using elimination method.
Notes
2x + 3y = 13 ...(1)
5x – 7y = –11 ...(2)
Solution: To eliminate y, multiply equation (1) by 7 and equation (2) by 3, we get
14x + 21y = 91 ...(3)
15x – 21y = –33 ...(4)
Adding (3) and (4), we get
29 x = 58
58
or x= =2
29
Substituting x = 2 in (1), we get
2 × 2 + 3y = 13
or 3y = 13 – 4 = 9
9
or y= =3
3
Therefore, x = 2 and y = 3 is the solution of the given system of equations.
LET US SUM UP
• An equation in one variable of degree one is called a linear equation in variable.
Notes
• The general form of a linear equation in one variable is ax + b = 0, a ≠ 0, a and b are
real numbers.
• The value of the variable which satisfies the linear equation is called its solution or root.
• To solve a word problem, it is first translated into algebriac statements and then solved.
• The general form of a linear equation in two variables is ax + by + c =0, where a, b, c
are real numbers and atleast one of a or b is non zero.
• The equation ax + c = 0 can be expressed as linear equation in two variables as
ax + 0y + c = 0.
• To draw the graph of a linear equation in two variables, we find atleast two points in
plane whose coordinates are solutions of the equation and plot them.
• The graph of a linear equaion in two variables is a line.
• To solve two simultaneous equations in two variables, we draw their graphs on the
same graph paper.
(i) if graph is intersecting lines, point of intersection gives unique solution.
(ii) If graph is the same line, system has infinitely many solutions
(iii) If graph is parallel lines, system of equation has no solution
• Algebraic methods of solving system of linear equations are
(i) Substitution method
(ii) Elimination method
• To solve word problems, we tranlate the given information (data) into linear equations
and solve them.
TERMINAL EXERCISE
1. Choose the correct option:
(i) Which one of the following is a linear equation in one variable?
(A) 2x + 1 = y –3 (B) 3t – 1 = 2t + 5
(C) 2x – 1 = x2 (D) x2 – x +1 = 0
(ii) Which one of the following is not a linear equation?
(A) 5 + 4x = y + 3 (B) x + 2y = y – x
(C) 3 – x = y2 + 4 (D) x + y = 0
(iii) Which of the following numbers is the solution of the equation 2(x + 3) = 18?
(A) 6 (B) 12
Notes (C) 13 (D) 21
(iv) The value of x, for which the equation 2x – (4 – x) = 5 – x is satisfied, is:
(A) 4.5 (B) 3
(C) 2.25 (D) 0.5
(v) The equation x – 4y = 5 has
(A) no solution (B) unique solution
(C) two solutions (D) infinitely many solutions
2. Solve each of the following equations
x+2
(i) 2z + 5 = 15 (ii) = −2
3
4 − 2y y + 1
(iii) + =1 (iv) 2.5x – 3 = 0.5x +1
3 2
3. A certain number increased by 8 equals 26. Find the number.
4. Present ages of Reena and Meena are in the ration 4 : 5. After 8 years, the ratio of their
ages will be 5 : 6. Find their present ages.
5. The denominator of a rational number is greater than its numerator by 8. If the
denominaor is decreased by 1 and numerator is increased by 17, the number obtained
3
is . Find the rational number
2
6. Solve the following system of equations graphically:
(i) x – 2y = 7 (ii) 4x + 3y = 24
x+y=–2 3y – 2x = 6
(iii) x + 3y = 6 (iv) 2x – y = 1
2x – y = 5 x+y =8
7. Solve the following system of equations :
(i) x + 2y – 3 = 0 (ii) 2x + 3y = 3
x – 2y + 1= 0 3x + 2y = 2
(iii) 3x – y = 7 (iv) 5x – 2y = –7
4x – 5y = 2 2x + 3y = –18
5.7
1. x = 4, y = 1, unique solution
4. No solution
5. x= 2, y = 1, unique solution
5.8
1. x = 8, y = 6 2. x = –2, y = 5
3. x = 5, y = –2 4. x = 1, y = 3
5.9
1. x = 2, y = –3 2. x = 1, y = 2
3. x = 11, y = 2 4. x = 1, y = 3
5. x = 5, y = –2 6. x = 6, y = –1
5.10
1. 14 years, 42 years
2. 7 m
3. 75 prizes Rs 500 and 125 prizes of Rs 100 each.
4. 17 of Rs 100 each and 16 of Rs 50 each.
1. (i) (B) (ii) (C) (iii) (A) (iv) (C) (v) (D)
3. 18
13
5.
21
(iii) x = 3, y = 1 (iv) x = 3, y = 5
7. (i) x = 1, y = 1 (ii) x = 0, y = 1
Notes
(iii) x = 3, y = 2 (iv) x = –3, y = –4
8. 74
10. 10 m, 6m
Notes
6
QUADRATIC EQUATIONS
In this lesson, you will study about quadratic equations. You will learn to identify quadratic
equations from a collection of given equations and write them in standard form. You will
also learn to solve quadratic equations and translate and solve word problems using quadratic
equations.
OBJECTIVES
After studying this lesson, you will be able to
• identify a quadratic equation from a given collection of equations;
• write quadratic equations in standard form;
• solve quadratic equations by (i) factorization and (ii) using the quadratic formula;
• solve word problems using quadratic equations.
(i) It is a quadratic equation since 3x2 = 5 can be written as 3x2 – 5 = 0 and 3x2 – 5 is a
quadratic polynomial.
(ii) x2 + 2x + 3 = 0 is a quadratic equation as x2 + 2x + 3, is a polynomial of degree 2.
(iii) x3 + 1 = 3x2 can be written as x3 – 3x2 + 1 = 0. LHS is not a quadratic polynomial
since highest exponent of x is 3. So, the equation is not a quadratic equation.
(iv) (x + 1) (x + 3) = 2x + 1 is a quadratic equation, since (x + 1) (x + 3) = 2x + 1 can be
written as
x2 + 4x + 3 = 2x + 1
or x2 + 2x + 2 = 0
Now, LHS is a polynomial of degree 2, hence (x + 1) (x + 3) = 2x + 1 is a quadratic
equation.
1 5
(v) x + = is not a quadratic equation.
x 2
However, it can be reduced to quadratic equation as shown below:
1 5
x+ =
x 2
x2 +1 5
or = ,x ≠ 0
x 2
or 2(x2 + 1) = 5x , x ≠ 0
or 2x2 – 5x + 2 = 0, x ≠ 0
x2 +1 5
(iii) (5y + 1) (3y – 1) = y + 1 (iv) =
x +1 2
(v) 3x + 2x2 = 5x – 4
3 1
Therefore, x = − and x = are solutions of the given equation.
2 3
Example 6.5: Solve x2 + 2x + 1 = 0
Notes
Solution: We have x2 + 2x + 1 = 0
or (x + 1)2 = 0
or x+1=0
which gives x=–1
Therefore, x = –1 is the only solution.
Note: In Examples 6.3 and 6.4, you saw that equations had two distinct solutions. However,
in Example 6.5, you got only one solution. We say that it has two solutions and these are
coincident.
Quadratic Formula
Now you will learn to find a formula to find the solution of a quadratic equation. For this,
we will rewrite the general quadratic equation ax2 + bx + c = 0 by completing the square.
We have ax2 + bx + c = 0
Multiplying both sides by '4a' to make the coefficient of x2 a perfect square, of an even
number, we get
4a2x2 + 4abx + 4ac = 0
or (2ax)2 + 2(2ax)b + (b)2 + 4ac = b2 [adding b2 to both sides]
or (2ax)2 + 2(2ax)b + (b)2 = b2 – 4ac
(2ax + b )2 = {± }
2
or b 2 − 4ac
or 2ax + b = ± b 2 − 4ac
− b ± b 2 − 4ac
or x=
2a
This gives two solutions of the quadratic equation ax2 + bx + c = 0. The solutions (roots) Notes
are:
− b + b 2 − 4ac − b − b 2 − 4ac
and
2a 2a
Here, the expression (b2 – 4ac), denoted by D, is called Discriminant, because it
determines the number of solutions or nature of roots of a quadratic equation.
For a quadratic equation ax2 + bx + c = 0, a ≠ 0, if
2
− b + b 2 − 4ac
(i) D = b – 4ac > 0, the equation has two real distinct roots, which are
2a
− b − b 2 − 4ac
and .
2a
−b
(ii) D = b2 – 4ac = 0, then equation has two real equal roots, each equal to
2a
(iii) D = b2 – 4ac < 0, the equation will not have any real root, since square root of a
negative real number is not a real number.
Thus, a quadratic equation will have at the most two roots.
Example 6.6: Without determining the roots, comment on the nature (number of solutions)
of roots of the following equations:
(i) 3x2 – 5x – 2 = 0
(ii) 2x2 + x + 1 = 0
(iii) x2 + 2x + 1 = 0
Solution: (i) The given equation is 3x2 – 5x – 2 = 0. Comparing it with ax2 + bx + c = 0,
we get a = 3, b = – 5 and c = – 2.
Now D = b2 – 4ac = (–5)2 – 4 × 3 × (–2)
= 25 + 24 = 49
− b ± b 2 − 4ac 19 ± 1 19 ± 1
x= = =
2a 12 12
19 + 1 5 19 − 1 3
So, roots are = and =
12 3 12 2
5 3
Thus, the two roots are and .
3 2
Example 6.8: Find the value of m sothat the equation 3x2 + mx – 5 = 0 has equal roots.
Solution: Comparing the given equation with ax2 + bx + c = 0
We have, a = 3, b = m, c = – 5
For equal roots
D = b2 – 4ac =0
or m2 – 4 × 3 × (–5) = 0
or m2 = 60
This gives m = ± 2 15
− 6 ± 36 + 864 − 6 ± 900
Therefore y= =
2 2
− 6 ± 30
or y=
2
− 6 + 30 − 6 − 30
Therefore, y= or
2 2
or y = 12 or – 18
Since, side of square can not be negative, so y = 12
Therefore, x = y + 6 = 12 + 6 = 18
Hence, sides of squares are 18 m and 12 m.
Example 6.11: The product of digits of a two digit number is 12. When 9 is added to the
number, the digits interchange their places. Determine the number.
Solution: Let the digit at ten's place be x
and digit at unit's place be y
Therefore, number = 10 x + y
When digits are interchanged, the number becomes 10y + x
Therefore 10x + y + 9 = 10y + x
4
Example 6.12: The sum of two natural numbers is 12. If sum of their reciprocals is ,
9
find the numbers.
Solution: Let one number be x
Therefore, other number = 12 – x
4
Since, sum of their reciprocals is , we get
9
1 1 4
+ = , x ≠ 0, 12 − x ≠ 0
x 12 − x 9
12 − x + x 4
=
x (12 − x ) 9
or
12 4
or =
12x − x 2
9
12 × 9
or = 12x − x 2
4
or 27 = 12x – x2
or x2 – 12x + 27 = 0
or (x – 3) (x – 9) = 0
It gives x = 3 or x = 9
Notes
When first number x is 3, other number is 12 – 3 = 9 and when first number x is 9, other
number is 12 – 9 = 3.
Therefore, the required numbers are 3 and 9.
LET US SUM UP
• An equation of the form ax2 + bx + c = 0, a ≠ 0 and a, b, c are real numbers is called
a quadratic equation in standard form.
• The value(s) of the variable which satisfy a quadratic equation are called it roots or
solutions.
• The zeros of a quadratic polynomial are the roots or solutions of the corresponding
quadratic equation.
• If you can factorise ax2 + bx + c = 0, a ≠ 0, into product of linear factors, then the
roots of the quadratic equation ax2 + bx + c = 0, can be obtained by equating each
factor to zero.
• Roots of the quadratic equation ax2 + bx + c = 0, a ≠ 0 are given by
− b ± b 2 − 4ac
2a
TERMINAL EXERCISE
1. Which of the following are quadratic equations?
( )
(i) y 5 y − 3 = 0 (ii) 5x 2 − 3 x + 8 = 0
1
(iii) 3x − =5 (iv) x(2x + 5) = x2 + 5x + 7
x
2. Solve the following equations by factorisation method:
(i) (x – 8) (x + 4) = 13 (ii) 3y2 – 7y = 0
(iii) x2 + 3x – 18 = 0 (iv) 6x2 + x – 15 = 0
3. Find the value of m for which 5x2 – 3x + m = 0 has equal roots.
4. Find the value of m for which x2 – mx – 1 = 0 has equal roots.
5. Solve the following quadratic equations using quadratic formula:
(i) 6x2 – 19x + 15 =0 (ii) x2 + x – 1 = 0
(iii) 21 + x = 2x2 (iv) 2x2 – x – 6 = 0
6. The sides of a right angled triangle are x – 1, x and x + 1. Find the value of x and hence
the sides of the triangle.
7. the sum of squares of two consecutive odd integers is 290. Find the integers.
8. The hypotenuse of a right angled triangle is 13 cm. If the difference of remaining two
sides is 7 cm, find the remaining two sides.
9. The sum of the areas of two squares is 41 cm2. If the sum of their perimeters is 36 cm,
find the sides of the two squares.
10. A right angled isosceles triangle is inscribed in a circle of radius 5 cm. Find the sides of
the triangle.
Notes 6.1
1. (ii), (iii), (v)
6.2
1. (i) No, 3y2 – y – 3 = 0 (ii) No, 2x2 + 2x – 5 =0
(iii) No, 6t2 + t – 1 = 0 (iv) No, 3x2 + x – 5 = 0
6.3
3 7
1. (i) , −2 (ii) 3, – 6 (iii) , −1
2 3
1 1 3 1
(iv) 2, 3 (v) , (vi) ,
5 5 2 2
6.4
1. (i) Two real distinct roots
(ii) Two real equal roots
(iii) Two real equal roots
(iv) No real roots
9
3. (i) ± 2 2 (ii) (iii) 3 (iv) For no value of m
20
6.5
1. 8, 10 2. 16m, 9m 3. 85, 58
4. 83 (v) 5, 10
7 3 5
2. (i) 8, 4 (ii) 0, (iii) 3, – 6 (iv) ,−
3 2 3
9
3.
20
4. For no value of m
Notes
3 5 − 1± 5 7 3
5. (i) , (ii) (iii) ,−3 (iv) 2,
2 3 2 2 2
6. 3, 4, 5
7. 11, 13 or –13 , –11
8. 5 cm, 12 cm
9. 5 cm, 4 cm
Notes 7
ARITHMETIC PROGRESSIONS
In your daily life you must have observed that in nature, many things follow patterns such
as petals of flowers, the holes of a honey-comb, the spirals on a pine apple etc. In this
lesson, you will study one special type of number pattern called Arithmetic Progression
(AP). You will also learn to find general term and the sum of first n terms of an arithmetic
progression.
OBJECTIVES
After studying this lesson, you will be able to
• identify arithmetic progression from a given list of numbers;
• determine the general term of an arithmetic progression;
• find the sum of first n terms of an arithmetic progression.
(i) Rita deposits ` 1000 in a bank at the simple interest of 10% per annum. The amount
at the end of first, second, third and fourth years, in rupees will be respectively
1100, 1200, 1300, 1400
Do you observe any pattern? You can see that amount increases every year by a fixed
amount of ` 100.
Notes
Can you see any pattern in the list of these numbers? You can observe that
1 = 12, 4 = 22, 9 = 32, 16 = 42, ...
i.e., these are squares of natural numbers.
Now consider some more lists of numbers and try to recognise a pattern if possible:
1, 3, 5, 7, 9 ..... (1)
2, 4, 6, 8, 10 ... (2)
1, 4, 7, 10, 13 .... (3)
5, 3, 1, –1, –3... (4)
1, 3, 9, 27, 81, ... (5)
2, 3, 5, 7, 11, 13... (6)
You can observe that numbers in the list (1) are odd natural numbers. The first number is
1, second number is 3, third number is 5, etc. All these numbers follow a pattern. The
pattern is that all these numbers, except the first is obtained by adding 2 to its previous
number.
In lists (2), (3) and (4), each number except the first is obtained by adding 2, 3, and – 2
respectively to its previous number.
In (5), each number, except the first is obtained by multiplying 3 to its previous number. In
the list (6), you can see that it is the list of prime numbers and it is not possible to give any
rule till date, which gives the next prime number.
The numbers in a list are generally denoted by
a1, a2, a3, ...., an, ...
or t1, t2, t3, ...., tn, ...
which are respectively called first, second, third and nth term in the list of numbers. We
sometimes call each of these lists as sequence or pattern of numbers.
100 × 101
or S = = 5050
2
We shall use the same method to find the sum of first ‘n’ terms of an AP.
The first ‘n’ terms of an AP are
a, a + d, a + 2d, ..., a + (n – 2)d, a + (n – 1)d
Let us denote the sum of n terms by Sn. Therefore,
Sn = a + (a + d) + (a + 2d) + .... + [a + (n – 2)d] + [a + (n – 1)d] (3)
Writing these terms in reverse order, we get
Sn = [a + (n – 1)d] + [a + (n – 2)d] + ... + (a + d) + a (4)
We now add (3) and (4), term by term. We can see that the sum of any term in (3) and the
corresponding term in (4) is 2a + (n – 1)d. We get
2Sn = [2a + (n – 1)d] + [2a + (n – 1)d] + ... + [2a + (n – 1)d] + [2a + (n – 1)d] , n times
or 2Sn = n[2a + (n – 1)d]
n
or Sn = [2a + (n – 1)d],
2
which gives general formula for finding the sum of first ‘n’ terms of an AP.
This can be rewritten as
n
Sn = [a +{a+ (n – 1)d}]
2
n
= (a + tn), [as nth term tn = a + (n – 1)d]
2
n
Sn = (a + l) (4)
2
Example 7.7: Find the sum of the first 12 terms of the following AP Notes
(i) 11, 16, 21, 26 ....
(ii) – 151, – 148, – 145, – 142
Solution: (i) The given AP is
11, 16, 21, 26 ....
Here, a = 11, d = 16 – 11 = 5 and n = 12.
You know that sum of first n terms of an AP is given by
n
Sn = [2a + (n – 1)d]
2
12
Therefore, S12 = [2 × 11 + (12 – 1)5]
2
= 6 [22 + 55] = 6 × 77 = 462
Hence, required sum is 462.
n
Sn = [2a + (n – 1)d]
2
Hence, sum of first 12 terms is
12
S12 = [2 × (– 151) + (12 – 1)3]
2
= 6[– 302 + 33] = 6 × (– 269)
= – 1614
Therefore, required sum is – 1614.
Example 7.8: How may terms of the AP 2, 4, 6, 8, 10 .... are needed to get sum 210?
Solution: For the given AP, a = 2, d = 2 and Sn = 210.
n
We have: Sn = [2a + (n – 1)d]
Notes 2
n
or 210 = [2 × 2 + (n – 1)2]
2
or 420 = n[2n + 2]
or 420 = 2n2 + 2n
or 2n2 + 2n – 420 = 0
or n2 + n – 210 = 0
or n2 + 15n – 14n – 210 = 0
or n(n + 15) – 14(n + 15) = 0
or (n + 15) (n – 14) = 0
or n = – 15 or n = 14
Since, n cannot be negative, so, n = 14
Therefore, first 14 terms are needed to get the sum 210.
Example 7.9: Find the following sum
2 + 5 + 8 + 11 + .... + 59
Solution: Here 2, 5, 8, 11, ... are in AP and a = 2, d = 3 and tn = 59.
To find the sum, you need to find the value of n.
Now, tn = a + (n – 1) d
So, 59 = 2 + (n – 1) 3
or 59 = 3n – 1
or 60 = 3n
Therefore, n = 20
n
Now, Sn = [2a + (n – 1)d]
2
20
or S20 = [2 × 2 + (20 – 1)3]
2
n
Now, Sn = [a + l ]
2
142
= [7 + 994] = 71×1001
2
= 71071
Therefore, required sum is 71071.
Example 7.11: The sum of first three terms of an AP is 36 and their product is 1620. Find
the AP.
Solution: We can take three terms of the AP as a, a + d and a + 2d. However, the product
will be rather difficult and solving the two equations simultaneoulsy will be time consuming.
The elegant way is to assume the first three terms as a – d, a and a + d, so that the sum of
three terms becomes 3a.
Let first three terms of the AP b a – d, a and a + d
Therefore, a – d + a + a + d = 36
or 3a = 36,
which gives a = 12
Now, since product is 1620, we have:
(a – d) a (a + d) = 1620
or (12 – d) 12 (12 + d) = 1620
or 122 – d2 = 135
or 144 – d2 = 135
or d2 = 9
Therefore, d = 3 or – 3
Notes If d = 3, the numbers are 12 – 3, 12 and 12 + 3
i.e. 9, 12, 15 (Since a = 12)
If d = – 3, the numbers are15, 12 and 9
Therefore, the first three terms of the AP 9, 12, 15 and 15, 12, 9
satisfy the given conditions.
LET US SUM UP
• A progression in which each term, except the first, is obtained by adding a constant to
the previous term is called an AP.
• The first term of an AP is denoted by a and common difference by d.
n
• The sum of first n terms of an AP is given by Sn = [2a + (n – 1)d]
2
• The sum of an AP whose first term is a and last term is l and number of terms is n is Notes
n
given by Sn = (a + l)
2
TERMINAL EXERCISE
1. Which of the following patterns are arithmetic progressions?
(i) 2, 5, 8, 12, 15, ....
(ii) – 3, 0, 3, 6, 9 .....
(iii) 1, 2, 4, 8, 16, .....
2. Write the nth term of each of the following arithmetic progressions:
(i) 5, 9, 13, 17, ....
(ii) – 7, – 11, – 15, – 19
3. The fourth term of an AP is equal to three times its first term and seventh term exceeds
twice the third term by 1. Find the first term and common difference.
4. The 5th term of an AP is 23 and 12th term is 37. Find the first term and common
difference.
5. The angles of a triangle are in AP. If the smallest angle is one-third the largest angle,
find the angles of the triangle.
6. Which term of AP
(i) 100, 95, 90, 85, ...., is – 25?
1 1 3 5 25
(ii) , , ,1, ..... is ?
4 2 4 4 4
7. The nth term of an AP is given by tn = a + bn. Show that it is an AP. Find its first term
and common difference.
8. If 7 times the 7th term of an AP is equal to 11 times the 11th term, show that the 18th
term is zero.
9. Each term of an AP whose first term is a and common difference is d, is doubled. Is the
resulting pattern an AP? If so, find its first term and common difference.
10. If k + 2, 4k – 6 and 3k – 2 are three consecutive terms of an AP, find k.
5
[Hint: If AP is a, a + d, a + 2d, ... , then S5 = [a + (a + 4d)]
2
In the next five terms, the first term is a + 5d and last term is a + 9d.
14. If sum of first n terms of an AP is 2n + 3n2, find rth term of the A.P. [Hint tr = Sr – Sr-1]
15. Find the sum of all 3-digit numbers which leave the remainder 1, when divided by 4.
[Hint: First term = 101, last term = 997]
3 220
(iii) d = 375, Sn = – 11400 (iv) a = − , Sn =
8 3
3. 3, 2
4. 15, 2
5. 30o, 60o, 90o
6. (i) 26th term (ii) 25th term
7. a + b, b
9. Yes, first term = 2a, common difference = 2d
10. 3 11. (i) 22 terms (ii) 13 terms
12. 10,000 14. 6r – 1 15. 123525
Secondary Course
Mathematics
Notes Practice Work-Algebra
Instructions:
1
2. The reciprocal of
(− 3 / 5)−2 is 1
2
3
(A) −
5
2
−5
(B)
3
(C) (− 5 / 3)−2
−2
⎛ 3⎞
(D) ⎜ ⎟
⎝ 5⎠
Notes
3. In an A.P., the sum of three numbers is 15 and their product is 45. Then the three
numbers are 1
(A) 1, 3, 15
(B) 2, 4, 9
(C) 1, 5, 9
(D) 0, 5, 9
x −1 1
4. If y = , then 2 y − is equal to 1
x +1 2y
3x 2 − 10 x − 3
(A)
(
2 x2 −1 )
3 x 2 − 10 x + 1
(B)
x2 −1
3x 2 + 10 x + 3
(C)
(
2 x2 −1 )
3x 2 − 10 x + 3
(D)
(
2 x2 −1 )
4 x 2 − 25
5. The lowest form of the expression 2 is 1
2 x + 11x = 15
2x − 5
(A)
x+3
2x + 5
(B)
x+3
2x – 5
(C)
x−3
2x − 5
(D)
x−3
−3 −11 x
⎛7⎞ ⎛8⎞ ⎛7⎞
Notes 6. Find x, so that ⎜ ⎟ × ⎜ ⎟ =⎜ ⎟ : 2
⎝8⎠ ⎝7⎠ ⎝8⎠
8. The HCF of two polynomials is (x–2) and their LCM is x4 + 2x3 – 8x – 16. If one of
the polynomials is x3 – 8, find the other polynomial. 2
50
9. The sum of a number and its reciprocal is , find the number. 2
7
10. The length of a rectangle is 5 cm less than twice its breadth. If the perimeter is 110 cm,
find the area of the rectangle. 2
11. Show that the sum of an AP whose first term is a, the second term is b and the last term
(a + c )(b + c − 2a )
2(b − a )
is c, is equal to . 4
12. Had Ajay scored 10 more marks in his test out of 30 marks, 9 times these marks
would have been the square of his actual marks. How many marks did he get in the
test? 6
8 Notes
You must have seen advertisements in newspapers, television and hoardings etc of the
following type:
“Sale, up to 60% off”.
“Voters turnout in the poll was over 70%”.
“Ramesh got 93% aggregate in class XII examination”.
“Banks have lowered the rate of interest on fixed deposits from 8.5% to 7%”.
In all the above statements, the important word is ‘percent’. The word ‘percent’ has been
derived from the Latin word ‘percentum’ meaning per hundred or out of hundred.
In this lesson, we shall study percent as a fraction or a decimal and shall also study its
applications in solving problems of profit and loss, discount, simple interest, compound
interest, rate of growth and depreciation etc.
OBJECTIVES
After studying this lesson, you will be able to
• illustrate the concept of percentage;
• calculate specified percent of a given number or a quantity;
• solve problems based on percentage;
• solve problems based on profit and loss;
• calculate the discount and the selling price of an article, given marked price of
the article and the rate of discount;
• solve inverse problems pertaining to discount;
• calculate simple interest and the amount, when a given sum of money is invested
for a specified time period on a given rate of interest;
8.1 PERCENT
3 7
Recall that a fraction means 3 out of 4 equal parts. means 7 out of 13 equal parts
4 13
23
and means 23 out of 100 equal parts.
100
23
A fraction whose denominator is 100 is read as percent, for example is read as
100
twenty three percent.
The symbol ‘%’ is used for the term percent.
A ratio whose second term is 100 is also called a percent,
So, 33 : 100 is equivalent to 33%.
3 1
Recall that while comparing two fractions,and , we first convert them to equivalent
5 2
fractions with common denominator (L.C.M. of the denominators).
3 3 2 6
thus = × = , and
5 5 2 10
1 1 5 5
= × =
2 2 5 10
6 5 3 1
Now, because > ∴ >
10 10 5 2
3 3 20 60
= × = or 60%
5 5 20 100
1 1 50 50 Notes
= × = or 50%
2 2 50 100
3 1
and so, > as 60% is greater than 50%.
5 2
3 3 25 75 1
= × = = 75 × = 75% and
4 4 25 100 100
4 4 4 16 1
= × = = 16 × = 16%
25 25 4 100 100
Note: To write a fraction as percent, we may multiply the fraction by 100, simplify
it and attach % symbol. For example,
4 4
= × 100% = 16%
25 25
Conversely,
To write a percent as a fraction, we drop the % sign, multiply the number by
1
(or divide the number by 100) and simplify it. For example,
100
1 47 1 17 3
47% = 47 × = , 17% = 17 × = , 3% =
100 100 100 100 100
1 45 9 210 21 x
45% = 45 × = = , 210% = = , x% = .
100 100 20 100 10 100
47 470 1
4.7 = = = 470 × = 470%
10 100 100
459 459 1
0.459 = = × = 45.9%
1000 10 100
63 63 1
0.0063 = = × = 0.63%
10000 100 100
Thus, to write a decimal as a percent, we move the decimal point two places to the
right and put the % sign
Conversely,
To write a percent as a decimal, we drop the %sign and insert or move the decimal
point two places to the left. For example,
43% = 0.43 75% = 0.75 12% = 0.12
9% = 0.09 115% = 1.15 327% = 3.27
0.75% = 0.0075 4.5% = 0.045 0.2% = 0.002
Let us take a few more examples:
Example 8.1: Shweta obtained 18 marks in a test of 25 marks. What was her percentage
of marks?
Solution: Total marks = 25
Marks obtained = 18
18
∴ Fraction of marks obtained =
25
18 4 72
∴ Marks obtained in percent = × = = 72%
25 4 100
Alternatively:
18
Marks obtained in percent = × 100% = 72%
25
1
Solution: Fraction of the total number of shoes on sale =
4
Notes
1 3
∴ Fraction of the total number of shoes on normal price = 1 − =
4 4
3 25 75 3
= × = = 75% or × 100% = 75%
4 25 100 4
Example 8.3: Out of 40 students in a class, 32 opted to go for a picnic. What percent of
students opted for picnic?
Solution: Total number of students in a class = 40
Number of students, who opted for picnic = 32
∴ Number of students, in percent, who opted for picnic
32
× 100% = 80%
=
40
Example 8.4: In the word ARITHMETIC, what percent of the letters are I’s?
Solution: Total number of letters = 10
Number of I’s = 2
2
∴ Percent of I’s = × 100% = 20%
10
Example 8.5: A mixture of 80 litres, of acid and water, contains 20 litres of acid. What
percent of water is in the mixture?
Solution: Total volume of the mixture = 80 litres
Volume of acid = 20 litres
∴ Volume of water = 60 litres
60
∴ Percentage of water in the mixture = × 100% = 75%
80
Notes 7
(a) 53% (b) 85% (c) 16 % (d) 3.425% (e) 6.25%
8
3
(f) 70% (g) 15 % (h) 0.0025% (i) 47.35% (j) 0.525%
4
3. Write each of the following decimals as a percent:
(a) 0.97 (b) 0.735 (c) 0.03 (d) 2.07 (e) 0.8
(f) 1.75 (g) 0.0250 (h) 3.2575 (i) 0.152 (j) 3.0015
4. Write each of the following percents as a decimal:
(a) 72% (b) 41% (c) 4% (d) 125% (e) 9%
(f) 410% (g) 350% (h) 102.5% (i) 0.025% (j) 10.25%
5. Gurpreet got half the answers correct, in an examination. What percent of her answers
were correct?
6. Prakhar obtained 18 marks in a test of total 20 marks. What was his percentage of
marks?
7. Harish saves ` 900 out of a total monthly salary of ` 14400. Find his percentage of
saving.
8. A candidate got 47500 votes in an election and was defeated by his opponent by a
margin of 5000 votes. If there were only two candidates and no votes were declared
invalid, find the percentage of votes obtained by the winning candidate.
9. In the word PERCENTAGE, what percent of the letters are E’s?
10. In a class of 40 students, 10 secured first division, 15 secured second division and 13
just qualified. What percent of students failed.
25
25% of 90 = × 90 = 22.50
100
or 25% of 90 = 0.25 × 90 = 22.50
60% of Rs. 120 = 0.60 × Rs. 120 = Rs. 72.00
120% of 80 kg = 1.20 × 80 kg = 96 kg
x
∴ × 360 = 144
100
144
Or x= × 100 = 40%
360
144
Alternatively, 144 out of 360 is equal to the fraction
360
144
∴ Percent = × 100% = 40%
360
Example 8.9: If 120 is reduced to 96, what is the reduction percent?
Solution: Here, reduction = 120 – 96 = 24
24
∴ Reduction percent = × 100% = 20%
120
60 690 × 100
∴ × x = 690 or x = = 1150
100 60
∴ Total number of students in the school = 1150
∴ Hence number of boys = 1150 – 690 = 460
Example 8.12: A’s income is 25% more than that of B. B’s income is 8% more than that
of C. If A’s income is ` 20250, then find the income of C.
Solution: Let income of C be ` x
Income of B = x + 8% of x
8 x 108
= x+ = ×x
100 100
108 x 108 x
Income of A = + 25% of
100 100
108 x 125
= ×
100 100
108 125
∴ × x× = 20250
100 100
100 100
or x = 20250 × × = 15000
108 125
∴ Income of C is ` 15000.
15 x 115 x
x+ = 19320 or = 19320
100 100
19320 × 100
∴x = = 16800
115
Hence, the required sum = ` 16800.
Let us recall the terms and formulae related to profit and loss.
Cost Price (C.P.): The Price at which an article is purchased, is called its cost price.
Selling Price (S.P.): The Price at which an article is sold, is called its selling price.
Profit (Gain): When S.P. > C.P., then there is profit, and
Profit = S.P. – C.P.
⎛ Profit ⎞ ⎛ Loss ⎞
Formulae Profit % = ⎜ × 100 ⎟%, Loss% = ⎜ × 100 ⎟%
⎝ C.P. ⎠ ⎝ C.P. ⎠ Notes
S.P. ×100
=
(S.P.)×100
C.P. =
(100 + Profit% ) (100 − Loss%)
Note: Gain % or loss % is always calculated on C.P.
Let us take some examples to illustrate the applications of these formulae in solving problems
related to profit and loss:
Example 8.16: A shopkeeper buys an article for Rs. 360 and sells it for Rs. 270. Find his
gain or loss percent.
Solution: Here C.P. = Rs. 360, and S.P. = Rs. 270
Since C.P. > S.P., ∴ there is a loss.
Loss = C.P. – S.P. = Rs (360 – 270) = Rs. 90
⎛ Loss ⎞
Loss % = ⎜ × 100 ⎟%
⎝ C.P. ⎠
90
= × 100 = 25%
360
Example 8.17: Sudha purchased a house for ` 4,52,000 and spent ` 28,000 on its
repairs. She had to sell it for ` 4,92,000. Find her gain or loss percent.
Solution: Here C.P. = Cost price + Overhead charges
= ` (452000 + 28000) = ` 4,80,000
S.P. = ` 4,92,000
Since, S.P. > C.P., ∴ Gain = ` (492000 – 480000) = ` 12000
12000 × 100 5
Gain % = = % = 2 .5 %
480000 2
Example 8.18: By selling a book for ` 258, a publisher gains 20%. For how much should
he sell it to gain 30%?
100
and S.P. of 1 orange = ` =`5
20
∴ Profit on 1 orange = ` (5 – 4) = ` 1
1
Profit % = × 100 = 25%
4
Example 8.20: A man sold two horses for ` 29700 each. On one he lost 10% while he
gained 10% on the other. Find his total gain or loss percent in the transaction.
Solution: S.P. of first horse = ` 29700
Loss = 10%
29700 × 100
∴ C.P. = ` = ` 33,000
90
S.P. of 2nd horse = ` 29700,
Profit = 10%
29700 × 100
C.P. = ` = ` 27,000
110
Total CP = ` (33000 + 27000) = ` 60,000
Total SP = ` (2 × 29700) = ` 59400
Net Loss = ` (60000 – 59400) = ` 600
15 75
S.P. of 15 articles = ` × 15 = `
12 4
⎛ 75 ⎞ 15
Gain = ` ⎜ −15 ⎟ = `
⎝ 4 ⎠ 4
15 / 4
Gain % = × 100 = 25%
15
Example 8.22: A watch was sold at a profit of 12%. Had it been sold for ` 33 more, the
profit would have been 14%. Find the cost price of the watch.
Solution: Let the cost price of the watch be ` x
x × 112 112 x
∴ S.P. = =
100 100
⎛ 112x ⎞
If the watch is sold for Rs. 33 more then S.P. = ⎜ + 33 ⎟
⎝ 100 ⎠
New profit = 14%
⎛ 112 x ⎞
⎜ + 33 ⎟ ×100
∴ C.P. = x = ⎝ ⎠
100
114
8.5.2 Discount
You must have seen advertisements of the following types, especially during the festival
season.
SALE
discount upto 50%
} DIWALI BONANZA
20% discount on all items.
A discount is a reduction in the marked (or list) price of an article. “20% discount”means
a reduction of 20% in the marked price of an article. For example, if the marked price of
an article is ` 100, it is sold for ` 80, i.e. ` 20 less than the marked price. Let us define the
terms, we shall use:
Marked Price (or List price): The marked price (M.P.) of an article is the price at which
the article is listed for sale. Since this price is written (marked) on the article, so it is called
the marked price.
Discount: The discount is the reduction from the marked price of the article.
Net selling price (S.P.): In case of discount selling, the price of the article obtained by
subtracting discount from the marked price is called the Net Selling price or Selling price
(S.P.). Let us take the following examples, to illustrate:
2100
Discount % = × 100% = 25%
8400
Note: Discount is always calculated on Marked Price.
Example 8.25: A wholesaler’s list price of a fan is ` 1250 and is available to a retailer at
a discount of 20%. For how much should the retailer sell it, to earn a profit of 15%.
Solution: M.P. = ` 1250
Discount = 20% of ` 1250
20
=` × 1250 = ` 250
100
∴ Cost Price of the retailer = ` (1250 – 250)
= ` 1000
Profit = 15%
⎛ 10 ⎞
= ` 125 – ` ⎜ ×125 ⎟
⎝ 100 ⎠
= ` (125 – 12.50) = ` 112.50
∴ Gain = ` (112.50 – 100) = ` 12.50
12.50
Gain % = × 100 = 12.5%
100
Example 8.27: An article listed at ` 5400 is offered at a discount of 15%. Due to festival
season, the shopkeeper allows a further discount of 5%. Find the selling price of the
article.
Solution: M.P. = ` 5400, Discount = 15%
∴ SP = ` 5400 – 15% of ` 5400
15
= ` 5400 – ` × 5400
100
= ` (5400 – 810) = ` 4590
Festival discount = 5%
∴ Net selling Price = ` 4590 – 5% of ` 4590
5
= ` 4590 – ` × 4590
100
= ` (4590 – 229.50)
= ` 4360.50
∴ Net selling price of article = ` 4360.50.
Example 8.28: A retailer buys books from a wholesaler at the rate of ` 300 per book and
marked them at ` 400 each. He allows some discount and gets a profit of 30% on the cost
price. What percent discount does he allow to his customers?
10
Discount % = × 100 = 2.5%
400
When a person has to borrow some money as a loan from his friends. relatives, bank etc.
he promises to return it after a specified time period along with some extra money for using
the money of the lender.
The money borrowed is called the Principal, usually denoted by P, and the extra money
paid is called the Interest, usually denoted by I.
1
(b) ` 20,000 15% 1 yrs
2
Solution: (a) I = P. R. T.
⎡ 5 ⎤
= ` ⎢8000 × × 2⎥ = ` 800
⎣ 100 ⎦
⎡ 15 3 ⎤
I = ` ⎢20000 × ×
100 2 ⎥⎦
(b) = ` 4500
⎣
Example 8.30: Find at what rate of simple interest per annum will ` 5000 amount to
` 6050 in 3 years.
Solution: Here A = ` 6050, P = ` 5000, T = 3 yrs
∴ I = ` (6050 – 5000) = ` 1050
I I × 100
I = P × R × T or r% = ∴r =
P×T P×T
1050 × 100
r= =7 ∴ R = 7%
5000 × 3
1 25 Notes
Solution: Here A = ` 4875, R = 12 % = %, T = 4 yrs
2 2
I=P×R×T
⎛ 25 ⎞ P
I = ` ⎜P× × 4⎟ = `
⎝ 200 ⎠ 2
⎛ P⎞ 3P
∴ A = ` ⎜P + ⎟ = `
⎝ 2⎠ 2
3P
Thus, = ` 4875 or 3P = ` 9750 or P = ` 3250
2
Example 8.32: In how many years will a sum of ` 2000 yield an interest (Simple) of
` 560 at the rate of 14% per annum?
Solution: Here P = ` 2000, I = ` 560 R = 14%
14
I = P × R × T or 560 = 2000 × ×T
100
560 × 100
∴T = = 2 years
2000 × 14
Thus, in 2 years, a sum of ` 2000 will yield an interest of ` 560 at 14% per annum.
Example 8.33: A certain sum of money at simple interest amounts to ` 1300 in 4 years
and to ` 1525 in 7 years. Find the sum and rate percent.
P×R ×4
Solution: Here 1300 = +P ...(i)
100
P×R ×7
and 1525 = +P ...(ii)
100
P×R ×3 P×R
Subtracting (i) from (ii) 225 = or = 75
100 100
Putting in (i) we get
1300 = 75 × 4 + P or P = ` (1300 – 300) = ` 1000
225
∴ Interest for 1 year = ` = ` 75
3
∴ 1300 = P + Interest for 4 yrs = P + 4 × 75 or P = ` (1300 – 300) = ` 1000
75 × 100
R= = 7 .5 %
1000 × 1
Example 8.34: A certain sum of money doubles itself in 10 years. In how many years will
1
it become 2 times at the same rate of simple interest.
2
Solution: Let P = ` 100, T = 10 yrs, A = ` 200, ∴ I = ` 100
100 × R × 10
∴100 = or R = 10%
100
Now P = ` 100, R = 10% and A = ` 250 ∴ I = ` 150
10
∴150 = 100 × × T or T = 15 yrs
100
1
Thus, in 15 yrs, the sum will become 2 times.
2
Example 8.35: Out of ` 70,000 to invest for one year, a man invests ` 30,000 at 4% and
` 20,000 at 3% per annum simple interest. At what rate percent, should he lend the
remaining money, so that he gets 5% interest on the total amount he has?
Solution: Interest on total amount at 5% for one year
5
= ` 70,000 × × 1 = ` 3500
100
4
Interest on ` 30,000 at 4% for 1 year = ` 30000 × ×1
100
= ` 1200
R 1700 × 100
∴1700 = 20000 × × 1 or R = = 8.5%
100 20000
∴ The remaining amount should be invested at 8.5% per annum.
In the previous section, you have studied about simple interest. When the interest is calculated
on the Principal for the entire period of loan, the interest is called simple interest and is
given by
Notes
I=P×R×T
But if this interest is due (not paid) after the decided time period, then it becomes a part of
the principal and so is added to the principal for the next time preiod, and the interest is
calculated for the next time period on this new principal. Interest calculated, this way is
called compound interest.
The time period after which the interest is added to the principal for the next time period is
called the Conversion Period.
The conversion period may be one year, six months or three months and the interest is said
to compounded, annually, semi-annually or quarterly, respectively. Let us take an example:
Example 8.36: Find the compound interest on a sum of Rs. 2000, for two years when the
interest is compounded annually at 10% per annum.
Solution: Here P = ` 2000 and R = 10%
∴ Interest for the first conversion time period (i.e. first year)
10
= ` 2000 × × 1 = ` 200
100
∴ Principal for the second year (or 2nd conversion period)
= ` (2000 + 200) = ` 2200
10
∴ Interest for the 2nd time period = ` 2200 × × 1 = ` 220
100
∴ Amount payable at the end of two years = ` (2200 + 220)
= ` 2420
∴ Total interest paid at the end of two years = ` (2420 – 2000)
= ` 420
or [` (200 + 220) = ` 420]
∴ Compound interest = ` 420
Thus, for calculating the compound interest, the interest due after every coversion period
is added to the principal and then interest is calculated for the next period.
8.5.4.1 Formula for Compound Interest
Let a sum P be borrowed for n years at the rate of r% per annum, then
r Pr
Interest for the first year = P × ×1=
100 100
⎛ r ⎞ Pr ⎛ r ⎞ ⎛ r ⎞⎛ r ⎞
Amount after 2 years = P⎜1 + ⎟+ ⎜1 + ⎟ = P⎜1 + ⎟⎜ 1 + ⎟
⎝ 100 ⎠ 100 ⎝ 100 ⎠ ⎝ 100 ⎠⎝ 100 ⎠
2
⎛ r ⎞
= P⎜1 + ⎟
⎝ 100 ⎠
3
⎛ r ⎞
Similarly, amount after 3 years = P⎜1 + ⎟ and so on.
⎝ 100 ⎠
n
⎛ r ⎞
Amount after n years = P⎜1 + ⎟
⎝ 100 ⎠
r
Thus, if A represents the amount and R represents r% or , then
100
n
⎛ r ⎞
A = P(1 + R) = P⎜1 +
n ⎟
⎝ 100 ⎠
and compound interest = A – P = P (1 + R)n – P
⎡⎛ r ⎞
n
⎤
n
= P[(1 + R) –1] or P ⎢ ⎜ 1 + ⎟ − 1⎥
⎣⎢⎝ 100 ⎠ ⎥⎦
Note: Simple interest and compound interest are equal for first year (first conversion period)
Example 8.37: Calculate the compound interest on ` 20,000 for 3 years at 5% per
annum, when the interest is compounded annually.
Solution: Here P = ` 20,000, R = 5% and n =3
∴ CI = P[(1 + R)n –1]
⎡⎛ 5 ⎞
3
⎤
= ` 20000 ⎢ ⎜ 1 + ⎟ − 1⎥
⎣⎢⎝ 100 ⎠ ⎦⎥
⎡⎛ 21 ⎞3 ⎤ ⎡ 9261 − 8000 ⎤
= ` ⎢⎜ 20 ⎟ − 1⎥ = ` 20000 × ⎢ ⎥⎦
⎢⎣⎝ ⎠ ⎥⎦ ⎣ 8000
= ` 3152.50
1
and n = 1 yrs = 3 half years
2
⎡⎛ 5 ⎞
3
⎤
∴ CI = P[(1 + R) –1] = `
n 20,000 ⎢ ⎜ 1 + ⎟ − 1⎥
⎣⎢⎝ 100 ⎠ ⎦⎥
⎡ 9261 ⎤
= ` 20,000 × ⎢ − 1 = ` 3152.50
⎣ 8000 ⎥⎦
Example 8.39: Calculate the compound interest on ` 20,000 for 9 months at the rate of
4% per annum, when the interest is compounded quarterly.
Solution: Here P = ` 20,000, R = 4% per annum
= 1% per quarter of year
and n = 3/4 yrs = 3 quarters
⎡⎛ 1 ⎞
3
⎤
∴ CI = P[(1 + R) –1] = `
n 20,000 ⎢⎜ 1 + ⎟ − 1⎥
⎢⎣⎝ 100 ⎠ ⎥⎦
= ` 606.02
1
Example 8.40: calculate the amount and compound interest on ` 12000 for 1 years at
2
the rate of 10% per annum compounded annually.
1
Solution: Here P = ` 12000, R = 10% and n = 1 years
2
Since interest is compounded, annually, so, amount at the end of 1 year is given by
11
= ` 12000 × = ` 13200 Notes
10
∴ Principal for next 6 months = ` 13200
10
and Rate R = % = 5%
2
1
⎛ 5 ⎞ 21
∴ A = ` 13200⎜1 + ⎟ = ` 13200 ×
⎝ 100 ⎠ 20
= ` 13860
1
∴ Amount after 1 years = ` 13860
2
Compound interest = ` [13860 – 12000]
= ` 1860
1
Note: We can calculate the amount for 1 yrs as
2
1 1
⎛ 10 ⎞ ⎛ 5 ⎞
A = ` 12000 ⎜1 + ⎟ ⎜1 + ⎟
⎝ 100 ⎠ ⎝ 100 ⎠
Example 8.41: At what rate percent per annum, will a sum of ` 15,625 become
` 17576 in three years, when the interest is compounded annually?
Solution: Here A = ` 17576, P = ` 15,625 and n = 3
Let R = r% per annum
3
⎛ r ⎞
∴17576 = 15625⎜1 + ⎟
⎝ 100 ⎠
3 3
⎛ r ⎞ 17576 ⎛ 26 ⎞
∴ ⎜1 + ⎟ = =⎜ ⎟
⎝ 100 ⎠ 15625 ⎝ 25 ⎠
⎛ r ⎞ 26 r 26 1
∴ ⎜1 + ⎟= or = −1 =
⎝ 100 ⎠ 25 100 25 25
1
∴ Time = 3 half years = 1 years
2
Example 8.43: Find the difference between simple interest and compound interest for
1
1 years at 4% per annum, for a sum of ` 24000, when the interest is compounded semi-
2
annually..
Solution: Here P = ` 24000, R = 4% per annum
3
T= years R = 2% per semi-annually
2
1 3
n=1 years = years = 3 semi years
2 2
4 3
Simple Interest = P × R × T = ` 24000 × ×
100 2
= ` 1440.
⎡ R ⎞ ⎤
n
For compound interest, A = P ⎢⎛⎜1 + ⎟ ⎥
⎢⎣⎝ 100 ⎠ ⎥⎦
⎡⎛ 2 ⎞ ⎤
3
A = ` 24000⎢⎜1 + 100 ⎟ ⎥
⎣⎢⎝ ⎠ ⎦⎥
1
Example 8.44: A sum of money is invested at compound interest for 1 year at 4%
2
compounded annually. If the interests were compounded semi-annually, it would have
fetched ` 20.40 more than in the previous case. Find the sum.
Solution: Let the sum be ` x.
Here R = 4% annually, or 2% semi-annually
1
T=1 yrs or 3 semi years
2
In first case
1 1
⎡ 4 ⎤ ⎡ 2 ⎤
A = ` x ⎢1 + ⎢⎣1 + 100 ⎥⎦
⎣ 100 ⎥⎦
⎛ 26 ⎞⎛ 51 ⎞ 1326 x
= ` x⎜ ⎟⎜ ⎟ = `
⎝ 25 ⎠⎝ 50 ⎠ 1250
In 2nd case
3 3
⎛ 2 ⎞ ⎛ 51 ⎞
A = ` x ⎜1 + ⎟ = ` x⎜ ⎟
⎝ 100 ⎠ ⎝ 50 ⎠
132651
=`
125000
⎡ 132651 1326 ⎤
∴ Difference = ` ⎢ x− x
⎣125000 1250 ⎥⎦
51x
=`
125000
In our daily life, we come across the terms like growth of population, plants, viruses etc
and depreciation in the value of articles like machinery, crops, motor cycles etc.
The problems of growth and depreciation can be solved using the formula of compound
interest derived in the previous section.
n
⎛ r ⎞
Vn = Vo ⎜1 – ⎟ in case of depreciation.
⎝ 100 ⎠
If the rate of growth/depreciation varies for each conversion period, then
⎛ r ⎞⎛ r ⎞⎛ r ⎞
Vn = Vo ⎜1 + 1 ⎟⎜1 + 2 ⎟⎜1 + 3 ⎟.... for growth, and
⎝ 100 ⎠⎝ 100 ⎠⎝ 100 ⎠
⎛ r ⎞⎛ r ⎞⎛ r ⎞
Vn = Vo ⎜1 − 1 ⎟⎜1 – 2 ⎟⎜1 – 3 ⎟.... for depreciation.
⎝ 100 ⎠⎝ 100 ⎠⎝ 100 ⎠
Let us take some examples to illustrate the above concepts.
Example 8.45: The population of a city is 9765625. What will be its population after 3
years, if the rate of growth of population is 4% per year?
Solution: Here Vo = 9765625, r = 4% and n = 3
3
⎡ 4 ⎤
∴ V3 = 9765625 ⎢1 +
⎣ 100 ⎥⎦
3
⎛ 26 ⎞
= 9765625 × ⎜ ⎟
⎝ 25 ⎠
= 10985000.
Hence, the population of that city after 3 years will be = 10985000.
Example 8.46: The cost of a car was ` 3,50,000 in January 2005. If the rate of depreciation
is 15% for the first year and 10% for the subsequent years, find its value after 3 years.
Solution: Here Vo = ` 3,50,000
r1 = 15%, r2 = 10% and r3 = 10%
⎛ r ⎞⎛ r ⎞⎛ r ⎞
∴ V3 = Vo ⎜1 − 1 ⎟⎜1 – 2 ⎟⎜1 – 3 ⎟
⎝ 100 ⎠⎝ 100 ⎠⎝ 100 ⎠
17 9 9
= ` 350000 × × × = ` 2,40,975/-
Notes 20 10 10
∴ The value of car after 3 years = ` 240975.
Example 8.47: A plant gains its height at the rate of 2% per month of what was its height
in the beginning of the month. If its height was 1.2 m in the beginning of January 2008, find
its height in the beginning of April 2008, correct upto 3 places of decimal.
Solution: Here Vo = 1.2 m, r = 2%, n = 3
n
⎛ r ⎞
∴ V3 = Vo ⎜1 + ⎟
⎝ 100 ⎠
3 3
⎛ 2 ⎞ ⎛ 51 ⎞
= 1.2⎜1 + ⎟ = 1.2⎜ ⎟ = 1.2734 m
⎝ 100 ⎠ ⎝ 50 ⎠
= 1.273 m
Hence, height of plant in the beginning of April = 1.273 m.
Example 8.48: The virus of a culture decreases at the rate of 5% per hour due to a
medicine. If the virus count in the culture at 11.00 AM was 2.3 × 107, find the virus count
at 1.00 PM on the same day.
Solution: Vo = 2.3 × 107, r = 5%, n =2
2
⎛ 5 ⎞
V2 = 2.3 × 10 7 ⎜1 − ⎟ = 2.3 × 10 × (0.95)
7 2
⎝ 100 ⎠
= 2.076 × 107
Hence, the virus count at 1.00 PM is 2.076 × 107.
LET US SUM UP
• Percent means ‘per hundred’.
• Percents can be written as fractions as well as decimals and vice-versa.
• To write a percent as a fraction, we drop the % sign and divide the number by 100.
• To write a fraction as a percent, we multiply the fraction by 100, simplify it and suffix
the % sign.
• To determine the specific percent of a number or quantity, we change the percent to a
fraction or a decimal and then multiply.
• When the selling price is more than the cost price of the goods, there is a profit (or
gain).
• When the selling price is less than the cost price of the goods, there is a loss.
Profit (Gain) = S.P. – C.P. ; Loss = C.P. – S.P.
Gain Loss
Gain% = × 100 ; Loss% = × 100
C.P. C.P.
100 + Gain% 100 – Loss%
S.P. = × C.P. ; S.P. = × C.P.
100 100
• The simple interest (I.) on a principal (P) at the rate of R% for a time T years, is
calculated, using the formula
I. = P × R × T
⎛ r ⎞⎛ r ⎞⎛ r ⎞
Vn = Vo ⎜1 + 1 ⎟⎜1 + 2 ⎟⎜1 + 3 ⎟.... for growth, and
⎝ 100 ⎠⎝ 100 ⎠⎝ 100 ⎠
⎛ r ⎞⎛ r ⎞⎛ r ⎞
Vn = Vo ⎜1 − 1 ⎟⎜1 – 2 ⎟⎜1 – 3 ⎟.... for depreciation.
⎝ 100 ⎠⎝ 100 ⎠⎝ 100 ⎠
TERMINAL EXERCISE
1. Write each of the following as a percent
9 7
(a) (b) (c) 0.34 (d) 0.06
20 10
2. Write each of the following as a decimal:
(a) 36% (b) 410% (c) 2% (d) 0.35%
1
26. Simple intrest on a sum of money is rd of the sum itself and the number of years is
3
thrice the rate percent. Find the rate of interest.
27. In what time will ` 2700 yield the same interest at 4% per annum as ` 2250 in 4 years
at 3% per annum?
28. The difference between simple interest on a sum of money for 3 years and for 2 years
at 10% per annum is ` 300. Find the sum.
29. Find the sum which when invested at 4% per annum for 3 years will becomes
` 70304, when the interest is compounded annually.
30. the difference between compound interest and simple interest at 10% per annum in 2
years (compounded annually) is ` 50. Find the sum.
31. A sum of money becomes ` 18522 in three years and ` 19448.10 in 4 years at the
same rate of interest, compounded annually. Find the sum and the rate of interest per
annum.
32. Find the sum of money which will amount to ` 26460 in six months at 20% per annum,
when the interest is compounded quarterly.
33. At what rate percent per annum will a sum of ` 12000 amount to ` 15972 in three
years, when the interest is compounded annually?
34. The price of a scooter depreciates at the rate of 20% in the first year, 15% in the
second year and 10% afterwards, what will be the value of a scooter now costing
` 25000, after 3 years.
35. The population of a village was 20,000, two years ago. It increased by 10% during
first year but decreased by 10% in the second year. Find the population at the end of
2 years.
9 Notes
INSTALMENT BUYING
You must have seen advertisements like, “Pay just ` 500 and take home a color TV, rest
in easy instalments”, or “buy a car of your choice by paying ` 50,000 and the balance in
easy instalments”. Such plans attract customers, specially the common man, who could
not buy some costly articles like car, scooter, fridge, colour TV, etc. due to cash constraints.
Under these plans, a fixed amount is paid at the time of purchase and the rest of the
amount is to be paid in instalments, which may be monthly, quarterly, half yearly or yearly,
as per the agreement signed between the customer and the seller.
Instalment purchase scheme, thus, enables a person to buy costly goods, on convenient
terms of payment. Under this scheme, the customer, after making a partial payment in the
beginning, takes away the article for use after signing the agreement to pay the balance
amount in instalments. Such a scheme also encourages the buyer to save at regular intervals,
so as to pay the instalments.
In this lesson, we shall study different types of instalment plans and shall find out how much
easy they are, by calculating the interest charged under these plans.
OBJECTIVES
After studying this lesson, you will be able to
• explain the advantages/disadvantages of buying a commodity under instalment
plan;
• determine the amount of each instalment, when goods are purchased under
instalment plan at a given rate of interest (simple interest);
• determine the rate of interest when the amount of each (equal) instalment and
the number of instalments is given;
• determine the amount of each instalment under instalment plan when compound
interest is charged yearly, half yearly or quarterly;
• solve problems pertaining to instalment plan.
7r
or = 28 i.e., r = 40, i.e. rate = 40%
10
⎛ r 2⎞
` 240 at the end of two months will become = ` ⎜ 240 + 240 × × ⎟
⎝ 100 12 ⎠
⎛ 2r ⎞
= ` ⎜ 240 + ⎟ ...(i)
⎝ 5⎠
r 1 ⎛ 5r ⎞
= ` 125 + 125 × × = Rs. ⎜125 + ⎟ ...(ii)
100 12 ⎝ 48 ⎠
2r 5r ⎛2 5 ⎞
∴ 240 + = 125 + + 125 i.e., ⎜ − ⎟ r = 10
5 48 ⎝ 5 48 ⎠
2400
⇒r= = 33.8 (approx)
71
Hence, rate of interest = 33.8%
Alternative method:
Cash price of the fan = ` 450
Cash down payment = ` 210
Payment in 2 instalments = ` (125 × 2) = ` 250
Total amount paid under instalment plan = ` (210 + 250)
= ` 460
∴ Interest paid = ` (460 – 450) = ` 10
The Principal for the first month = ` (450 – 210) = ` 240
Principal for the 2nd month = ` (240 – 125) = ` 115
16 1
∴ (3x − 1520 ) = (4560 − 3x ) .
100 12
30 1
So, (5x − 14000 ) = (70000 − 10x )× ×
100 12
40 (5x – 14000) = 10(7000 – x)
20x – 56000 = 7000 – x
or 21x = 63000
or x = 3000
Thus, the amount of each instalment = ` 3000
Example 9.7: The cost of a washing machine is ` 12000. The company asks for ` 5200
in advance and the rest to be paid in equal monthly instalments. The rate of interest to be
charged is 12% per annum. If a customer can pay ` 1400 each month, then how many
instalments he will have to pay?
Solution: Let number of instalments be ‘n’
Cash price of washing machine = ` 12000
Price under instalment plan = ` (5200 + 1400n)
∴ Interest charged = ` (5200 + 1400n – 12000)
= ` (1400n – 6800)
Principal owed each month is
First month = ` 6800
2nd month = ` 5400
12 1
So, 20000 × × = (1400n − 6800 )
100 12
1400 n = 7000 i.e. n = 5
Thus, the number of instalments = 5
3 20 x − 2500
∴ (4600 − x ) = (x − 2500 )× × =
12 100 20
20(4600 – x) = x – 2500
or 21x = 92000 + 2500
(1530 − x ) = (3 x − 2250 )× 1 16
× =
(x − 750 )
So,
12 100 25
25(1530 – x) = x – 750
or 26x = 38250 + 750 = 39000
39000
or x= = 1500
26
Thus, the cash price of mixi = ` 1500
2
25 ⎛ 25 ⎞
This gives P1 = x and P2 = ⎜ ⎟ x
26 ⎝ 26 ⎠
2
25 ⎛ 25 ⎞ 25 ⎛ 25 ⎞ 25 51
Hence, 12750 = x+⎜ ⎟ x = x⎜1 + ⎟ = . x
26 ⎝ 26 ⎠ 26 ⎝ 26 ⎠ 26 26
26 26
⇒ x = 12750 × × = 6760
25 51
Thus, each instalment = ` 6760.
Example 9.13: A juicer is available for ` 3500 cash but was sold under instalment plan
where the purchaser agreed to pay ` 1500 cash down and 3 equal quarterly instalments.
If the dealer charges interest at 12% p.a. compounded quarterly, find the amount of each
instalment to the nearest rupee.
Solution: Cash price of the juicer = ` 3500
Cash down payment = ` 1500
Balance to be paid = ` (3500 – 1500) = ` 2000
12
Rate of interest = 12% p.a. = = 3% quarterly
4
103 (103)
2
x = 2000 × × = ` 707
100 30909
∴ Each instalment = ` 707
Example 9.14: A television set is sold for ` 7110 cash down payment along with
2 equal monthly instalments of ` 5581.50 each. If the dealer charges interest at 20% p.a.
compounded monthly under the instalment plan, find the cash price of the television set.
Solution: Cash down payment = ` 7110
11163
Amount of each monthly instalment = ` 5581.50 = `
2
20
Rate of interest = 20% p.a. = monthly
12
Let P1, P2 be the Principals for 1st and 2nd instalment respectively
2
11163 ⎛ 20 ⎞ 11163 ⎛ 20 ⎞
= P1 ⎜1 + ⎟ and = P2 ⎜1 + ⎟
2 ⎝ 1200 ⎠ 2 ⎝ 1200 ⎠
11163 60 11163 60 60
This gives P1 = × = Rs.5490 and P2 = × × = Rs.5400
2 61 2 61 61
Thus, cash Price = ` [7110 + 5490 + 5400] = ` 18000
Example 9.15: A dealer offeres a micro-oven for ` 5800 cash. A customer agrees to pay
` 1800 cash down and 3 equal annual instalments. If the dealer charges interest at 12%
p.a. compounded annually, what is the amount of each instalment.
Solution: Cash price of the micro-oven = ` 5800
Cash down payment = ` 1800
Balance to be paid = ` 4000
25 ⎛ 25 625 ⎞
or x⎜1 + + ⎟ = 4000
28 ⎝ 28 784 ⎠
28 784
or x = 4000 × × = ` 1665.40
25 2109
Hence each instalment = ` 1665.40
Example 9.16: A flat is available for ` 1600000 cash or ` 585500 cash down payment
and three equal half yearly instalments. If the interest charged is 16% per annum
compounded half yearly, calculate the value of each instalment. Find also the total interest
charged.
Solution: Cash price of the flat = ` 1600000
Cash down payment = ` 585500
Balance to be paid = ` 1014500
Rate of interest = 16% per annum = 8% semi annually
Let the amount of each instalment be ` x and Let P1, P2 and P3 be the Principals
for each instalment respectively.
⎛ 8 ⎞ ⎛ 27 ⎞ ⎛ 25 ⎞
So, x = P1 ⎜1 + ⎟ or x = P1 ⎜ ⎟ or P1 = x⎜ ⎟
⎝ 100 ⎠ ⎝ 25 ⎠ ⎝ 27 ⎠
2 3
⎛ 25 ⎞ ⎛ 25 ⎞
Similarly, P2 = x⎜ ⎟ and P3 = x⎜ ⎟
⎝ 27 ⎠ ⎝ 27 ⎠
∴ P1 + P2 + P3 = 1014500
⎛ 25 ⎞ ⎡ 25 ⎛ 25 ⎞ ⎤
2
x⎜ ⎟ ⎢1 + + ⎜ ⎟ ⎥ = 1014500 Notes
⎝ 27 ⎠ ⎣⎢ 27 ⎝ 27 ⎠ ⎦⎥
25 2029
x. . = 1014500
27 729
1014500 × 27 × 729
x=
25 × 2029
= ` 393660
Interest paid = ` [393660 × 3 – 1014500]
= ` [1180980 – 1014500]
= ` 166480.
1. A bicycle is available for ` 1661 cash or by paying ` 400 cash down and balance in
three equal half yearly instalments. If the interest charged is 10% per annum compounded
semi-annually, find the instalment.
2. A washing machine is available for ` 15000 cash or ` 2000 cash down with two equal
half yearly instalments. If the rate of interest charged is 16% per annum compounded
half yearly, find the instalment.
4. A car was available for ` 70000 cash or by paying ` 21200 cash down along with
three equal annual instalments. If the dealer charges interest of 25% per annum,
compounded annually, find the amount of each instalment.
5. A microwave oven was purchased by paying a cash down payment of ` 2800 along
with 2 equal annual instalments of ` 2420 each. If the rate of interest charged under
the instalment plan was 10% p.a. compounded annually, find the cash price of the
article.
LET US SUM UP
• Under an instalment scheme, the customer, after making a partial payment in the
Notes beginning takes away the article for use, after signing the agreement to pay the balance
amount in instalments.
• Under instalment plan, the buyer pays some extra amount, which is interest on the
deferred payments.
• Instalment scheme encourages the buyer to save at regular intervals, so as to pay the
instalments.
• The price at which the article is available, if full payment is made in cash, is called the
cash price of the article.
• The partial payment made at the time of purchase under instalment plan is called Cash
down payment.
• The payments, which the buyer has to make at regular intervals, are called instalments.
TERMINAL EXERCISE
1. A sewing machine is available for ` 2600 cash payment or under an instalment plan for
` 1000 cash down payment and 3 equal monthly instalments of ` 550 each. Find the
rate of interest charged under the instalment plan.
2. Anil purchased a typewriter priced at ` 8000 cash payment under the instalment plan
by making a cashdown payment of ` 3200 and 5 equal monthly instalments of ` 1000
each. Find the rate of interest charged under the instalment plan.
3. A table is sold for ` 2000 cash or ` 500 as cash payment followed by 4 equal monthly
instalments of ` 400 each. Find the rate of interest charged under the instalment plan.
4. A T.V. set has a cash price of ` 7500 or ` 2000 as cash down payment followed by
6 monthly instalments of ` 1000 each. Find the rate of interest charged under instalment
plan.
5. An article is available for ` 7000 cash or for ` 1900 cash down payment and six equal
1
monthly instalments. If the rate of interest charged is 2 % per month, determine each
2
instalment.
6. An article is sold for ` 1000 cash or Rs. 650 cash down payment followed by 5 equal
monthly instalments. If the rate of interest charged is 18% per annum, compute the
monthly instalment.
1 1 1 1
1. 19 % 2. 17 % 3. 33 4. 33
21 7 3 3
5. ` 920 6. ` 63.35 7. 5 8. ` 4000
9. 20.7% 10. 26.43% 11. ` 146.12 12. ` 3000
13. ` 2850.86 14. ` 366 (Approx) 15. ` 220 16. ` 6060
17. ` 53240 18. ` 20,000 19. ` 19448
20. ` 689210, ` 99230
Instructions:
10 Notes
Observe the top of your desk or table. Now move your hand on the top of your table. It
gives an idea of a plane. Its edges give an idea of a line, its corner, that of a point and the
edges meeting at a corner give an idea of an angle.
OBJECTIVES
After studying this lesson, you will be able to
• illustrate the concepts of point, line, plane, parallel lines and interesecting lines;
• recognise pairs of angles made by a transversal with two or more lines;
• verify that when a ray stands on a line, the sum of two angles so formed is 1800;
• verify that when two lines intersect, vertically opposite angles are equal;
• verify that if a transversal intersects two parallel lines then corresponding angles
in each pair are equal;
• verify that if a transversal intersects two parallel lines then
(a) alternate angles in each pair are equal
(b) interior angles on the same side of the transversal are supplementary;
• prove that the sum of angles of a triangle is 1800
• verify that the exterior angle of a triangle is equal to the sum of two interior
opposite angles; and
• explain the concept of locus and exemplify it through daily life situations.
• find the locus of a point equidistent from (a) two given points, (b) two intersecting
lines.
• solve problems based on starred result and direct numerical problems based on
unstarred results given in the curriculum.
A C
Fig. 10.1
A point is used to show the location and is represented by capital letters A, B, C etc.
10.1.1 Line
Now mark two points A and B on your note book. Join them with the help of a ruler or a
scale and extend it on both sides. This gives us a straight line or simply a line.
Fig. 10.2
In geometry, a line is extended infinitely on both sides and is marked with arrows to give
this idea. A line is named using any two points on it, viz, AB or by a single small letter l, m
etc. (See fig. 10.3)
Fig. 10.3
The part of the line between two points A and B is called a line segment and will be named
AB.
Observe that a line segment is the shortest path between two points A and B. (See Fig.
10.4)
Notes
Fig. 10.4
10.1.2 Ray
If we mark a point X and draw a line, starting from it extending infinitely in one direction
only, then we get a ray XY.
Fig. 10.5
X is called the initial point of the ray XY.
10.1.3 Plane
Fig. 10.6
Similarly, floor of a room also gives the idea of part of a plane.
Plane also extends infintely lengthwise and breadthwise.
Mark a point A on a sheet of paper.
How many lines can you draw passing though this point? As many as you wish.
Fig. 10.7
Fig. 10.8
Out of these lines, how many pass through both the points A and B? Out of all the lines
passing through A, only one passes through B. Thus, only one line passes through both the
points A and B. We conclude that one and only one line can be drawn passing through
two given points.
Now we take three points in plane.
C
Fig. 10.9
We observe that a line may or may not pass through the three given points.
If a line can pass through three or more points, then these points are said to be collinear.
For example the points A, B and C in the Fig. 10.9 are collinear points.
If a line can not be drawn passing through all three points (or more points), then they are
said to be non-collinear. For example points P, Q and R, in the Fig. 10.9, are non-
collinear points.
Since two points always lie on a line, we talk of collinear points only when their number is
three or more.
Let us now take two distinct lines AB and CD in a plane.
Fig. 10.10
How many points can they have in common? We observe that these lines can have. either
(i) one point in common as in Fig. 10.10 (a) and (b). [In such a case they are called
10.1.4 Angle
Mark a point O and draw two rays OA and OB starting from O. The figure we get is
called an angle. Thus, an angle is a figure consisting of two rays starting from a common
point.
Fig. 10.11(A)
This angle may be named as angle AOB or angle BOA or simply angle O; and is written as
∠ΑΟΒ or ∠ΒΟΑ or ∠Ο. [see Fig. 10.11A]
An angle is measured in degrees. If we take any point O and draw two rays starting from
it in opposite directions then the measure of this angle is taken to be 1800 degrees, written
as 1800.
B O A
Fig. 10.12
Fig. 10.13
Two lines or rays making a right angle with each other are called perpendicular lines. In
Fig. 10.13 we can say OA is perpendicular to OB or vice-versa.
An angle less than 900 is called an acute angle. For example ∠POQ is an acute angle in
Fig. 10.14(a).
An angle greater than 900 but less than 1800 is called an obtuse angle. For example,
∠XOY is an obtuse angle in Fig. 10.14(b).
(a) (b)
Fig. 10.14
Fig. 10.15
Notes
(a)
(b)
Fig. 10.16
Observe the angles in each pair in Fig. 10.16[(a) and (b)]. They add up to make a total of
90o.
A pair of angles, whose sum is 90o, is called a pair of complementary angles. Each angle
is called the complement of the other.
Again observe the angles in each pair in Fig. 10.17[(a) and (b)].
These add up to make a total of 180o.
A pair of angles whose sum is 1800, is called a pair of supplementary angles.
Each such angle is called the supplement of the other.
Draw a line AB. From a point C on it draw a ray CD making two angles ∠X and ∠Y.
C
Fig. 10.18
Fig. 10.19
∠AOC and ∠DOB are angles opposite to each other. These make a pair of vertically
oppposite angles. Measure them. You will always find that
∠AOC = ∠DOB.
∠AOD and ∠BOC is another pair of vertically opposite angles. On measuring, you will
again find that
∠AOD = ∠BOC
We conclude :
If two lines intersect each other, the pair of vertically opposite angles are
equal.
An activity for you.
Attach two strips with a nail or a pin as shown in the figure.
Fig. 10.20
Fig. 10.21
When a transversal intersects two lines, eight angles are formed.
Fig. 10.22
These angles in pairs are very important in the study of properties of parallel lines. Some of
the useful pairs are as follows :
(a) ∠1 and ∠5 is a pair of corresponding angles. ∠2 and ∠6, ∠3 and ∠7 and ∠4 and
∠8 are other pairs of corresponding angles.
(b) ∠3 and ∠6 is a pair of alternate angles. ∠4 and ∠5 is another pair of alternate angles.
(c) ∠3 and ∠5 is a pair of interior angles on the same side of the transversal.
∠4 and ∠6 is another pair of interior angles.
In Fig. 10.22 above, lines m and n are not parallel; as such, there may not exist any relation
between the angles of any of the above pairs. However, when lines are parallel, there are
some very useful relations in these pairs, which we study in the following:
When a transversal intersects two parallel lines, eight angles are formed, whatever be the
position of parallel lines or the transversal.
Notes
Fig. 10.23
If we measure the angles, we shall alwys find that
∠1 = ∠5, ∠2 = ∠6, ∠3 = ∠7 and ∠4 = ∠8
that is, angles in each pair of corresponding angles are equal.
Also ∠3 = ∠6 and ∠4 = ∠5
that is, angles in each pair of alternate angle are equal.
Also, ∠3 + ∠5 = 180o and ∠4 + ∠6 = 180o.
Hence we conclude :
When a transversal intersects two parallel lines, then angles in
(i) each pair of corresponding angles are equal
(ii) each pair of alternate angles are equal
(iii) each pair of interior angles on the same side of the transversal are supple-
mentary,
You may also verify the truth of these results by drawing a pair of parallel lines (using
parallel edges of your scale) and a transversal and measuring angles in each of these pairs.
Converse of each of these results is also true. To verify the truth of the first converse, we
draw a line AB and mark two points C and D on it.
Fig. 10.24
At C and D, we construct two angles ACF and CDH equal to each other, say 50o, as
shown in Fig. 10.24. On producing EF and GH on either side, we shall find that they do
not intersect each other, that is, they are parallel.
Fig. 10.25
(i) In Fig. 10.25, ∠FOD and ∠BOD are
(A) supplementary angles (B) complementary angles
(C) vertically opposite angles (D) a linear pair of angles Ans. (B)
(ii) In Fig. 10.25, ∠COE and ∠BOE are
(A) complementary angles (B) supplementary angles
(C) a linear pair (D) adjacent angles Ans. (D)
(iii) In Fig. 10.25, ∠BOD is equal to
(A) xo (B) (90 + x)o
(C) (90 – x)o (D) (180 – x)o Ans (C)
(iv) An angle is 4 times its supplement; the angle is
(A) 39o (B) 72o
(C) 108o (D) 144o Ans (D)
Notes
Fig. 10.26
(A) 30o (B) 40o
(C) 50o (D) 60o Ans (C)
Fig. 10.27
In the above figure, l is parallel to m and p is parallel to q.
(vi) ∠3 and ∠5 form a pair of
(A) Alternate angles (B) interior angles
(C) vertically opposite (D) corresponding angles AAns (D)
(vii) In Fig. 10.27, if ∠1 = 80o, then ∠6 is equal to
(A) 80o (B) 90o
(C) 100o (D) 110o Ans (C)
Fig. 10.28
(viii) In Fig. 10.28, OA bisects ∠LOB, OC bisects ∠MOB and ∠AOC = 900. Show that
the points L, O and M are collinear.
Fig. 10.29
In Fig. 10.29, AB || CD and PQ intersects them at R and S respectively.
(i) ∠ARS and ∠BRS form
(A) a pair of alternate angles
(B) a linear pair
(C) a pair of corresponding angles
(D) a pair of vertically opposite angles
(ii) ∠ARS and ∠RSD form a pair of
(A) Alternate angles (B) Vertically opposite angles
(C) Corresponding angles (D) Interior angles
(iii) If ∠PRB = 60o, then ∠QSC is
(A) 120o (B) 60o
Notes
72o
Fig. 10.30
(iv) In Fig. 10.30 above, AB and CD intersect at O. ∠COB is equal to
(A) 36o (B) 72o
(C) 108o (D) 144o
)o
+ 10 o
(5x 5x
Fig. 10.31
2. In Fig. 10.31 above, AB is a straight line. Find x
3. In Fig. 10.32 below, l is parallel to m. Find angles 1 to 7.
Fig. 10.32
Notes
Fig. 10.33
It is a closed figure formed by three line segments having six elements, namely three angles
(i) ∠ABC or ∠B (ii) ∠ACB or ∠C (iii) ∠CAB or ∠A and three sides : (iv) AB (v) BC
(vi) CA
It is named as Δ ABC or Δ BAC or Δ CBA and read as triangle ABC or triangle BAC or
triangle CBA.
Fig. 10.36
In Fig. 10.36 (a), ∠A = 80o, ∠B = 40o and ∠C = 60o
∴ ∠A + ∠B + ∠C = 80o + 40o + 60o = 180o
In Fig. 10.36(b), ∠P = 30o, ∠Q = 40o, ∠R = 110o
∴ ∠P + ∠Q + ∠R = 30o + 40o + 110o = 180o
What do you observe? Sum of the angles of triangle in each case in 1800.
We will prove this result in a logical way naming it as a theorem.
Theorem : The sum of the three angles of triangle is 180o.
Fig. 10.37
Given : A triangle ABC
To Prove : ∠A + ∠B + ∠C = 1800
Construction : Through A, draw a line DE parallel to BC.
Proof : Since DE is parallel to BC and AB is a transversal.
Fig. 10.38
In Fig. 10.39, observe that there are six exterior angles of the ΔABC, namely ∠1, ∠2,
∠3, ∠4, ∠5 and ∠6.
Fig. 10.39
In Fig. 10.38, ∠ACD so obtained is called an exterior angle of the ΔABC. Thus,
The angle formed by a side of the triangle produced and another side of the
triangle is called an exterior angle of the triangle.
Corresponding to an exterior angle of a triangle, there are two interior opposite angles.
Interior opposite angles are the angles of the triangle not forming a linear
pair with the given exterior angle.
For example in Fig. 10.38, ∠A and ∠B are the two interior opposite angles correspond-
ing to the exterior angle ACD of ΔABC. We measure these angles.
∠A = 60o
∠B = 50o
Fig. 10.40
(ii) In Fig. 10.40 ∠A is equal to
(A) 30o (B) 35o
(C) 45o (D) 75o Ans (C)
(iii) In a triangle, one angle is twice the other and the third angle is 600. Then the
largest angle is
(A) 60o (B) 80o
(C) 100o (D) 120o Ans (B)
Example 10.4:
Fig. 10.41
In Fig. 10.41, bisctors of ∠PQR and ∠PRQ intersect each other at O. Prove that
1
∠QOR = 90o + ∠P.
2
1
Solution : ∠QOR = 180o – [∠PQR + ∠PRQ)]
2
1
= 180o – (∠PQR + ∠PRQ) Notes
2
1
= 180o – (180ο – ∠P)
2
1 1
= 180o – 90o + ∠P = 90o + ∠P
2 2
Fig. 10.42
In Fig. 10.42, CD is parallel to BA. ∠ACB is equal to
(A) 55o (B) 60o
(C) 65o (D) 70o
2. The angles of a triangle are in the ratio 2 : 3 : 5, find the three angles.
3. Prove that the sum of the four angles of a quadrilateral is 360o.
Notes
Fig. 10.43
5. Prove that if one angle of a triangle is equal to the sum of the other two angles,
then it is a right triangle.
6. In Fig. 10.44, ABC is triangle such that ∠ABC = ∠ACB. Find the angles of the
triangle.
Fig. 10.44
10.4 LOCUS
During the game of cricket, when a player hits the ball, it describes a path, before being
caught or touching the ground.
Fig. 10.44
The path described is called Locus.
A figure in geometry is a result of the path traced by a point (or a very small particle)
moving under certain conditions.
For example:
(1) Given two parallel lines l and m, also a point P between them equidistant from both
the lines.
.
Notes
Fig. 10.45
If the particle moves so that it is equidistant from both the lines, what will be its path?
.. . .
Fig. 10.46
The path traced by P will be a line parallel to both the lines and exactly in the middle of
them as in Fig. 10.46.
.
(2) Given a fixed point O and a point P at a fixed distance d.
.
Fig. 10.47
If the point P moves in a plane so that it is always at a constant distance d from the
fixed point O, what will be its path?
Fig. 10.48
The path of the moving point P will be a circle as shown in Fig. 10.48.
(3) Place a small piece of chalk stick or a pebble on top of a table. Strike it hard with a
pencil or a stick so that it leaves the table with a certain speed and observe its path
after it leaves the table.
Notes
Fig. 10.49
The path traced by the pebble will be a curve (part of what is known as a parabola) as
shown in Fig. 10.49.
Thus, locus of a point moving under certain conditions is the path or the geometrical figure,
every point of which satisfies the given conditon(s).
.
P
. .
A B
Fig. 10.50
We have to find the locus of a point P such that PA = PB.
Joint AB. Mark the mind point of AB as M. Clearly, M is a point which is equidistant from
A and B. Mark another point P using compasses such that PA = PB. Join PM and extend
it on both sides. Using a pair of divider or a scale, it can easily be verified that every point
on PM is equidistant from the points A and B. Also, if we take any other point Q not lying
on line PM, then QA ≠ QB.
Also ∠AMP = ∠BMP = 90o
That is, PM is the perpendicular bisector of AB.
Fig. 10.51
D
Fig. 10.52
We have to find the locus of a point P which is equidistant from both AB and CD.
Draw bisectors of ∠BOD and ∠BOC.
Fig. 10.53
If we take any point P on any bisector l or m, we will find perpendicular distances PL and
PM of P from the lines AB and CD are equal.
that is, PL = PM
If we take any other point, say Q, not lying on any bisector l or m, then QL will not be
equal to QM.
Thus, we may conclude :
The locus of a point equidistant from two intersecting lines is the pair of
lines, bisecting the angles formed by the given lines.
Fig. 10.54
In a similar way find the other bisector by folding again and getting crease 2. Any point on
this crease 2 is also equidistant from both the lines.
Example 10.5 : Find the locus of the centre of a circle passing through two given points.
Solution : Let the two given points be A and B. We have to find the position or positions
of centre O of a circle passing through A and B.
.
O
. .
A B
Fig. 10.55
Point O must be equidistant from both the points A and B. As we have already learnt, the
locus of the point O will be the perpendicular bisector of AB.
Fig. 10.56
LET US SUM UP
• A line extends to inifinity on both sides and a line segment is only a part of it
between two points.
• Two distinct lines in a plane may either be intersecting or parallel.
• If three or more lines intersect in one point only then they are called cocurrent lines.
• Two rays starting from a common point form an angle.
• A pair of angles, whose sum is 900 is called a pair of complementary angles.
• A pair of angles whose sum is 1800 is called a pair of supplementary angles.
• If a ray stands on a line then the sum of the two adjacent angles, so formed is 1800
• If two lines intersect each other the pairs of vertically opposite angles are equal
• When a transversal intersects two parallel lines, then
(i) corresponding angles in a pair are equal.
(ii) alternate angles are equal.
(iii) interior angles on the same side of the transversal are supplementary.
• The sum of the angles of a triangle is 1800
• An exterior angle of a triangle to equal to the sum of the two interior opposite angles
• Locus of a point equidistant from two given points is the perpendicular bisector
of the line segment joing the points.
Notes
TERMINAL EXERCISE
1. In Fig. 10.57, if x = 42, then determine (a) y (b) ∠AOD
Fig. 10.57
2.
Fig. 10.58
In the above figure p, q and r are parallel lines intersected by a transversal l at A, B
and C respectively. Find ∠1 and ∠2.
3. The sum of two angles of a triangle is equal to its third angle. Find the third angle.
What type of triangle is it?
4.
Fig. 10.59
In Fig. 10.59, sides of Δ ABC have been produced as shown. Find the angles of the
triangle.
Notes
Fig. 10.60
In Fig. 10.60, sides AB, BC and CA of the triangle ABC have been produced as
shown. Show that the sum of the exterior angles so formed is 360o.
6.
Fig. 10.61
In Fig. 10.61 ABC is a triangle in which bisectors of ∠B and ∠C meet at O. Show
that ∠BOC = 125o.
7.
Fig. 10.62
In Fig. 10.62 above, find the sum of the angles, ∠A , ∠F , ∠C , ∠D , ∠B and ∠E.
8.
Fig. 10.63
Fig. 10.64
In Fig. 10.64 above, in Δ PQR, PT is bisector of ∠P and QR is produced to S.
Show that ∠PQR + ∠PRS = 2 ∠PTR.
10. Prove that the sum of the (interior) angles of a pentagon is 5400.
11. Find the locus of a point equidistant from two parallel lines l and m at a distance of 5
cm from each other.
12. Find the locus of a point equidistant from points A and B and also equidistant from
rays AB and AC of Fig. 10.65.
B .
A . .
C
Fig. 10.65
Fig. 10.65
3. Possible locations will be four points two points P and Q on the bisector of ∠AOC
and two points R and S on the bisector of ∠BOC.
Fig. 10.66
4. Two on either side of AB and lines parallel to AB at a distance of 5 cm from AB.
Notes
11
CONGRUENCE OF TRIANGLES
You might have observed that leaves of different trees have different shapes, but leaves of
the same tree have almost the same shape. Although they may differ in size. The geometrical
figures which have same shape and same size are called congruent figures and the property
is called congruency.
In this lesson you will study congruence of two triangles, some relations between their
sides and angles in details.
OBJECTIVES
After studying this lesson, you will be able to
• verify and explain whether two given figures are congruent or not.
• state the criteria for congruency of two triangles and apply them in solving
problems.
• prove that angles opposite to equal sides of a triangle are equal.
• prove that sides opposite to equal angles of a triangle are equal.
• prove that if two sides of triangle are unequal, then the longer side has the
greater angle opposite to it.
• state and verify inequalities in a triangle involving sides and angles.
• solve problems based on the above results.
Fig. 11.1
(ii) Objcts, which have same shpaes but different sizes as shown in Fig. 11.2
Fig. 11.2
(iii) Two one-rupee coins.
Fig. 11.3
(iv) Two postage stamps on post cards
Fig. 11.4
Notes
Fig. 11.5
We will deal with the figures which have same shapes and same sizes.
Two figures, which have the same shape and same size are called congruent
figures and this property is called congruence.
11.1.1. Activity
Take a sheet of paper, fold it in the middle and keep a carbon (paper) between the two
folds. Now draw a figure of a leaf or a flower or any object which you like, on the upper
part of the sheet. You will get a carbon copy of it on the sheet below.
The figure you drew and its carbon copy are of the same shape and same size. Thus, these
are congruent figures. Observe a butterfly folding its two wings. These appear to be one.
Congruent figures, when palced one over another, exactly coincide with one another or
cover each other. In other words, two figures will be congruent, if parts of one figure are
equal to the corresponding parts of the other. For example :
(1) Two line - segments are congruent, when they are of equal length.
A B C D
Fig. 11.6
(2) Two squares are congruent if their sides are equal.
Fig. 11.7
Notes
Fig. 11.8
Fig. 11.9
PQ = XY, PR = XZ, QR = YZ
∠P = ∠X, ∠Q = ∠Y and ∠R = ∠Z
Thus we can say
Δ PQR is congruent to Δ XYZ and we write
Δ PQR ≅ Δ XYZ
Relation of congruence between two triangles is always written with corresponding or
matching parts in proper order.
Here Δ PQR ≅ Δ XYZ
also means P corresponds to X, Q corresponds to Y and R corresponds to Z.
B C
Fig. 11.10
Construct another triangle PQR such that QR = BC, ∠Q = ∠B and PQ = AB.
(See Fig. 11.11)
Q R
Fig. 11.11
If we trace or cut out triangle ABC and place it over triangle PQR. we will observe that
one covers the other exactly. Thus, we may say that they are congruent.
Alternatively we can also measure the remaining parts, and observe that
Criterion 1 : If any two sides and the included angle of one triangle are
equal to the corresponding sides and the included angle of the other triangle,
the two triangles are congruent.
Fig. 11.12
Construct another Δ PQR such that, QR = BC, ∠Q = ∠B and ∠R = ∠C. (See Fig. 11.13)
Fig. 11.13
By superimposition or by measuring the remaining corresponding parts, we observe that
∠P = ∠A, PQ = AB and PR = AC establishing that Δ PQR ≅ Δ ABC, which again means
that equality of the three corresponding parts (two angles and the inluded side) of two
triangles results in congruent triangles.
We also know that the sum of the three angles of a triangle is 180o, as such if two angles of
one triangle are equal to the corresponding angles of another triangle, then the third angles
will also be equal. Thus instead of included side we may have any pair of corresponding
sides equal. Thus we have
Criterion 2 : If any two angles and one side of a triangle are equal to
corresponding angles and the side of another triangle, then the two triangles
are congruent.
Notes This criterion is referred to as ASA or AAS (Angle Side Angle or Angle Angle Side)
11.3.1 Activity
In order to explore another criterion we again take a triangle ABC (See Fig. 11.14)
Fig. 11.14
Now take three thin sticks equal in lengths to sides AB, BC and CA of Δ ABC. Place them
in any order to form Δ PQR or Δ P′Q′R′ near the Δ ABC (Fig. 11.15)
Q Q′
P′
R
Fig. 11.15 R′
Fig. 11.16
Solution :
In Δ PAX and Δ QAY,
∠XPA = ∠YQA (Each is 90o)
∠PAX = ∠QAY (Vertically opposite angles)
1
Prove that BD = AC.
2
Fig. 11.17
Solution : Produce BD to E such that BD = DE. Join CE
Fig.. 11.18
In Δ ADB and Δ CDE,
AD = CD (D being and point of AC)
DB = DE (By construction)
and ∠ADB = ∠CDE (Vertically opposite angles)
∴ Δ ADB ≅ Δ CDE (i)
∴ AB = EC
Also ∠DAB = ∠DCE
But they make a pair of alternate angles
∴ AB is parallel to EC
∴ ∠ABC + ∠ECB = 1800 (Pair of interior angles)
1
But BD = EB
2
1
∴ BD = AC
2
Fig. 11.19
(a) RHS (b) ASA
(c) SAS (d) SSS
2. In Fig. 11.20, Δ ABC ≅ Δ PQR. This congruence may also be written as
Fig. 11.20
Fig. 11.21
6. In Fig. 11.22, AB is parallel to CD. If O is the mid-point of BC, show that it is also
the mid-point of AD.
Fig. 11.22
Notes
Fig. 11.23
8. From Fig. 11.24, show that the triangles are congruent and make pairs of equal
angles.
Fig. 11.24
∴ ∠B = ∠A ...(ii)
From (i) and (ii),
∠A = ∠B = ∠C
Hence the result.
Example 11.6 : ABC is an isosceles triangle in which AB = AC
(Fig. 11.28), If BD ⊥ AC and CE ⊥ AB, proe that BD = CE.
1
Solution : BE = AB
2
1
and CD = AC
2
Fig. 11.29
∴ BE = CD ...(i)
In Δ BEC and Δ CDB,
BE = CD [By (i)]
BC = CB (Common)
and ∠ΕBC = ∠DCB ( Q AB = AC)
∴ Δ BEC ≅ Δ CDB (SAS)
Hence, CE = BD (c.p.c.t)
Example 11.8 : In Δ ABC (Fig. 11.30) AB = AC and
∠DAC = 124o; find the angles of the triangle.
Solution ∠BAC = 180o – 124o = 56o
∠B = ∠C
(Angles opposite to equal sides of a triangle)
Fig. 11.30
1240
∠B = ∠C = = 620
2
Notes
Hence ∠A = 56o, ∠B = 62o, and ∠C = 62o
Fig. 11.31
2. Prove that ΔABC is an isosceles triangle, if the altitude AD bisects the base BC
(Fig. 11.32).
Fig. 11.32
3. If the line l in Fig. 11.33 is parallel to the base BC of the isosceles ΔABC, find the
angles of the triangle.
Fig. 11.33
4. ΔABC is an isosceles triangle such that AB = AC. Side BA is produced to a point D
such that AB = AD. Prove that ∠ΒCD is a right angle.
Notes
Fig. 11.34
5. In Fig. 11.35. D is the mid point of BC and perpendiculars DF and DE to sides AB
and AC respectively are equal in length. Prove that ΔABC is an isosceles triangle.
Fig. 11.35
6. In Fig. 11.36, PQ = PR, QS and RT are the angle bisectors of ∠Q and ∠R
respectively. Prove that QS = RT.
Fig. 11.36
7. ΔPQR and ΔSQR are isosceles triangles on the same base QR (Fig. 11.37). Prove
that ∠PQS = ∠PRS.
Fig. 11.37
8. In ΔABC, AB = AC (Fig. 11.38). P is a point in the interior of the triangle such that
∠ΑΒP = ∠ΑCP . Prove that AP bisects ∠BAC.
Notes
Fig. 11.38
Fig. 11.39
In Fig. 11.39, triangle ABC has side AB longer than the side AC. Measure ∠Β and
∠C. You will find that these angles are not equal and ∠C is greater than ∠B. If you
repeat this experiment, you will always find that this observation is true. This can be
proved easily, as follows.
11.5.1 Theorem
If two sides of a triangle are unequal, then the longer side has the greater angle oppo-
site to it.
Given. A triangle ABC in which AB > AC.
To prove. ∠ΑCB > ∠ΑΒC
Construction. Make a point D on the side AB such that
AD = AC and join DC.
Proof: In ΔACD,
AD = AC Fig. 11.40
Observe that in a triangle if one angle is right or an obtuse then the side opposite to that
angle is the longest.
You have already learnt the relationship among the three angles of a triangle i.e., the sum of
the three angles of a triangle is 180o. We shall now study whether the three sides of a
triangle are related in some way.
Draw a triangle ABC.
Fig. 11.42
Sum of any two sides of a triangle is greater than the third side.
ACTIVITY
Fix three nails P, Q and R on a wooden board or any surface.
Fig. 11.43
Take a piece of thread equal in length to QR and another piece of thread equal in
length (QP + PR). Compare the two lengths, you will find that the length correspond-
ing to (QP + PR) > the length corresponding to QR confirming the above property.
in (c) 3.5 + 2.5 > 5.2, 3.5 + 5.2 > 2.5 and 2.5 + 5.2 > 3.5 and
Ans. (c)
Notes
Fig. 11.46
Notes
Fig. 11.47
5. Prove that the sum of the three sides of triangle is greater than the sum of its three
medians.
(Use Example 11.10)
6. In Fig. 11.48, if AB = AD then prove that BC > CD.
[Hint : ∠ADB = ∠ABD].
Fig. 11.48
7. In Fig. 11.49, AB is parallel to CD. If ∠A > ∠B then prove that BC > AD.
Fig. 11.49
LET US SUM UP
• Figures which have the same shape and same size are called congruent figures.
• Two congruent figures, when placed one over the other completely cover each
other. All parts of one figure are equal to the corresponding parts of the other figure.
TERMINAL EXERCISE
1. Two lines AB and CD bisect each other at O. Prove that CA = BD (Fig. 11.50)
Fig. 11.50
2. In a ΔABC, if the median AD is perpendicular to the base BC then prove that the
triangle is an isosceles triangle.
3. In Fig. 11.51, ΔABC and ΔCDE are such that BC = CE and AB = DE. If ∠B = 60o,
∠ACE = 30o and ∠D = 90o, then prove that the two triangles are congruent.
Fig. 11.51
Notes
Fig. 11.52
5. In a right triangle, one of the acute angles is 30o. Prove that the hypotenuse is twice
the side opposite to the angle of 30o.
6. Line segments AB and CD intersect each other at O such that O is the midpoint of
AB. If AC is parallel to DB then prove that O is also the mid piont of CD.
7. In Fig. 11.53, AB is the longest side and DC is the shortest side of a quadrilateral
ABCD. Prove that ∠C > ∠A and ∠D> ∠B. [Hint : Join AC and BD].
Fig. 11.53
8. ABC is an isosceles triangle in which AB = AC and AD is the altitude from A to the
base BC. Prove that BD = DC.
Fig. 11.54
Notes
Fig. 11.55
Notes
12
CONCURRENT LINES
You have already learnt about concurrent lines, in the lesson on lines and angles. You have
also studied about triangles and some special lines, i.e., medians, right bisectors of sides,
angle bisectors and altitudes, which can be drawn in a triangle. In this lesson, we shall
study the concurrency property of these lines, which are quite useful.
OBJECTIVES
After studying this lesson, you will be able to
• define the terms concurrent lines, median, altitude, angle bisector and
perpendicular bisector of a side of a triangle.
• Verify the concurrnence of medians, altitudes, perpendicular bisectors of sides
and angle bisectors of a triangle.
In triangle ABC, the line AD bisects ∠A of the triangle. (See Fig. 12.4)
C
A B
Fig. 12.4
C C
D E D
E
I I
A B A B
F
C
D
E D
E
I
B
A A F B
F
Fig. 12.7
Thus we conclude the following:
Angle bisectors of a triangle pass through the same point, that is they are
concurrent
The point of concurrency I is called the ‘Incentre’ of the triangle.
Can you reason out, why the name incentre for this point?
Recall that the locus of a point equidistant from two intersecting lines is the pair of angle
bisectors of the angles formed by the lines. Since I is a point on the bisector of ∠BAC, it
must be equidistant from AB and AC. Also I is a point on angle bisector of ∠ABC, (See
A N B
F
Fig. 12.8
Thus, we have IL = IM = IN (Fig. 12.8). Taking I as the centre and IL as the radius, we
can draw a circle touching all the three sides of the triangle called ‘Incircle’ of the triangle.
I being the centre of the incircle is called the Incentre and IL, the radius of the incircle is
called the inradius of the triangle.
Note: The incentre always lies in the interior of the triangle.
ABC is a triangle, line DP bisects side BC at right angle. A line which bisects a side of a
triangle at right angle is called the perpendicular bisector of the side. Since a triangle has
three sides, so we can draw three perpendicular bisectors in a triangle. DP is one of the
three perpendicular bisectors of ΔABC (Fig. 12.9). We draw the second perpendicular
bisector EQ, intersecting DP at O (Fig. 12.10). Now if we also draw the third perpendicular
bisector FR, then we observe that it also passes through the point O (Fig. 12.11). In other
words, we can say that the three perpendicular bisectors of the sides are concurrent at O.
A A
P P
E
O
B C B C
D Q D
A
P
E
F
O
B C
Q D
R
Fig. 12.11
B D C B D C
(b)
(a)
Fig. 12.12
Can you reason out: why the name circumcentre for this point?
Recall that the locus of a point equidistant from two given points is the perpendicular
bisector of the line joining the two points. Since O lies on the perpendicular bisector of
BC, so it must be equidistant from both the point B and C i.e., BO = CO (Fig. 12.13).
F
E
O
B C
D
Fig. 12.13
The point O also lies on the perpendicular bisector of AC, so it must be equidistant from
both A and C, that is, AO = CO. Thus, we have AO = BO = CO.
In ΔABC, the line AL is the perpendicular drawn from vertex A to the opposite side BC.
(Fig. 12.14).
A A
B L C L B C
Fig. 12.14
Perpendicular drawn from a vertex of a triangle to the oposite side is called its altitude.
How many altitudes can be drawn in a triangle? There are three vertices in a triangle, so
we can draw three of its altitudes. AL is one of these altitudes. Now we draw the second
altitude BM, which intersects the first altitude at a point H (see Fig. 12.15). We also draw
the third altitude CN and observe that it also passes through the point H (Fig. 12.16). This
shows that the three altitudes of the triangle pass through the same point.
A
A
M
N M
H H
B L C
B L C
Fig. 12.15 Fig. 12.16
A
Notes A
M
C M
N B
B
H C
H
In a triangle, the three altitudes pass through the same point, that is, they
are concurrent.
The point of concurrency is called the ‘Orthocentre’of the triangle.
In ΔABC, AD joins the vertex A to the mid point D of the opposite side BC (Fig. 12.19)
A A
B D C B C
D
(a) (b)
Fig. 12.19
F E E
G E F
F G
B C B C B D C
D D
Fig. 12.21
∴ ΔABD ≅ ΔACD
∴ BD = CD
⇒ AD is also a median
⇒ Also ∠ADB = ∠ADC = 90o
⇒ AD is an altitude B C
D
Since, BD = DC, Fig. 12.22
AD is perpendicular bisector of side BC.
Example 12.2: In an equilateral triangle, show that the three angle bisectors are also the
three perpendicular bisectors of sides, three altitudes and the three medians of the triangle.
Solution: Since AB = AC A
F E
G
Notes
B C
D
Fig. 12.24
3
∴ AD = a, as BC = a
2
2 3 3
⇒ AG = circumradius in this case = × a= a
3 2 3
1 3 3
and GD = inradius in this case = × a= a.
3 2 6
A
G
B D E F C
Fig. 12.25
2. In an equilateral triangle show that the incentre, the circumcentre, the orthocentre and
the centroid are the same point.
3. In an equilateral ΔABC (Fig. 12.26). G is the centroid of the triangle. If AG is 4.8 cm,
find AD and BE.
Notes F E
B D C
Fig. 12.26
4. If H is the orthocentre of ΔABC, then show that A is the orthocentre of the ΔHBC.
5. Choose the correct answers out of the given alternatives in the following questions:
(i) In a plane, the point equidistant from vertices of a triangle is called its
(a) centroid (b) incentre
(c) circumcentre (d) orthocentre
(ii) In the plane of a triangle, the point equidistant from the sides of the triangle is
called its
(a) centroid (b) incentre
(c) circumcentre (d) orthocentre
LET US SUM UP
• Three or more lines in a plane which intersect each other in exactly one point are called
concurrent lines.
• A line which bisects an angle of a triangle is called an angle bisector of the triangle.
• A line which bisects a side of a triangle at right angle is called the perpendicular bisector
of the side of the triangle.
• A line drawn perpendicular from a vertex of a triangle to its opposite side is called an
altitude of the triangle.
• A line which joins a vertex of a triangle to the mid-point of the opposite side is called
a median of the triangle.
• In a triangle
(i) angle bisectors are concurrent and the point of concurrency is called incentre.
TERMINAL EXERCISE
1. In the given Fig. 12.27, D, E and F are the mid points of the sides of ΔABC. Show
3
that BE + CF > BC .
2
A
E
F
G
C
D
B
Fig. 12.27
2. ABC is an isoceles triangle such that AB = AC and D is the midpoint of BC. Show that
the centroid, the incentre, the circumcentre and the orthocentre, all lie on AD.
A
B D C
Fig. 12.28
3. ABC is an isoceles triangle such that AB = AC = 17 cm and base BC = 16 cm. If G
is the centroid of ΔABC, find AG.
4. ABC is an equilateral triangle of side 12 cm. If G be its centroid, find AG.
3. AG = 10 cm
4. AG = 4 3 cm
Notes
13
QUADRILATERALS
If you look around, you will find many objects bounded by four line-segments. Any surface
of a book, window door, some parts of window-grill, slice of bread, the floor of your
room are all examples of a closed figure bounded by four line-segments. Such a figure is
called a quadrilateral.
The word quadrilateral has its origin from the two words “quadric” meaining four and
“lateral” meaning sides. Thus, a quadrilateral is that geometrical figure which has four
sides, enclosing a part of the plane.
In this lesson, we shall study about terms and concepts related to quadrilateral with their
properties.
OBJECTIVES
After studying this lesson, you will be able to
• describe various types of quadrilaterals viz. trapeziums, parallelograms,
rectangles, rhombuses and squares;
• verify properties of different types of quadrilaterals;
• verify that in a triangle the line segment joining the mid-points of any two sides
is parallel to the third side and is half of it;
• verify that the line drawn through the mid-point of a side of a triangle parallel to
another side bisects the third side;
• verify that if there are three or more parallel lines and the intercepts made by
them on a transversal are equal, the corresponding intercepts on any other
transversal are also equal;
• verify that a diagonal of a parallelogram divides it into two triangles of equal
area;
• solve problem based on starred results and direct numerical problems based on
unstarred results given in the curriculum;
• prove that parallelograms on the same or equal bases and between the same
parallels are equal in area;
• verify that triangles on the same or equal bases and between the same parallels
Notes are equal in area and its converse.
13.1 QUADRILATERAL
Recall that if A, B, C and D are four points in a plane such that no three of them are
collinear and the line segments AB, BC, CD and DA do not intersect except at their end
points, then the closed figure made up of these four line segments is called a quadrilateral
with vertices A, B, C and D. A quadrilateral with vertices A, B, C and D is generally
denoted by quad. ABCD. In Fig. 13.1 (i) and (ii), both the quadrilaterals can be named as
quad. ABCD or simply ABCD.
In quadrilateral ABCD,
(i) (ii)
Fig. 13.1
(i) (ii)
Fig. 13.2
What is the sum of these angles You will find that ∠l + ∠2 + ∠3 + ∠4 = 360°.
i.e. sum of interior angles of a quadrilateral equals 360°.
Also what is the sum of exterior angles of the quadrilateral ABCD?
You will again find that ∠5 + ∠6 + ∠7 + ∠8 = 360°
i.e., sum of exterior angles of a quadrilateral is also 360°.
Rectangle Rhombus
Square
Fig. 13.3
Let us describe them one by one.
1. Trapezium
A quadrilateral which has only one pair of opposite sides parallel is called a trapezium. In
Fig. 13.4 [(i) and (ii)] ABCD and PQRS are trapeziums with AB || DC and PQ || SR
respectively.
Notes
(i) (ii)
Fig. 13.4
2. Kite
A quadrilateral, which has two pairs of equal sides next to each other, is called a kite.
Fig. 13.5 [(i) and (ii)] ABCD and PQRS are kites with adjacent sides AB and AD, BC
and CD in (i) PQ and PS, QR and RS in (ii) being equal.
P
A
B D Q S
(i) (ii)
R
C Fig. 13.5
3. Parallelogram
A quadrilateral which has both pairs of opposite sides parallel, is called a parallelogram. In
Fig. 13.6 [(i) and (ii)] ABCD and PQRS are parallelograms with AB||DC, AD||BC and
PQ||SR, SP||RQ. These are denoted by ||gm ABCD (Parallelogram ABCD) and ||gm PQRS
(Parallelogram PQRS).
(i) (ii)
Fig. 13.6
Fig. 13.8
6. Square
A square is a rectangle, with a pair of adjacent sides equal.
In other words, a parallelogram having all sides equal and each angle a right angle is called
a square.
Fig. 13.9
Fig. 13.10
3. In a quadrilateral, all its angles are equal. Find the measure of each angle.
4. The angles of a quadrilateral are in the ratio 5:7:7: 11. Find the measure of each angle.
5. If a pair of opposite angles of a quadrilateral are supplementary, what can you say
about the other pair of angles?
1. Properties of a Parallelogram
We have learnt that a parallelogram is a quadrilateral with both pairs of opposite sides
parallel. Now let us establish some relationship between sides, angles and diagonals of a
parallelogram.
Draw a pair of parallel lines l and m as shown in Fig. l3.12. Draw another pair of parallel
lines p and q such that they intersect l and m. You observe that a parallelogram ABCD is
formed. Join AC and BD. They intersect each other at O.
Notes
Fig. l3.12
Now measure the sides AB, BC, CD and DA. What do you find?
Draw another parallelogram and repeat the activity. You will find that
Notes
Fig. 13.13
Where does the point C fall?
Where does the point A fall?
You will observe that ΔADC will coincide with ΔABC. In other words ΔABC ≅ ΔADC.
Also AB = CD and BC = AD and ∠B = ∠D.
You may repeat this activity by taking some other parallelograms, you will always get the
same results as verified earlier, thus, proving the above two properties of the parallelogram.
Now you can prove the third property of the parallelogram, i.e., the diagonals of a
parallelogram bisect each other.
Again take a thin cardboard. Draw any parallelogram PQRS on it. Draw its diagonals
PR and QS which intersect each other at O as shown in Fig. l3.14. Now cut the
parallelogram PQRS.
Fig. 13.14
Also cut ΔPOQ and ΔROS.
Now place ΔROS and ΔPOQ in such a way that the vertex R coincides with the vertex P
and RO coincides with the side PO.
Where does the point S fall?
Where does the side OS fall?
Is ΔROS ≅ ΔPOQ? Yes, it is.
Notes You may also verify this property by taking another pair of triangles i.e. ΔPOS and ΔROQ
You will again arrive at the same result.
You may also verify the following properties which are the converse of the properties of a
parallelogram verified earlier.
2. Properties of a Rhombus
In the previous section we have defined a rhombus. We know that a rhombus is a
parallelogram in which a pair of adjacent sides is equal. In Fig. 13.15, ABCD is a
rhombus.
Fig. 13.15
Thus, ABCD is a parallelogram with AB = BC. Since every rhombus is a parallelogram,
therefore all the properties of a parallelogram are also true for rhombus, i.e.
3. Properties of a Rectangle
We know that a rectangle is a parallelogram one of whose angles is a right angle. Can you
say whether a rectangle possesses all the properties of a parallelogram or not?
Yes it possesses. Let us study some more properties of a rectangle.
Draw a parallelogram ABCD in which ∠ B = 90°.
Join AC and BD as shown in the Fig. 13.16
Fig. 13.16
Measure ∠BAD, ∠BCD and ∠ADC, what do you find?
What are the measures of these angles?
The measure of each angle is 90°. Thus, we can conclude that
∠A = ∠B = ∠C = ∠D = 90o
i.e., each angle of a rectangle measures 90°. Now measure the diagonals AC and BD. Do
you find that AC = BD.
Now, measure AO, OC, BO and OD.
Notes
You will find that AO = OC and BO = OD.
Draw some more rectangles of different dimensions. Label them again by ABCD. Join AC
and BD in each case. Let them intersect each other at O. Also measure AO, OC and BO,
OD for each rectangle. In each case you will find that
The diagonals of a rectangle are equal and they bisect each other. Thus, we have the
following properties of a rectangle;
4. Properties of a Square
You know that a square is a rectangle, with a pair of adjacent sides equal. Now, can you
conclude from definition of a square that a square is a rectangle and possesses all the
properties of a rectangle? Yes it is. Let us now study some more properties of a square.
Draw a square ABCD as shown in Fig. 13.17.
Fig 13.17
(i) AB = DC, AD = BC
(ii) ∠A = ∠B = ∠C = ∠D = 90o
(iii) AC = BD and AO = OC, BO = OD
x = 20
∴ The two measures of angles are 80o and 100o.
Notes i.e. ∠A = 80o and ∠B = 100o
Since ∠A = ∠C ⇒ ∠C = 100o
Also, ∠B = ∠D ⇒ ∠D =100o
Hence, the measures of angles of the
rhombus are 80o, 100o, 80o and 100o. Fig 13.19
Example 13.5: One of the diagonals of a rhombus is equal to one of its sides. Find the
angles of the rhombus.
Solution: Let in rhombus, ABCD,
AB = AD = BD
∴ ΔABD is an equilateral triangle.
∴ ∠DAB = ∠1 = ∠2 = 60o ....(1)
Similarly ∠BCD = ∠3 = ∠4 = 60o ....(2)
Also from (1) and (2)
Fig 13.20
∠ABC = ∠B = ∠1 + ∠3 = 60o + 60o = 120o
∠ADC = ∠D = ∠2 + ∠4 = 60o + 60o = 120o
Hence, ∠A = 60o, ∠B = 120o, ∠C = 60o and ∠D = 120o
Example 13.6: The diagonals of a rhombus ABCD intersect at O. If ∠ADC = 120o and
OD = 6 cm, find
(a) ∠OAD
(b) side AB
(c) perimeter of the rhombus ABCD
Solution: (a) Given that
∠ADC = 120o
i.e. ∠ADO + ∠ODC = 120o Fig 13.21
But ∠ADO = ∠ODC (ΔAOD ≅ ΔCOD)
∴ 2∠ADO = 120o
i.e. ∠ADO = 60o ...(i)
6. In Fig. 13.22, the diagonals of a rectangle PQRS intersect each other at O. If ∠ROQ
= 40o, find the measure of ∠OPS.
Notes
Fig 13.22
1
Of course, it is DE = BC
2
1
You will always find that DE = BC and DE || BC.
2
1
i.e. AE = AC
2
⎛1 ⎞
= ⎜ × 8 ⎟ cm [Q AC = 8 cm]
⎝2 ⎠
= 4 cm Fig 13.25
Hence, AE = 4 cm
Example 13.8: In Fig. 13.26, ABCD is a
trapezium in which AD and BC are its non-parallel
sides and E is the mid-point of AD. EF || AB.
Show that F is the mid-point of BC.
Solution: Since EG || AB and E is the mid-point
of AD (considering ΔABD)
∴ G is the mid point of DB Fig 13.26
Notes Example 13.9: ABC is a triangle, in which P, Q and R are mid-points of the sides AB, BC
and CA respectively. If AB = 8 cm, BC = 7 cm and CA = 6 cm, find the sides of the
triangle PQR.
Solution: P is the mid-point of AB and R the mid-point of AC.
1
∴ PR || BC and PR = BC
2
1
= × 7 cm [Q BC = cm]
2
= 3.5 cm
1
Similarly, PQ = AC
2
1
= × 6 cm [Q AC = 6 cm]
2
Fig 13.27
= 3 cm
1
and QR = AB
2
1
= × 8 cm [Q AB = 8 cm]
2
= 4 cm
Hence, the sides of ΔPQR are PQ = 3 cm, QR = 4 cm and PR = 3.5 cm.
Fig. 13.28
344 Mathematics Secondary Course
Quadrilaterals MODULE - 3
Geometry
2. In Fig. 13.29, D and E are the mid-points of the sides AB and AC respectively of a
ΔABC. If BC = 10 cm; find DE.
Notes
Fig. 13.29
3. In Fig. 13.30, AD is a median of the ΔABC and E is the mid-point of AD, BE is
produced to meet AC at F. DG || EF, meets AC at G. If AC = 9 cm, find AF.
[Hint: First consider ΔADG and next consider ΔCBF]
Fig. 13.30
4. In Fig. 13.31, A and C divide the side PQ of ΔPQR into three equal parts, AB||CD||QR.
Prove that B and D also divide PR into three equal parts.
Fig. 13.31
Notes
Fig. 13.32
Fig. 13.33
The intercepts made by parallel lines on a transversal have some special properties which
we shall study here.
Let l and m be two parallel lines and XY be an intercept made on the transversal “n”. If
there are three parallel lines and they are intersected by a transversal, there will be two
intercepts AB and BC as shown in Fig. 13.34 (ii).
(i) (ii)
Fig. 13.34
Fig. 13.35
Also, measure LM, MN and NX. Do you find that they are also equal? Yes, they are.
Repeat this experiment by taking another set of two or more equidistant parallel lines and
measure their intercepts as done earlier. You will find in each case that the intercepts made
are equal.
Thus, we conclude the following:
If there are three or more parallel lines and the intercepts made by them on
a transversal are equal, the corresponding intercepts made on any other
transversal are also equal.
AB = BC and LM = MN.
Example 13.11: In Fig. 13.37, l || m || n and PQ = QR. If XZ = 20 cm, find YZ.
Solution: We have PQ = QR
∴ By intercept theorem,
Notes XY = YZ
Also XZ = XY + YZ
= YZ + YZ
∴ 20 = 2YZ ⇒ YZ = 10 cm
Hence, YZ = 10 cm
Fig. 13.37
Fig. 13.38
2. From Fig. 13.39, when can you say that AB = BC and XY = YZ?
Fig. 13.39
Notes
Fig. 13.40
Fig. 13.41
1
Now, Area of ΔADC = DC × PD ....(i)
2
1
Area of ΔACB = AB × QC ....(ii)
2
As AB = DC and PD = QC
∴ Area (ΔADC) = Area (ΔACB)
Thus, we conclude the following:
Theorm: Parallelogrm on the same base (or equal bases) and between the
same parallels are equal in area.
Fig. 13.43
Here BC = QR
and Area (ΔABC) = Area (ΔDBC) = Area (ΔPQR) [Given] ..(i)
Draw perpendiculars DE and PS from D and P to the line m meeting it in E and S
respectively.
1
Now Area (ΔABC) = BC × DE
2
1
Area (ΔDBC) = BC × DE ...(ii)
2
1
and Area (ΔPQR) = QR × PS
2
Also, BC = QR (given) ...(iii)
From (i), (ii) and (iii), we get
1 1
BC × DE = QR × PS
2 2
Notes 1 1
or BC × DE = BC × PS
2 2
∴ DE = PS
i.e., Altitudes of ΔABC, ΔDBC and ΔPQR are equal in length.
Thus, we conclude the following:
Triangles on the same or equal bases, having equal areas have their
corresponding altitudes equal.
40
∴ Altitude of ||gm BCEF = cm or 5 cm. Fig. 13.44
8
Example 13.13: In Fig. 13.45, the area of ΔABC is given to be 18 cm2. If the altitude DL
equals 4.5 cm, find the base of the ΔBCD.
Solution: Area (ΔBCD) = Area (ΔABC) = 18 cm2
Let the base of ΔBCD be x cm
1
∴ Area of ΔBCD = x × DL
2
⎛1 ⎞
= ⎜ x × 4.5 ⎟ cm2
⎝2 ⎠
⎛9 ⎞
or 18 = ⎜ x ⎟ Fig. 13.45
⎝4 ⎠
⎛ 4⎞
∴ x = ⎜18 × ⎟ cm = 8 cm.
⎝ 9⎠
Fig. 13.46
gm gm
Solution: Area (|| ABCD) = Area (|| ACED)
The diagonal AC divides the ||gm ABCD into two triangles of equal area.
1
∴ Area (ΔBCD) = Area (||gm ABCD)
2
∴ Area (||gm ABCD) = Area (||gm ACED) = 2 × 12 cm2
= 24 cm2
∴ Area of Trapezium ABED
= Area (ΔABC) + Area (||gm ACED)
= (12 + 24) cm2
= 36 cm2
Fig. 13.47
LET US SUM UP
Notes • A quadrilateral is a four sided closed figure, enclosing some region of the plane.
• The sum of the interior or exterior angles of a quadrilateral is equal to 360o each.
• A quadrilateral is a trapezium if its only one pair of opposite sides is parallel.
• A quadrilateral is a parallelogrm if both pairs of sides are parallel.
• In a parallelogram:
(i) opposite sides and angles are equal.
(ii) diagonals bisect each other.
• A parallelogram is a rhombus if its adjacent sides are equal.
• The diagonals of a rhombus bisect each other at right angle.
• A parallelogram is a rectangle if its one angle is 90o.
• The diagonals of a rectangle are equal.
• A rectangle is a square if its adjacent sides are equal.
• The diagonals of a square intersect at right angles.
• The diagonal of a parallelogram divides it into two triangles of equal area.
• Parallelogram on the same base (or equal bases) and between the same parallels are
equal in area.
• The triangles on the same base (or equal bases) and between the same parallels are
equal in area.
• Triangles on same base (or equal bases) having equal areas have their corrsponding
altitudes equal.
TERMINAL EXERCISE
1. Which of the following are trapeziums?
Fig. 13.48
Notes
Fig. 13.49
3. In Fig. 13.50, ABCD is a parallelogram with an area of 48 cm2. Find the area of (i)
shaded region (ii) unshaded region.
Fig. 13.49
4. Fill in the blanks in each of the following to make them true statements:
(i) A quadrilateral is a trapezium if ....
(ii) A quadrilateral is a parallelogram if ....
(iii) A rectangle is a square if ...
(iv) the diagonals of a quadrilateral bisect each other at right angle. If none of the
angles of the quadrilateral is a right angle, it is a ...
(v) The sum of the exterior angles of a quadrilateral is ...
5. If the angles of a quadrilateral are (x – 20)o, (x + 20)o, (x – 15)o and (x + 15)o, find x
and the angles of the quadrilateral.
6. The sum of the opposite angles of a parallelograms is 180o. What type of a parallelogram
is it?
7. The area of a ΔABD in Fig. 13.51 is 24 cm2. If DE = 6 cm, and AB || CD, BD || CE,
AE || BC, find
Notes
Fig. 13.51
(i) Altitude of the parallelogram BCED.
(ii) Area of the parallelogram BCED.
8. In Fig. 13.52, the area of parallelogram ABCD is 40 cm2. If EF = 8 cm, find the
altitude of ΔDCE.
Fig. 13.52
Notes
14
SIMILARITY OF TRIANGLES
Looking around you will see many objects which are of the same shape but of same or
different sizes. For examples, leaves of a tree have almost the same shape but same or
different sizes. Similarly, photographs of different sizes developed from the same negative
are of same shape but different sizes, the miniature model of a building and the building
itself are of same shape but different sizes. All those objects which have the same
shape but not necessarily the same size are called similar objects.
Let us examine the similarity of plane figures (Fig. 14.1):
(i) Two line-segments of the same length are congruent as well as similar and of different
lengths are similar but not congruent.
Notes
OBJECTIVES
After studying this lesson, you will be able to
• identify similar figures;
• distinguish between congurent and similar plane figures;
• prove that if a line is drawn parallel to one side of a triangle then the other two
sides are divided in the same ratio;
• state and use the criteria for similarity of triangles viz. AAA, SSS and SAS;
• verify and use unstarred results given in the curriculum based on similarity
experimentally;
• prove the Baudhayan/Pythagoras Theorem;
• apply these results in verifying experimentally (or proving logically) problems
based on similar triangles.
Notes
Fig. 14.2
In Fig. 14.2, the two pentagons seem to be of the same shape.
We can see that if ∠A = ∠A′, ∠B = ∠B′, ∠C = ∠C′, ∠D = ∠D′ and ∠E = E′ and
AB BC CD DE EA
= = = = . then the two pentagons are similar. Thus we say
A' B' B' C' C' D' D' E' E' A'
that
Any two polygons, with corresponding angles equal and corresponding sides
proportional, are similar.
Thus, two polygons are similar, if they satisfiy the following two conditions:
Fig. 14.3
AD AE
=
DB EC
Notes
We can easily verify this by measuring AD, DB, AE and
EC. You will find that
AD AE Fig. 14.4
=
DB EC
We state the converse of the above result as follows:
If a line divides any two sides of a triangle in the same ratio, the line is parallel to
third side of the triangle.
AD AE
Thus, in Fig 14.4, if DE divides side AB and AC of ΔABC such that = , then
DB EC
DE || BC.
We can verify this by measuring ∠ADE and ∠ABC and finding that
∠ADE = ∠ABC
These being corresponding angles, the line DE and BC are parallel.
We can verify the above two results by taking different triangles.
Let us solve some examples based on these.
Example 14.1: In Fig. 14.5, DE || BC. If AD = 3 cm, DB = 5 cm and AE = 6 cm, find
AC.
Solution: DE || BC (Given). Let EC = x
AD AE
∴ =
DB EC
3 6
∴ =
5 x
⇒ 3 x = 30
⇒ x = 10 Fig. 14.5
∴ EC = 10 cm
∴ AC = AE + EC = 16 cm
5
Example 14.2: In Fig. 14.6, AD = 4 cm, DB = 5 cm, AE = 4.5 cm and EC = 5 cm.
8
Is DE || BC? Given reasons for your answer.
AD 4
∴ =
Notes DB 5
AE 4.5 9 8 4
= = × =
Similarly, EC 45 2 45 5
8
Fig. 14.6
AD AE
∴ =
DB EC
∴ According to converse of Basic Proportionality Theorem
DE || BC
(i) (ii)
Fig. 14.7
2. In Fig. 14.8 [(i)], find whether DE || BC is parallel to BC or not? Give reasons for your
answer.
(i)
Fig. 14.8
BD AB
= (Fig. 14.9)
DC AC
We can easily verify this by measuring BD, DC, AB
and AC and finding the ratios. We will find that
Fig. 14.9
BD AB
=
DC AC
Repeating the same activity with other triangles, we may verify the result.
Let us solve some examples to illustrate this.
Example 14.3: The sides AB and AC of a triangle are of length 6 cm and 8 cm respectively.
The bisector AD of ∠A intersects the opposite side BC in D such that BD = 4.5 cm (Fig.
14.10). Find the length of segment CD.
Solution: According to the above result, we have
BD AB
=
DC AC
( Q AD is internal bisector of ∠A of ΔABC)
4.5 6
or =
x 8
⇒ 6x = 4.5 × 8
x=6 Fig. 14.10
BD 36 3
∴ = =
DC 48 4
Notes 3
⇒ 4BD = 3DC or BD = DC
4
BC = BD + DC = 28 cm
3
∴ DC + DC = 28
4
⎛ 4⎞ Fig. 14.11
∴ DC = ⎜ 28 × ⎟cm = 16 cm
⎝ 7⎠
∴ BD = 12 cm and DC = 16 cm
Fig. 14.12
2. In Fig. 14.13, PS is the bisector of ∠P of ΔPQR. The dimensions of some of the sides
are given in Fig. 14.13. Find x.
Fig. 14.13
Notes
Fig. 14.14
Fig. 14.15
AB BC CA
∠A = ∠D, ∠B = ∠E, ∠C = ∠F and = = .
DE EF FD
We shall show that in the case of triangles if either of the above two conditions is satisfied
then the other automatically holds.
Construct two Δ’s ABC and PQR in which ∠P = ∠A, ∠Q = ∠B and ∠R = ∠C as shown
in Fig. 14.16.
Fig. 14.16
Measure the sides AB, BC and CA of the ΔABC and also measure the sides PQ, QR and
RP of ΔPQR.
AB BC CA
Now find the ratio , and .
PQ QR RP
What do you find? You will find that all the three ratios are equal and therefore the triangles
are similar.
Try this with different triangles with equal corresponding angles. You will find the same
result.
If in two triangles, the corresponding angles are equal the triangles are similar
Notes
Q R
(i) (ii)
Fig. 14.17
Take a line AB = 3 cm and at A construct an angle of 60o. Cut off AC = 4.5 cm. Join BC.
Fig. 14.18
Now take PQ = 6 cm. At P, draw an angle of 60o and cut off PR = 9 cm (Fig. 14.18) and
join QR.
Measure ∠B, ∠C, ∠Q and ∠R. We shall find that ∠B = ∠Q and ∠C = ∠R
Notes
Thus, ΔABC ~ ΔPQR
Thus, we conclude that
If one angle of a triangle is equal to one angle of the other triangle and the sides
containing these angles are proportional, the triangles are similar.
Thus, we have three important criteria for the similarity of triangles. They are given below:
(i) If in two triangles, the corresponding angles are equal, the triangles are similar.
(ii) If the corresponding sides of two triangles are proportional, the triangles are
similar.
(iii) If one angle of a triangle is equal to one angle of the other triangle and the
sides containing these angles are proportional, the triangles are similar.
Example 14.5: In Fig. 14.19 two triangles ABC and PQR are given in which ∠A = ∠P
and ∠B = ∠Q. Is ΔABC ~ ΔPQR?.
Fig. 14.19
Solution: We are given that
∠A = ∠P and ∠B = ∠Q
We also know that
∠A + ∠B + ∠C = ∠P + ∠Q + ∠R = 180o
Therefore ∠C = ∠R
Thus, according to first criterion of similarity (AAA)
ΔABC ~ ΔPQR
Notes
m
9c
4.
8
cm
m
4c
Fig. 14.20
Solution: It is given that ΔABC ~ ΔPQR
AB AC
∴ =
PQ PR
Let PR = x cm
4 4.8
∴ =
9 x
⇒ 4 x = 9 × 4.8
⇒ x = 10.8
i.e., PR = 10.8 cm.
(i)
Fig. 14.21
Notes
(ii)
Fig. 14.22
(iii)
Fig. 14.23
It states that
The ratio of the areas of two similar triangles is equal to the ratio of the
squares of their corresponding sides.
Let us verify this result by the following activity. Draw two right triangles ABC and PQR
which are similar i.e., their sides are proportional (Fig. 14.25).
Fig. 14.25
Draw AD ⊥ BC and PS ⊥ QR.
Measure the lengths of AD and PS.
Find the product AD × BC and PS × QR
You will find that AD × BC = BC2 and PS × QR = QR2
Now AD × BC = 2 . Area of ΔABC
PS × QR = 2. Area of ΔPQR
BC AB AC
As = =
QR PQ PR
Notes Example 14.7: Find the ratio of the area of two similar triangles if one pair of their
corresponding sides are 2.5 cm and 5.0 cm.
Solution: Let the two triangles be ABC and PQR
Let BC = 2.5 cm and QR = 5.0 cm
AP AQ 2
∴ = =
BP QC 3
BP QC 3
∴ = = Fig. 14.26
AP AQ 2
BP QC 3 5
∴1 + = 1+ = 1+ =
AP AQ 2 2
AB AC 5 AP AQ 2
⇒ = = ⇒ = =
AP AQ 2 AB AC 5
∴ ΔAPQ ~ ΔABC
Area (ΔAPQ ) AP 2 ⎛ AP ⎞ ⎛ 2 ⎞
2 2
4
∴ = =⎜ ⎟ =⎜ ⎟ = (Q ΔAPQ ~ ΔABC)
Area (ΔABC) AB ⎝ AB ⎠ ⎝ 5 ⎠
2
25
Notes
Fig. 14.27
2. Find the ratio of the areas of two similar triangles if two of their corresponding sides
are of length 3 cm and 5 cm.
3. In Fig. 14.28, ABC is a triangle in which DE || BC. If AB = 6 cm and AD = 2 cm, find
the ratio of the areas of ΔADC and trapezium DBCE.
Fig. 14.28
4. P, Q and R are respectively the mid-points of the sides AB, BC and CA of the ΔABC.
Show that the area of ΔPQR is one-fourth the area of ΔABC.
5. In two similar triangles ABC and PQR, if the corresponding altitudes AD and PS are
in the ratio of 4 : 9, find the ratio of the areas of ΔABC and ΔPQR.
⎡ AB AD BC CA ⎤
⎢Hint : Use PQ = PS = QR = PR ⎥
⎣ ⎦
6. If the ratio of the areas of two similar triangles is 16 : 25, find the ratio of their
corresponding sides.
Example 14.12: ABC is a right triangle, right angled at C. If CD, the length of perpendicular
from C on AB is p, BC = a, AC = b and AB = c (Fig. 14.33), show that:
(i) pc = ab
Notes
1 1 1
(ii) 2
= 2+ 2
p a b
Solution: (i) CD ⊥ AB
∴ ΔABC ~ ΔACD
c a
∴ =
b p
⇒ pc = ab
(ii) AB2 = AC2 + BC2 Fig. 14.33
or c2 = b2 + a2
2
⎛ ab ⎞
⎜⎜ ⎟⎟ = b 2 + a 2
⎝ p⎠
1 a 2 + b2 1 1
or = 2 2 = 2+ 2
p2 a b a b
2. Two poles of height 6 m and 11 m, stand on a plane ground. If the distance between
their feet is 12 m, find the distance between their tops.
Notes
Fig. 14.34
5. L and M are the mid-points of the sides AB and AC of ΔABC, right angled at B.
Show that 4LC2 = AB2 + 4 BC2
6. P and Q are points on the sides CA and CB respectively of ΔABC, right angled at C
Prove that AQ2 + BP2 = AB2 + PQ2
7. PQR is an isosceles right triangle with ∠Q = 90o. Prove that PR2 = 2PQ2.
8. A ladder is placed against a wall such that its top reaches upto a height of 4 m of the
wall. If the foot of the ladder is 3 m away from the wall, find the length of the ladder.
LET US SUM UP
• Objects which have the same shape but different or same sizes are called similar objects.
• Any two polygons, with corresponding angles equal and corresponding sides
proportional are similar.
• If a line is drawn parallel to one-side of a triangle, it divides the other two sides in the
same ratio and its converse.
• The bisector of an interior angle of a triangle divides the opposite side in the ratio of
sides containing the angle.
• Two triangles are said to be similar, if
(a) their corresponding angles are equal and
(b) their corresponding sides are proportional
• Criteria of similarity
- AAA criterion
- SSS criterion
- SAS criterion
• If a perpendicular is drawn from the vertex of the right angle of a right angled triangle
to the hypotenuse, the triangles so formed are similar to each other and to the given
triangle.
Notes
• The ratio of the areas of two similar triangles is equal to the ratio of squares of their
corresponding sides.
• In a right triangle, the square on the hypotenuse is equal to sum of the squares on the
remaining two sides – (Baudhayan Pythagoras Theorem).
• In a triangle, if the square on one side is equal to the sum of the squares on the
remaining two sides, then the angle opposite to the first side is a right angle – converse
of (Baudhayan) Pythagoras Theorem.
TERMINAL EXERCISE
1. Write the criteria for the similarity of two polygons.
2. Enumerate different criteria for the similarity of the two triangles.
3. In which of the following cases, Δ’s ABC and PQR are similar.
(i) ∠A = 40o, ∠B = 60o, ∠C = 80o, ∠P = 40o, ∠Q = 60o and ∠R = 80o
(ii) ∠A = 50o, ∠B = 70o, ∠C = 60o, ∠P = 50o, ∠Q = 60o and ∠R = 70o
(iii) AB = 2.5 cm, BC = 4.5 cm, CA = 3.5 cm
PQ = 5.0 cm, QR = 9.0 cm, RP = 7.0 cm
(iv) AB = 3 cm, QR = 7.5 cm, RP = 5.0 cm
PQ = 4.5 cm, QR = 7.5 cm, RP = 6.0 cm.
4. In Fig. 14.35, AD = 3 cm, AE = 4.5 cm, DB = 4.0 cm, find CE, give that DE || BC.
Notes
14. A ladder is placed against a wall and its top reaches a point at a height of 8 m from the
ground. If the distance between the wall and foot of the ladder is 6 m, find the length of
the ladder.
Notes
15. In an equilateral triangle, show that three times the square of a side equals four times
the square of medians.
AD AE
3. (i) and (iii) 4. 6 cm 5. 4.5 cm 6. Yes : =
DB EC
7. 4.5 cm 8. 4 : 9 9. 25 : 49 10. (i), (ii), (iv) and (v)
11. 3 a2 12. 13 m 14. 10 m
15 Notes
CIRCLES
You are already familiar with geometrical figures such as a line segment, an angle, a triangle,
a quadrilateral and a circle. Common examples of a circle are a wheel, a bangle, alphabet
O, etc. In this lesson we shall study in some detail about the circle and related concepts.
OBJECTIVES
After studying this lesson, you will be able to
• define a circle
• give examples of various terms related to a circle
• illustrate congruent circles and concentric circles
• identify and illustrate terms connected with circles like chord, arc, sector, segment,
etc.
• verify experimentally results based on arcs and chords of a circle
• use the results in solving problems
15.1.3 Arc
A part of a circle is called an arc. In Fig. 15.5(a) ABC is an arc and is denoted by arc
ABC
(a) (b)
Fig. 15.5
15.1.4 Semicircle
A diameter of a circle divides a circle into two equal arcs, each of which is known as a
semicircle.
In Fig. 15.5(b), PQ is a diameter and PRQ is semicircle and so is PBQ.
15.1.5 Sector
The region bounded by an arc of a circle and two radii at
its end points is called a sector.
In Fig. 15.6, the shaded portion is a sector formed by the
arc PRQ and the unshaded portion is a sector formed by
the arc PTQ.
Fig. 15.8
Activity for you :
Take a wheel and mark a point P on the wheel where it touches the ground. Rotate the
wheel along a line till the point P comes back on the ground. Measure the distance be-
tween the Ist and last position of P along the line. This distance is equal to the circumfer-
ence of the circle. Thus,
The length of the boundary of a circle is the circumference of the circle.
Activity for you
Consider different circles and measures their circumference(s) and diameters. Observe
that in each case the ratio of the circumference to the diameter turns out to be the same.
The ratio of the circumference of a circle to its diameter is always a con-
stant. This constant is universally denoted by Greek letter π .
c c
Therefore, = = π , where c is the circumference of the circle, d its diameter and r is
d 2r
its radius.
22
An approximate value of π is . Aryabhata -I (476 A.D.), a famous Indian Mathema-
7
tician gave a more accurate value of π which is 3.1416. In fact this number is an
irrational number.
An arc of a circle whose length is greater than that of a semicircle of the same circle is
called a major arc. In Fig. 15.9, arc PBQ is a major arc.
Fig. 15.10
15.4 CONGRUENT CIRCLES OR ARCS
Two cirlces (or arcs) are said to be congruent if we can superimpose (place) one over the
other such that they cover each other completely.
Fig. 15.11
then PQ = RS
Equal chords of a circle subtend equal angles at the centre and conversely
if the angles subtended by the chords at the centre of a circle are equal,
then the chords are equal.
Note : The above results also hold good in case of congruent circles.
PN = NQ.
The perpendicular drawn from the centre of a circle to a chord bisects the
chord.
Activity for you :
(i) Draw a circle with centre O (See Fig. 15.18).
(ii) Draw a chord PQ.
(iii) Find the mid point M of PQ.
(iv) Join O and M.
Fig. 15.18
Notes The line joining the centre of a circle to the mid point of a chord is perpen-
dicular to the chord.
Activity for you :
Take three non collinear points A, B and C. Join AB and
BC. Draw perpendicular bisectors MN and RS of AB
and BC respectively.
Since A, B, C are not collinear, MN is not parallel to RS.
They will intersect only at one point O. Join OA, OB and
OC and measure them.
Fig. 15.20
We observe that OA = OB = OC
Now taking O as the centre and OA as radius draw a circle which passes through A, B
and C.
Repeat the above procedure with another three non-collinear points and observe that
there is only one circle passing through three given non-collinear points.
There is one and only one circle passing through three non-collinear points.
Note. It is important to note that a circle can not be drawn to pass through three collinear
points.
Activity for you :
(i) Draw a circle with centre O [Fig. 15.20a]
(ii) Draw two equal chords AB and PQ of the circle.
(iii) Draw OM ⊥ PQ and ON ⊥ PQ
(iv) Measure OM and ON and observe that they are equal.
Fig. 15.20a
Equal chords of a circle are equidistant from the centre.
In Fig. 15.20 b, OM = ON
Measure and observe that AB = PQ. Thus,
Chords, that are equidistant from the centre of a
circle, are equal.
The above results hold good in case of congruent circles
also.
We now take a few examples using these properties of Fig. 15.20b
circle.
1
∴ OD = CA
2
i.e. CA = 2OD.
Example 15.5 : A regular hexagon is inscribed in a circle. What angle does each side of
the hexagon subtend at the centre?
Solution : A regular hexagon has six sides which are equal.
Therefore each side subtends the same angle at the centre.
Let us suppose that a side of the hexagon subtends an
angle xo at the centre.
Then, we have
6xo = 360o ⇒ x = 60o Fig. 15.23
Hence, each side of the hexagon subtends an angle of 60o at the centre.
2
⎛7⎞
and r = ( x + 3) + ⎜ ⎟
2 2
...(ii)
⎝2⎠
Therefore from (i) and (ii),
2 2
⎛ 13 ⎞ ⎛7⎞
x + ⎜ ⎟ = ( x + 3) 2 + ⎜ ⎟
2
⎝2⎠ ⎝2⎠
169 49
∴ 6x = −9−
4 4
or 6x = 21
7
∴ x=
2
2 2
⎛ 7 ⎞ ⎛ 13 ⎞ 49 169 218
∴ r = ⎜ ⎟ +⎜ ⎟ =
2
+ =
⎝2⎠ ⎝ 2 ⎠ 4 4 4
218
∴ r=
2
218
Hence the radius of the circle is r = cm.
2
⎛ 22 ⎞
(i) 7 cm (ii) 11 cm. ⎜ Take π = ⎟
⎝ 7 ⎠
8. In the Fig. 15.26, RS is a diameter which bisects the chords PQ and AB at the points
M and N respectively. Is PQ || AB ? Given reasons.
LET US SUM UP
• The circumference of a circle of radius r is equal to 2 π r.
TERMINAL EXERCISE
1. If the length of a chord of a circle is 16 cm and the distance of the chord from the
centre is 6 cm, find the radius of the circle.
2. Two circles with centres O and O′ (See Fig. 15.28) are congurent. Find the length of
the arc CD.
Fig. 15.28
3. A regular pentagon is inscribed in a circle. Find the angle which each side of the
pentagon subtends at the centre.
4. In Fig. 15.29, AB = 8 cm and CD = 6 cm are two parallel chords of a circle with
centre O. Find the distance between the chords.
Fig. 15.29
Notes
Fig. 15.30
6. In Fig. 15.31, AB and CD are two equal chords of a circle with centre O. Is chord
BD = chord CA? Give reasons.
Fig. 15.31
7. If AB and CD are two equal chords of a circle with centre O (Fig. 15.32) and
OM ⊥ AB, ON ⊥ CD. Is OM = ON ? Give reasons.
Fig. 15.32
8. In Fig. 15.33, AB = 14 cm and CD = 6 cm are two parallel chords of a circle with
centre O. Find the distance between the chords AB and CD.
Fig. 15.33
Notes
Fig. 15.34
OM ⊥ CD, ON ⊥ AB and ∠ OPM = ∠ OPN. Now answer:
Is (i) OM = ON, (ii) AB = CD ? Give reasons.
10. C1 and C2 are concentric circles with centre O (See Fig. 15.35), l is a line intersecting
C1 at points P and Q and C2 at points A and B respectively, ON ⊥ l, is PA = BQ?
Give reasons.
Fig. 15.35
ANSWERS TO CHECK YOUR PROGRESS
15.1
1. (i) Chord (ii) APB 2. Diameter 3. Constant
4. Aryabhata-I 5. Concentric 6. 5 5 cm.
7. (i) 44 cm (ii) 69.14 cm 8. Yes 9. Yes
16 Notes
You must have measured the angles between two straight lines. Let us now study the
angles made by arcs and chords in a circle and a cyclic quadrilateral.
OBJECTIVES
After studying this lesson, you will be able to
• verify that the angle subtended by an arc at the centre is double the angle
subtended by it at any point on the remaining part of the circle;
• prove that angles in the same segment of a circle are equal;
• cite examples of concyclic points;
• define cyclic quadrilterals;
• prove that sum of the opposite angles of a cyclic quadrilateral is 180o;
• use properties of cyclic qudrilateral;
• solve problems based on Theorems (proved) and solve other numerical problems
based on verified properties;
• use results of other theorems in solving problems.
40o 2
then length of the arc PQR = 2πr. o
= πr
360 9
Inscribed angle : The angle subtended by an
arc (or chord) on any point on the remaining part
of the circle is called an inscribed angle.
In Fig. 16.3, ∠ PAQ is the angle inscribed by arc Fig. 16.3
PRQ at point A of the remaining part of the circle
or by the chord PQ at the point A.
∴ 2 ∠ PBQ = ∠ POQ
(Since the angle subtended by an arc at the centre
Fig. 16.5
is double the angle subtended by it at any point
on the remaining part of the circle)
But ∠ POQ = 1800
2 ∠ PBQ = 1800
∴ ∠ PBQ = 900
Thus, we conclude the following:
Angle in a semicircle is a right angle.
Theorem : Angles in the same segment of a circle are equal
Given : A circle with centre O and the angles ∠ PRQ and ∠ PSQ in the same segment
formed by the chord PQ (or arc PAQ)
To prove : ∠ PRQ = ∠ PSQ
Construction : Join OP and OQ.
Proof : As the angle subtended by an arc at the
centre is double the angle subtended by it at any
point on the remaining part of the circle, therefore
we have
Fig. 16.6
∠ POQ = 2 ∠ PRQ ...(i)
and ∠ POQ = 2 ∠ PSQ ...(ii)
From (i) and (ii), we get
Notes
Example 16.3 : In Fig. 16.9, O is the centre of the circle and AD bisects ∠ BAC. Find
∠ BCD.
Solution : Since BC is a diameter
∠ BAC =90
o
∴ ∠ BCD = 45o
Example 16.4 : In Fig. 16.10, O is the centre of the circle, ∠ POQ = 70o and PS⊥OQ.
Find ∠ MQS.
Solution :
(Angle subtended at the centre of a circle is twice the angle subtended by it on the
remaining part of the circle)
∴ ∠ PSQ = 35o
Since ∠ MSQ + ∠ SMQ + ∠ MQS = 180o
Notes
Fig. 16.11
2. In Fig. 16.12, AOB is a diameter of a circle with centre O. Is ∠APB = ∠AQB = 90o.
Give reasons.
Fig. 16.12
3. In Fig. 16.13, PQR is an arc of a circle with centre O. If ∠ PTR = 350, find ∠ PSR.
Fig. 16.13
4. In Fig. 16.14, O is the centre of a circle and ∠ AOB = 600. Find ∠ ADB.
Fig. 16.14
Fig. 16.16
Let us now take three points P, Q and R which do not lie on the same straight line. In this
case you can draw only one circle passing through these three non-colinear points
(Fig. 16.17).
Fig. 16.17
Further let us now take four points P, Q, R, and S which do not lie on the same line. You
will see that it is not always possible to draw a circle passing through four non-collinear
points.
In Fig. 16.18 (a) and (b) points are noncyclic but concyclic in Fig. 16.18(c)
(c)
Fig. 16.18
Hence proved.
Converse of this theorem is also true.
Fig. 16.21
and ∠ S + ∠ Q = 180o
Therefore draw a circle passing through the point P, Q and R and observe that it also
passes through the point S. So we conclude that quadrilateral PQRS is cyclic quadrilateral.
We solve some examples using the above results.
Example 16.5 : ABCD is a cyclic parallelogram.
Show that it is a rectangle.
Solution : ∠ A + ∠ C = 180o
(ABCD is a cyclic quadrilateral)
Since ∠A= ∠C Fig. 16.22
Similarly, ∠ Q + ∠ S = 180o
∴ ∠ S = 180o – ∠ Q =180o – 65o
Fig. 16.25
∴ ∠ S = 115o.
Fig. 16.26
2. In Fig. 16.27, PQRS is a cyclic quadrilateral, and the side PS is extended to the point
A. If ∠ PQR = 800, find ∠ ASR.
Notes
Fig. 16.27
3. In Fig. 16.28, ABCD is a cyclic quadrilateral whose diagonals intersect at O. If
∠ ACB = 50 and ∠ ABC = 110 , find ∠ BDC.
o o
Fig. 16.28
4. In Fig. 16.29, ABCD is a quadrilateral. If ∠ A = ∠ BCE, is the quadrilateral a cyclic
quadrilateral? Give reasons.
Fig. 16.29
LET US SUM UP
• The angle subtended by an arc (or chord) at the centre of a circle is called central
angle and an ngle subtended by it at any point on the remaining part of the circle is
called inscribed angle.
• Points lying on the same circle are called concyclic points.
• The angle subtended by an arc at the centre of a circle is double the angle subtended
by it at any point on the remaining part of the circle.
TERMINAL EXERCISE
1. A square PQRS is inscribed in a circle with centre O. What angle does each side
subtend at the centre O?
2. In Fig. 16.30, C1 and C2 are two circles with centre O1 and O2 and intersect each
other at points A and B. If O1O2 intersect AB at M then show that
(i) ΔO1AO2 ≅ ΔO1BO2
(ii) M is the mid point of AB
(iii) AB ⊥ O1O2
Fig. 16.30
[(Hint. From (i) conclude that ∠ 1 = ∠ 2 and then prove that ΔAO1M ≅ ΔBO1M
(by SAS rule)].
3. Two circles intersect in A and B. AC and AD are the diameters of the circles. Prove
that C, B and D are collinear.
Notes
Fig. 16.31
[Hint. Join CB, BD and AB, Since ∠ ABC = 90o and ∠ ABD = 90o]
4. In Fig. 16.32, AB is a chord of a circle with centre O. If ∠ ACB = 40o, find
∠ OAB.
Fig. 16.32
5. In Fig. 16.33, O is the centre of a circle and ∠ PQR = 115o. Find ∠ POR.
Fig. 16.33
6. In Fig. 16.34, O is the centre of a circle, ∠ AOB = 80o and ∠ PQB = 70o. Find
∠ PBO.
Fig. 16.34
17 Notes
Look at the moving cycle. You will observe that at any instant of time, the wheels of the
moving cycle touch the road at a very limited area, more correctly a point.
If you roll a coin on a smooth surface, say a table or floor, you will find that at any instant
of time, only one point of the coin comes in contact with the surface it is rolled upon.
What do you observe from the above situations?
(i) (ii)
Fotball
(iii)
Fig. 17.1
If you consider a wheel or a coin as a circle and the touching surface (road or table) as a
line, the above illustrations show that a line touches a circle. In this lesson, we shall study
about the possible contacts that a line and a circle can have and try to study their properties.
Notes
OBJECTIVES
After studying this lesson, you will be able to
• define a secant and a tangent to the circle;
• differentitate between a secant and a tangent;
• prove that the tangents drawn from an external point to a circle are of equal
length;
• verify the un-starred results (given in the curriculum) related to tangents and
secants to circle experimentally.
(i) The line does not intersect the circle at all, i.e., the line lies in the exterior of the circle.
(ii) The line intersects the circle at two distinct points. In that case, a part of the line lies in
the interior of the circle, the two points of intersection lie on the circle and the remaining
portion of the line lies in the exterior of the circle.
(iii) The line touches the circle in exactly one point. We therefore define the following:
Tangent:
A line which touches a circle at exactly one point is called a tangent line and
the point where it touches the circle is called the point of contact
Thus, in Fig. 17.2 (iii), XY is a tangent of the circle at P, which is called the point of contact.
Secant:
A line which interesects the circle in two distinct points is called a secant
line (usually referred to as a secant).
In Fig. 17.2 (ii), XY is a secant line to the circle and A and B are called the points of
intersection of the line XY and the circle with centre O.
CA = AR + CR
= (3 + 3.5) cm
∴ = 6.5 cm
∴ Perimeter of ΔABC = (7 + 7.5 + 6.5) cm = 21 cm
Example 17.4: In Fig. 17.10, ∠AOB = 50o. Find ∠ABO and ∠OBT.
Solution: We know that OA ⊥ XY
⇒ ∠OAB = 90o
∴ ∠ABO = 180o – (∠OAB + ∠AOB)
= 180o – (90o + 50o) = 40o
We know that ∠OAB = ∠OBT
⇒ ∠OBT = 40o
B
∴ ∠ABO = ∠OBT = 40o
Fig. 17.10
We know that PA × PB = PC × PD
⇒ 3 × 2 = (1.5) × x
3× 2
⇒ x= =4
1.5
∴ Length of the line-segment PD = 4 cm.
Example 17.6: In Fig. 17.17, PAB is a secant to the circle from a point P outside the
circle. PAB passes through the centre of the circle and PT is a tangent. If PT = 8 cm and
OP = 10 cm, find the radius of the circle, using PA × PB = PT2
Solution: Let x be the radius of the circle.
It is given that OP = 10 cm
∴ PA = PO – OA = (10 – x) cm
and PB = OP + OB = (10 + x) cm
PT = 8 cm
We know that PA × PB = PT2
Fig. 17.17
∴ (10 – x) (10 + x) = 8 2
or 100 – x2 = 64
or x2 = 36 or x = 6
i.e., radius of the circle is 6 cm.
Example 17.7: In Fig. 17.18, BA and DC are two chords of a circle intersecting each
other at a point P outside the circle. If PA = 4 cm, PB = 10 cm, CD = 3 cm, find PC.
LET US SUM UP
• A line which intersects the circle in two points is called a secant of the circle.
• A line which touches the circle at a point is called a tangent to the circle.
• A tangent is the limiting position of a secant when the two points of intersection coincide.
• A tangent to a circle is perpendicular to the radius through the point of contact.
• From an external point, two tangents can be drawn to a circle, which are of equal
length.
• If two chords AB and CD of a circle intersect at a point P (inside or outside the circle), then
PA × PB = PC × PD
• If PAB is a secant to a circle intersecting the circle at A and B, and PT is a tangent to
the circle at T, then
PA × PB = PT2
• The angles formed in the alternate segments by a chord through the point of contact of
a tangent to a circle are equal to the angles between the chord and the tangent.
• If a line makes with a chord angles which are respectively equal to the angles formed
by the chord in alternate segments, then the line is a tangent to the circle.
TERMINAL EXERCISE
1. Differentitate between a secant and a tangent to a circle with the help of a figure.
2. Show that a tangent is a line perpendicular to the
radius through the point of contact, with the help
of an activity.
3. In Fig. 17.28, if AC = BC and AB is a diameter
of circle, find ∠x, ∠y and ∠z.
Fig. 17.28
1 Fig. 17.30
[Hint: ∠OBC + ∠OCB = (∠ABC +
2
∠ACB)]
7. In Fig. 17.31, AB and CD are two chords of a
circle intersecting at the interior point P of a circle.
If PA = (x + 3) cm, PB = (x – 3) cm, PD = 3 cm
1
and PC = 5 cm, find x.
3
Fig. 17.31
Fig. 17.32
Fig. 17.33
(iii) ∠OPQ
Fig. 17.34
Notes
18
CONSTRUCTIONS
One of the aims of studying Geometry is to acquire the skill of drawing figures accurately.
You have learnt how to construct geometrical figures namely triangles, squares and circles
with the help of ruler and compasses. You have constructed angles of 30o, 60o, 90o, 120o
and 45o. You have also drawn perpendicular bisector of a line segment and bisector of an
angle.
In this lesson we will extend our learning to construct some other important geometrical
figures.
OBJECTIVES
After studying this lesson, you will be able to
• divide a given line segment internally in a given ratio;
• construct a triangle from the given data;
(i) SSS
(ii) SAS
(iii) ASA
(iv) RHS
(v) perimeter and base angles
(vi) base, sum/difference of the other two sides and one base angle.
(vii) two sides and a median corresponding to one of these sides.
• construct a triangle, similar to a given triangle; and;
• Construct tangents to a circle from a point:
(i) on it using the centre of the circle.
(i) outside it.
Fig. 18.1
Then D is the required point which divides AB internally in the ratio 2 : 3 as shown in
Fig. 18.1.
3
2. Draw a line segment PQ = 8 cm. Find point R on it such that PR = PQ.
4
[Hint: Divide the line segment PQ internally in the ratio 3 : 1]
Notes
Notes
Fig. 18.6
ΔABC is the required triangle.
Construction 7: To construct a triangle when sum of two sides, third side and one of the
angles on the third side are given.
Suppose you are required to construct a triangle ABC in which
AB + AC = 8.2 cm, BC = 3.6 cm and ∠B = 45o
To construct the triangle, we go through the following steps:
Step 1: Draw BC = 3.6 cm
Step 2: At B, construct ∠CBK = 45o
Fig. 18.7
Step 3: From BK, cut off BP = 8.2 cm.
Step 4: Join CP.
Step 5: Draw right bisector of CP intersecting BP at A.
Step 6: Join AC
ΔABC is required triangle.
Notes 4. Construct a right triangle in which one side is 3 cm and hypotenuse is 7.5 cm.
5. Construct a right angled isoceles triangle in which one of equal sides is 4.8 cm.
6. Construct a ΔABC given that AB + BC + AC = 10 cm, ∠B = 60o, ∠C = 30o.
7. Construct a ΔABC in which AB = 5 cm, ∠A = 60o, BC + AC = 9.8 cm.
8. Construct a ΔLMN, when ∠M = 30o, MN = 5 cm and LM – LN = 1.5 cm.
9. Construct a triangle PQR in which PQ = 5 cm, QR = 4.2 cm and median
RS = 3.8 cm.
TERMINAL EXERCISE
1. Draw a line segment PQ = 8 cm long. Divide it internally in the ratio 3 : 5. Also write
the steps of construction.
Note: You are also required to write the steps of construction in each of the
following problems.
2. Draw a line segment AB = 6 cm. Find a point C on AB such that AC : CB = 3 : 2.
Measure AC and CB
Notes
19
CO-ORDINATE GEOMETRY
The problem of locating a village or a road on a large map can involve a good deal of
searching. But the task can be made easier by dividing it into squares of managable size.
Each square is identified by a combination of a letter and a number, or of two numbers,
one of which refers to a vertical division of the map into columns, and the other to a
horizontal division into rows.
Fig. 19.1
OBJECTIVES
After studying this lesson, you will be able to
• fix the position of different points in a plane, whose coordinates are given, using
rectangular system of coordinates and vice-versa;
• find the distance between two different points whose co-ordinates are given;
• find the co-ordinates of a point, which divides the line segment joining two
points in a given ratio internally;
• find the co-ordinates of the mid-point of the join of two points;
• find the co-ordinates of the centroid of a triangle with given vertices;
• solve problems based on the above concepts.
Notes
Fig. 19.2
The horizontal number line XOX′ is called x-axis and the vertical number line YOY′ is
called y-axis. The point O, where both axes intersect each other is called the origin. The
two axes together are called rectangular coordinate system.
It may be noted that, the positive direction of x-axis is taken to the right of the origin O,
OX and the negative direction is taken to the left of the origin O, i.e., the side OX′.
Similarly, the portion of y-axis above the origin O, i.e., the side OY is taken as positive and
the portion below the origin O, i.e., the side OY′ is taken as negative.
Fig. 19.3
Fig. 19.4
Example 19.1: Write down x and y co-ordinates for each of the following points
(a) (l, 1) (b) (–3, 2) (c) (–7, –5) (d) (2, –6)
Solution : (a) x co-ordinate is l (b) x co-ordinate is –3
y co-ordinate is l y co-ordinate is 2.
(c) x co-ordinate is –7 (d) x co-ordinate is 2
y co-ordinate is –5. y co-ordinate is –6.
19.3 QUADRANTS
The two axes XOX′ and YOY′ divide the plane into four parts called quadrants.
Fig. 19.5
The four quadrants (See Fig. 19.5) are named as follows :
XOY : I Quadrant ; YOX’ : II Quadrant;
X’OY’ : III Quadrant ; Y’OX : IV Quadrant.
We have discussed in Section 19.4 that
(i) along x-axis, the positive direction is taken to the right of the origin and negative
direction to its left.
(ii) along y-axis, portion above the x-axis is taken as positive and portion below the
x-axis is taken as negative (See Fig. 19.6)
Notes
For example :
(a) P(5, 6) lies in the first quadrant as both x and y co-ordinates are positive.
(b) Q(–3, 4) lies in the second quadrant as its x co-ordiante is negative and y co-ordinate
is positive.
(c) R (–2, –3) lies in the third quadrant as its both x and y co-ordinates are negative.
(d) S(4, –1) lies in the fourth quadrant as its x co-ordinate is positive and y coordinate is
negative.
Fig. 19.9
QR = QM – RM = QM – PL = y2 – y1
∴ PQ = (x 2 − x1 )2 + (y 2 − y1 )2
Therefore,
(x − 0)2 + (3 − 0)2
It is given that
(x − 0)2 + (3 − 0)2 =5
or x 2 + 32 = 5
Squaring both sides,
x2 + 9 = 25
or x2 = 16
or x= ±4
Hence x = + 4 or – 4 units
Example 19.5: Show that the points (1, 1), (3, 0) and (–1, 2) are collinear.
Solution: Let P(1, 1), Q(3, 0) and R(–1, 2) be the given points
Now, PQ + RP = ( )
5 + 5 units = 2 5 units = QR
∴ P, Q and R are collinear points.
Example 19.6: Find the radius of the circle whose centre is at (0, 0) and which passes
through the point (–6, 8).
Solution: Let O(0, 0) and B(–6, 8) be the given points of the line segment OB.
∴ OB = (− 6 − 0)2 + (8 − 0)2
= 36 + 64 = 100
O Notes
= 10
Hence radius of the circle is 10 units.
Fig. 19.11
CHECK YOUR PROGRESS 19.2
1. Find the distance between each of the following pair of points:
(a) (3, 2) and (11, 8) (b) (–1, 0) and (0, 3)
(c) (3, –4) and (8, 5) (d) (2, –11) and (–9, –3)
2. Find the radius of the circle whose centre is at (2, 0) and which passes through the
point (7, –12).
3. Show that the points (–5, 6), (–1, 2) and (2, –1) are collinear
AP AK KP
= = ...(i)
PB PT TB
Now, AK = LM = OM – OL = x – x1
PT = MN = ON – OM = x2 – x
KP = MP – MK = MP – LA = y – y1
TB = NB – NT = NB – MP = y2 – y
∴ From (i), we have
m x − x1 y − y1
= =
n x 2 − x y2 − y
m x − x1
=
n x2 − x
or mx2 – mx = nx – nx1
or x(m + n) = mx2 + nx1
mx 2 + nx1
or x=
m+n
AP KP
Similarly, from the raltion = , we get
PB TB
m y − y1
=
n y 2 − y which gives on simplification.
my 2 + ny1
y=
m+n
mx 2 + nx1 my 2 + ny1
∴ x= , and y = ...(i)
m+n m+n
Hence co-ordiantes of a point which divides the line segment joining the points
(x1, y1) and (x2, y2) in the ratio m : n internally are :
⎛ mx 2 + nx 1 my 2 + ny 1 ⎞
⎜ , ⎟
⎝ m+n m+n ⎠
The co-ordinates of the mid-point of the line segment joining two points (x1, y1) and
(x2, y2) can be obtained by taking m = n in the section formula above.
Putting m = n in (1) above, we have
nx 2 + nx1 x 2 + x1
x= =
n+n 2
ny 2 + ny1 y 2 + y1
and y= =
n+n 2
The co-ordinates of the mid-point joining two points (x1, y1) and (x2, y2) are:
⎛ x 2 + x1 y 2 + y 1 ⎞
⎜ , ⎟
⎝ 2 2 ⎠
Notes
Let us take some examples to illustrate:
Example 19.7: Find the co-ordinates of a point which divides the line segment joining
each of the following points in the given ratio:
(a) (2, 3) and (7, 8) in the ratio 2 : 3 internally.
(b) (–1, 4) and (0, –3) in the ratio 1 : 4 internally.
Solution: (a) Let A(2, 3) and B(7, 8) be the given points.
Let P(x, y) divide AB in the ratio 2 : 3 internally.
Using section formula, we have
2 × 7 + 3 × 2 20
x= = =4
2+3 5
2 × 8 + 3 × 3 25
and y= = =5
2+3 5
∴ P(4, 5) divides AB in the ratio 2 : 3 internally.
(b) Let A(–1, 4) and B(0, –3) be the given points.
Let P(x, y) divide AB in the ratio 1 : 4 internally.
Using section formula, we have
1× 0 + 4 × (− 1) 4
x= =−
1+ 4 5
1× (− 3) + 4 × 4 13
and y= =
1+ 4 5
⎛ 4 13 ⎞
∴ P⎜ − , ⎟ divides AB in the ratio 1 : 4 internally.
⎝ 5 5⎠
Example 19.8: Find the mid-point of the line segment joining two points (3, 4) and (5,
12).
Solution: Let A(3, 4) and B(5, 12) be the given points.
Let C(x, y) be the mid-point of AB. Using mid-point formula, we have,
3+5
x= =4
2
4 + 12
and y= =8
2
Notes ∴ C(4, 8) are the co-ordinates of the mid-point of the line segment joining two points (3,
4) and (5, 12).
Example 19.9: The co-ordinates of the mid-point of a segment are (2, 3). If co-ordinates
of one of the end points of the line segment are (6, 5), find the co-ordinates of the other
end point.
Solution: Let other end point be A(x, y) A(x, y) C(2, 3) B(6, 5)
x+6 y+5
2= and 3=
2 2
or 4=x+6 or 6=y+5
or x=–2 or y=1
∴ (–2, 1) are the coordinates of the other end point.
⎛ x + x 3 y 2 + y3 ⎞
D=⎜ 2 , ⎟
⎝ 2 2 ⎠
Fig. 19.14
x2 + x3
2× + 1× x 1
2 x + x2 + x3 Notes
x= = 1
2 +1 3
y 2 + y3
2× + 1× y1
2 y + y 2 + y3
y= = 1
2 +1 3
Hence, the co-ordiantes of the centroid are given by
⎛ x1 + x 2 + x 3 y 1 + y 2 + y 3 ⎞
⎜ , ⎟
⎝ 3 3 ⎠
Example 19.10: The co-ordinates of the vertices of a triangle are (3, –1), (10, 7) and
(5, 3). Find the co-ordinates of its centroid.
Solution: Let A(3, –1), B(10, 7) and C(5, 3) be the vertices of a triangle.
Let G(x, y) be its centroid.
3 + 10 + 5
Then, x= =6
3
−1+ 7 + 3
and y= =3
3
Hence, the coordinates of the Centroid are (6, 3).
LET US SUM UP
Notes • If (2, 3) are the co-ordinates of a point, then x co-ordiante (or abscissa) is 2 and the
y co-ordinate (or ordinate) is 3.
• In any co-ordiante (x, y), ‘x’ indicates the distance from the y-axis and ý’ indicates the
distance from the x-axis.
• The co-ordinates of the origin are (0, 0)
• The y co-ordinate of every point on the x-axis is 0 and the x co-ordiante of every point
on the y-axis is 0.
• The two axes XOX’ and YOY’divide the plance into four parts called quadrants.
• The distance of the line segment joining two points (x1, y1) and (x2, y2) is given by:
(x 2 − x1 )2 + (y 2 − y1 )2
• The distance of the point (x1, y1) from the origin (0, 0) is x1 + y1
2 2
• The co-ordinates of a point, which divides the line segment joining two points (x1, y1)
and (x2, y2) in a ratio m : n internally are given by:
⎛ mx 2 + nx1 my 2 + ny1 ⎞
⎜ , ⎟
⎝ m+n m+n ⎠
• The co-ordinates of the mid-point of the line segment joining two points (x1, y1) and
(x2, y2) are given by:
⎛ x 2 + x1 y 2 + y1 ⎞
⎜ , ⎟
⎝ 2 2 ⎠
• The co-ordiantes of the centroid of a triangle whose vertices are (x1, y1), (x2, y2) and
(x3, y3) are given by
⎛ x 1 + x 2 + x 3 y1 + y 2 + y 3 ⎞
⎜ , ⎟
⎝ 3 3 ⎠
TERMINAL EXERCISE
1. In Fig. 19.15, AB = AC. Find x.
Notes
Fig. 19.15
2. The length of the line segment joining two points (2, 3) and (4, x) is 13 units. Find x.
3. Find the lengths of the sides of the triangle whose vertices are A(3, 4), B(2, –1) and
C(4, –6).
4. Prove that the points (2, –2), (–2, 1) and (5, 2) are the vertices of a right angled
triangle.
5. Find the co-ordinates of a point which divides the join of (2, –1) and (–3, 4) in the
ratio of 2 : 3 internally.
6. Find the centre of a circle, if the end points of a diameter are P(–5, 7) and Q(3, –11).
7. Find the centroid of the triangle whose vertices are P(–2, 4), Q(7, –3) and R(4, 5).
Notes 19.2
1. (a) 10 units (b) 10 units (c) 106 units (d) 185 units
2. 13 units
19.3
1. (a) (2, 1) (b) (–1, 1)
⎛ 5⎞ ⎛ 7 ⎞
2. (a) ⎜ 4,− ⎟ (b) ⎜ − ,5 ⎟
⎝ 2⎠ ⎝ 2 ⎠
⎛1 5⎞
3. ⎜ , ⎟
⎝3 3⎠
5. (0, 1)
6. (–1, –2)
7. (3, 2)
Secondary Course
Mathematics
Notes
Practice Work-Geometry
Instructions:
1
6. In ΔABC, AB = 10 cm and DE is parallel to BC such that AE = AC. Find AD. 2
4
(i) Fencing or construcing some kind of a boundary around the field, where the
crops were to be grown.
(ii) Allotting lands of different sizes for growing different crops.
(iii) Making suitable places for storing different products grown under different crops.
These problems led to the need of measurement of perimeters (lengths), areas and
volumes, which in turn gave rise to a branch of mathematics known as Mensuration.
In it, we deal with problems such as finding the cost of putting barbed wire around a
field, finding the number of tiles required to floor a room, finding the number of
bricks, required for creating a wall, finding the cost of ploughing a given field at a
given rate, finding the cost of constructing a water tank for supplying water in a
colony, finding the cost of polishing a table-top or painting a door and so on. Due to
the above type of problems, sometimes mensuration is referred to as the science of
“Furnitures and Walls”.
For solving above type of problems, we need to find the perimeters and areas of
simple closed plane figures (figure which lie in a plane) and surface areas and volumes
of solid figures (figures which do not lie wholly in a plane). You are already familiar
with the concepts of perimeters, areas, surface areas and volumes. In this module,
we shall discuss these in details, starting with the results and formulas with which
you are already familiar.
Perimeters and Areas of Plane Figures
MODULE - 4
Mensuration
20 Notes
You are already familair with a number of plane figures such as rectangle, square,
parallelogram, triangle, circle, etc. You also know how to find perimeters and areas of
these figures using different formulae. In this lesson, we shall consolidate this knowledge
and learn something more about these, particularly the Heron’s formula for finding the area
of a triangle and formula for finding the area of a sector of a circle.
OBJECTIVES
After studying this lesson, you will be able to
• find the perimeters and areas of some triangles and quadrilaterals, using formulae
learnt earlier;
• use Heron’s formula for finding the area of a triangle;
• find the areas of some rectilinear figures (including rectangular paths) by dividing
them into known figures such as triangles, squares, trapeziums, rectangles, etc.;
• find the circumference and area of a circle;
• find the areas of circular paths;
• derive and understand the formulae for perimeter and area of a sector of a
circle;
• find the perimeter and the area of a sector, using the above formulae;
• find the areas of some combinations of figures involving circles, sectors as well
as triangles, squares and rectangles;
• solve daily life problems based on perimeters and areas of various plane
figures.
1
(vi) Area of a triangle = base × corresponding altitude
2
1
(vii)Area of a rhombus = product of its diagonals
2
1
(viii) Area of a trapezium = (sum of the two parallel sides) × distance between them
2
(ix) circumference of a circle = 2 π × radius
(x) Area of a circle = π× (radius)2
20250 2
Therefore, area of the field = m
15
Notes
= 1350 m2 ...(2)
From (1) and (2), we have:
3x 2
= 1350
2
1350 × 2
x2 = = 900 = (30 )
2
or
3
or x = 30
Hence, corresponding altitutde is 30 m and the base is 3 × 30 m i.e., 90 m.
Example 20.5: Find the area of a rhombus whose diagonals are of lengths 16 cm and
12 cm.
1 1
Solution: Area of the rhombus = product of its diagonals = × 16 × 12 cm2
2 2
= 96 cm2
Example 20.6: Length of the two parallel sides of a trapezium are 20 cm and 12 cm and
the distance between them is 5 cm. Find the area of the trapezium.
1
Solution: Area of a trapezium = (sum of the two parallel sides)×distance between them
2
1
= (20 + 12) × 5 cm2 = 80 cm2
2
1
Area of a triangle = base × corresponding altitude
2
However, sometimes we are not given the altitude (height) corresponding to the given
base of a triangle. Instead of that we are given the three sides of the triangle. In this case
also, we can find the height (or altitude) corresponding to a side and calculate its area. Let
us explain it through an example.
Example 20.7: Find the area of the triangle ABC, whose sides AB, BC and CA are
respectively 5 cm, 6 cm and 7 cm.
Solution: Draw AD ⊥ BC as shown in Fig. 20.1.
Let BD = x cm
A
So, CD = (6 – x) cm
7
Now, from right triangle ABD, we have: 5
AB2 = BD2 + AD2 (Pythagoras Theorem)
i.e. 25 = x2 + AD2 ...(1) B D C
6
Similarly, from right triangle ACD, we have:
Fig. 20.1
2 2 2
AC = CD + AD
i.e. 49 = (6 – x)2 + AD2 ...(2)
From (1) and (2), we have:
49 – 25 = (6 – x)2 – x2
i.e. AD2 = 24 or AD = 24 = 2 6 cm
1 1
Thus, area of ΔABC = BC × AD = × 6 × 2 6 cm2 = 6 6 cm2
2 2
You must have observed that the process involved in the solution of the above example is
lengthy. To help us in this matter, a formula for finding the area of a triangle with three given
sides was provided by a Greek mathematician Heron (75 B.C. to 10 B.C.). It is as
follows:
a+b+c
where, a, b and c are the three sides of the triangle and s = . This formula can be
2
proved on similar lines as in Example 20.7 by taking a, b and c for 6, 7 and 5 respectively.
Let us find the area of the triangle of Example 20.7 using this formula.
Here, a = 6 cm, b = 7 cm and c = 5 cm
6+7+5
So, s = = 9 cm
2
9 × 3 × 2 × 3 cm
2
=
Let us take some more examples to illustrate the use of this formula.
Example 20.8: The sides of a triangular field are 165 m, 154 m and 143 m. Find the area
of the field.
= 11 × 11 × 3 × 7 × 2 × 2 m2 = 10164 m2
Example 20.9: Find the area of a trapezium whose parallel sides are of lengths 11 cm
amd 25 cm and whose non-parallel sides are of lengths 15 cm and 13 cm.
Solution: Let ABCD be the trapezium in which AB = 11 cm, CD = 25 cm, AD = 15 cm
and BC =13 cm (See Fig. 20.2)
Through B, we draw a line parallel to AD to intersect DC at E. Draw BF ⊥ DC.
Now, clearly BE = AD = 15 cm
A B
BC = 13 cm (given)
and EC = (25 – 11) cm = 14 cm
15 + 13 + 14
So, for ΔBEC, s = cm = 21 cm
2 D E F C
Fig. 20.2
Therefore area of ΔBEC = s(s − a )(s − b)(s − c )
= 21× 6 × 8 × 7 cm2
= 7 × 3 × 4 cm2 = 84 cm2 ...(1)
1
Again, area of ΔBEC = EC × BF
2
1
= × 14 × BF ...(2)
2
So, from (1) and (2), we have:
1
× 14 × BF = 84
2
84
i.e., BF = cm = 12 cm
7
= 496 m2 M L
H E
Example 20.11: There are two rectangular paths in G F
O N
the middle of a park as shown in Fig. 20.4. Find the
cost of paving the paths with concrete at the rate of S R C
D
` 15 per m2. It is given that AB = CD = 50 m,
Fig. 20.4
AD = BC = 40 m and EF = PQ = 2.5 m.
= ` 3281.25
Example 20.12: Find the area of the figure ABCDEFG (See Fig. 20.5) in which ABCG
is a rectangle, AB = 3 cm, BC = 5 cm, GF = 2.5 cm = DE = CF., CD = 3.5 cm, EF = 4.5
cm, and CD || EF.
Solution: Required area = area of rectangle ABCG + area of isosceles triangle FGC
+ area of trapezium DCEF ...(1)
Now, area of rectangle ABCG = l × b = 5 × 3 cm2 = 15 cm2 ...(2)
For area of ΔFGC, draw FM ⊥ CG.
As FG = FC (given), therefore
A B
M is the mid point of GC.
3
That is, GM = = 1.5 cm
2
Now, from ΔGMF,
G M C D
GF2 = FM2 + GM2
or (2.5)2 = FM2 + (1.5)2
or FM2 = (2.5)2 – (1.5)2 = 4 F E
So, FM = 2, i.e., length of FM = 2 cm Fig. 20.5
1
So, area of ΔFGC = GC × FM
2
1
= × 3 × 2 cm2 = 3 cm2 ...(3)
2
1
Also, area of trapezium CDEF = (sum of the parallel sides) × distance between them
2
1
= (3.5 + 4.5) × 2 cm2
2
1
= × 8 × 2 cm2 = 8 cm2 ...(4)
2
= 3.14 × 30 × 2 = 188.4 m2
So, cost of paving the bricks at ` 24 per m2
Fig. 20.9
= ` 24 × 188.4
= ` 4521.60
Notes
circle. Recall that a part of a circular region enclosed
between two radii of the corresponding circle is called
.
Q
and OAQB is called the major sector of the circle Fig. 20.10
(with major arc AQB).
Note: unless stated otherwise, by sector, we shall mean a minor sector.
(i) Perimeter of the sector: Clearly, perimeter of the sector OAPB is equal to OA +
OB + length of arc APB.
Let radius OA (or OB) be r, length of the arc APB be l and ∠AOB be θ.
We can find the length l of the arc APB as follows:
We know that circumference of the circle = 2 πr
Now, for total angle 360o at the centre, length = 2πr
2πr
So, for angle θ, length l = ×θ
360 o
πrθ
or l= ...(1)
180o
Thus, perimeter of the sector OAPB = OA + OB + l
πrθ πrθ
=r+r+ o = 2 r +
180 180o
(ii) Area of the sector
Area of the circle = πr2
Now, for total angle 360o, area = πr2
πr 2
So, for angle θ, area = ×θ
360o
Perimeter = 2r +
(
πr 360 o − θ )
180 o
πr 2
and area =
360 o
(
× 360 o − θ )
Example 20.15: Find the perimeter and area of the sector of a circle of radius 9 cm with
central angle 35o.
πrθ
Solution: Perimeter of the sector = 2r +
180 o
⎛ 22 9 × 35o ⎞
⎜
=⎜ 2 × 9 + × ⎟ cm
⎝ 7 180o ⎟⎠
⎛ 11× 1 ⎞ 47
= ⎜18 + ⎟ cm = cm
⎝ 2 ⎠ 2
πr 2 × θ
Area of the sector =
360o
⎛ 22 81× 35o ⎞
= ⎜⎜ × ⎟ 2
o ⎟ cm
⎝ 7 360 ⎠
⎛ 11× 9 ⎞ 2 99
=⎜ ⎟ cm = cm 2
⎝ 4 ⎠ 4
Example 20.16: Find the perimeter and area of the sector of a circle of radius 6 cm and
length of the arc of the sector as 22 cm.
Solution: Perimeter of the sector = 2r + length of the arc
= (2 × 6 + 22) cm = 34 cm
For area, let us first find the central angle θ.
πrθ
So, = 22
180 o
Notes 180 o × 7
or θ= = 210o
6
πr 2θ
So, area of the sector =
360 o
22 36 × 210o
= ×
7 360o
= 66 cm2
Alternate method for area:
Circumference of the circle = 2πr
22
= 2× × 6 cm
7
22
and area of the circle = πr2 = × 6 × 6 cm2
7
22 22
For length 2 × × 6 cm, area = × 6 × 6 cm2
7 7
22 6 × 6 × 7 × 22
So, for length 22 cm, area = × cm2
7 2 × 22 × 6
= 66 cm2
1 1
Also, ∠BOP = ∠COP = ∠BOC = × 120o = 60o
2 2
BP 3
Now, = sin∠BOP = sin60 o = [See Lessons 22-23]
OB 2
A
BP 3
i.e., =
3.5 2
O
3.5 3
So, BC = 2 × cm = 3.5 3 cm
2 C
B P
3
Therefore, area of ΔABC = BC 2
4 Fig. 20.12
3
= × 3.5 × 3.5 × 3 cm2
4
Now, area of the design = area of the circle – area of ΔABC
3
= (3.14 × 3.5 × 3.5 – × 3.5 × 3.5 × 3) cm2
4
Notes ⎛ 7.46 ⎞
= 12.25⎜ ⎟ cm2 = 12.25 × 1.865 cm2
⎝ 4 ⎠
Therefore, cost of making the design at ` 0.50 per cm2
= ` 12.25 × 1.865 × 0.50 = ` 114.23 (approx)
Example 20.18: On a square shaped handkerchief,
nine circular designs, each of radius 7 cm, are made
as shown in Fig. 20.13. Find the area of the remainig
portion of the handkerchief.
Solution: As radius of each circular design is 7 cm,
diameter of each will be 2 × 7 cm = 14 cm
So, side of the square handkerchief = 3 × 14 = 42 cm ...(1)
Therefore, area of the square = 42 × 42 cm2 Fig. 20.13
22
Also, area of a circle = πr2 = × 7 × 7 cm 2 = 154 cm2
7
So, area of 9 circles = 9 × 154 cm2 ...(2)
Therefore, from (1) and (2), area of the remaining portion
= (42 × 42 – 9 × 154) cm2
= (1764 – 1386) cm2 = 378 cm2
10 cm
LET US SUM UP
• Perimeter of a rectangle = 2 (length + breadth)
Notes
• Area of a rectangle = length × breadth
• Perimeter of a square = 4 × side
• Area of a square = (side)2
• Area of a parallelogram = base × corresponding altitude
1
• Area of a triangle = base × corresponding altitude
2
and also s(s − a )(s − b)(s − c ) , where a, b and c are the three sides of the triangle
a+b+c
and s = .
2
1
• Area of a rhombus = product of its diagonals
2
1
• Area of a trapezium = (sum of the two parallel sides) × distance between them
2
• Area of rectangular path = area of the outer rectangle – area of inner rectangle
• Area of cross paths in the middle = Sum of the areas of the two paths – area of the
common portion
• circumference of a circle of radius r = 2 πr
• Area of a circle of radius r = πr2
• Area of a circular path = Area of the outer circle – area of the inner circle
πrθ
• Length l of the arc of a sector of a circle of radius r with central angle θ is l =
180o
πrθ
• Perimeter of the sector a circle with radius r and central angle θ = 2r +
180o
πr 2θ
• Area of the sector of a circle with radius r and central and θ =
360 o
TERMINAL EXERCISE
1. The side of a square park is 37.5 m. Find its area.
2. The perimeter of a square is 480 cm. Find its area.
3. Find the time taken by a person in walking along the boundary of a square field of area
40 000 m2 at a speed of 4 km/h.
4. Length of a room is three times its breadth. If its breadth is 4.5 m, find the area of the
floor.
5. The length and breadth of a rectangle are in the ratio of 5 : 2 and its perimeter is 980
cm. Find the area of the rectangle.
6. Find the area of each of the following parallelograms:
(i) one side is 25 cm and corresponding altitude is 12 cm
(ii) Two adjacent sides are 13 cm and 14 cm and one diagonal is 15 cm.
7. The area of a rectangular field is 27000 m2 and its length and breadth are in the ratio
6:5. Find the cost of fencing the field by four rounds of barbed wire at the rate of ` 7
per 10 metre.
8. Find the area of each of the following trapeziums:
S. No. Lengths of parellel sides Distance between the parallel sides
(i) 30 cm and 20 cm 15 cm
(ii) 15.5 cm and 10.5 cm 7.5 cm
(iii) 15 cm and 45 cm 14.6 cm
(iv) 40 cm and 22 cm 12 cm
9. Find the area of a plot which is in the shape of a quadrilateral, one of whose diagonals
is 20 m and lengths of the perpendiculars from the opposite corners on it are of lengths
12 m and 18 m respectively.
10. Find the area of a field in the shape of a trapezium whose parallel sides are of lengths
48 m and 160 m and non-parallel sides of lengths 50 m and 78 m.
1.5 cm
1.5 cm
6 cm
Fig. 20.16
28. A farmer buys a circular field at the rate of ` 700 per m2 for ` 316800. Find the
perimeter of the field.
29. A horse is tied to a pole at a corner of a square field of side 12 m by a rope of length
3.5 m. Find the area of the part of the field in which the horse can graze.
30. Find the area of the quadrant of a circle whose circumference is 44 cm.
..
A P
Q
31. In Fig. 20.17, OAQB is a quadrant of a circle
of radius 7 cm and APB is a semicircle. Find
the area of the shaded region.
O B
Fig. 20.17
Fig. 20.19
Fig. 20.20
Fig. 20.21
In each of the questions 36 to 42, write the correct answer from the four given options:
36. The perimeter of a square of side a is
(A) a2 (B) 4a (C) 2a (D) 2a
37. The sides of a triangle are 15 cm, 20 cm, and 25 cm. Its area is
(A) 30 cm2 (B) 150 cm2 (C) 187.5 cm2 (D) 300 cm2
38. The base of an isosceles triangle is 8 cm and one of its equal sides is 5 cm. The
corresponding height of the triangle is
(A) 5 cm (B) 4 cm (C) 3 cm (D) 2 cm
39. If a is the side of an equilateral triangle, then its altitude is
3 2 3 3 3
(A) a (B) (C) a (D)
2 2a 2 2 2a
πr 2
(v) Area of a sector of circle of radius r and central angle 60o is .
6
(vi) Perimeter of a sector of circle of radius 5 cm and central angle 120o is 5 cm +
10π
cm
3
44. Fill in the blanks:
1
(i) Area of a rhombus = product of its ___________________
2
1
(ii) Area of a trapezium = (sum of its ________) × distance between ______
2
(iii) The ratio of the areas of two sectors of two circles of radii 4 cm and 8 cm and
central angles 100o and 50o respectively is __________
(iv) The ratio of the lengths of the arcs of two sectors of two circles of radii 10 cm and
5 cm and central angles 75o and 150o is _____________.
(v) Perimeter of a rhombus of diagonals 16 cm and 12 cm is __________
2. 15 2 cm
2. 36 3 cm2 ; 6 3 cm
20.3
1. 648 m2
2. 276 m2
3. 7225 m2
⎛ 5 ⎞
4. ⎜ 27 + 11 ⎟ cm2
⎝ 4 ⎠
20.4
1. 15 cm
2. 8750
3. 10.78 m2
20.5
1 154
1. Perimeter = 35 cm; Area = cm2
2 3
2. Perimeter = 23 cm, Area = 33 cm2
1
2. 4 × π × 5 – 10 × 10 cm2
2
Notes
2
= (50π – 100) cm2
21 Notes
In the previous lesson, you have studied about perimeters and areas of plane figures like
rectangles, squares, triangles, trapeziums, circles, sectors of circles, etc. These are called
plane figures because each of them lies wholly in a plane. However, most of the objects
that we come across in daily life do not wholly lie in a plane. Some of these objects are
bricks, balls, ice cream cones, drums, and so on. These are called solid objects or three
dimensional objects. The figures representing these solids are called three dimensional or
solid figures. Some common solid figures are cuboids, cubes, cylinders, cones and spheres.
In this lesson, we shall study about the surface areas and volumes of all these solids.
OBJECTIVES
After studying this lesson, you will be able to
• explain the meanings of surface area and volume of a solid figure,
• identify situations where there is a need of finding surface area and where there
is a need of finding volume of a solid figure;
• find the surface areas of cuboids, cubes, cylinders, cones spheres and hemispheres,
using their respective formulae;
• find the volumes of cuboids, cubes, cylinders, cones, spheres and hemispheres
using their respective formulae;
• solve some problems related to daily life situations involving surface areas and
volumes of above solid figures.
Notes
21.1 MEANINGS OF SURFACE AREA AND VOLUME
Look at the following objects given in Fig. 21.1.
Fig. 21.1
Geometrically, these objects are represented by three dimensional or solid figures as fol-
lows:
Objects Solid Figure
Bricks, Almirah Cuboid
Die, Tea packet Cube
Drum, powder tin Cylinder
Jocker’s cap, Icecream cone, Cone
Football, ball Sphere
Bowl. Hemisphere
You may recall that a rectangle is a figure made up of only its sides. You may also recall that
the sum of the lengths of all the sides of the rectangle is called its permeter and the measure
of the region enclosed by it is called its area. Similarly, the sum of the lengths of the three
sides of a triangle is called its permeters, while the measure of the region enclosed by the
triangle is called its area. In other words, the measure of the plane figure, i.e., the boundary
triangle or rectangle is called its perimeter, while the measure of the plane region enclosed
by the figure is called its area.
D C
or BE2 = l2 + b2 -(1)
h
Also, EC2 = BC2 + BE2 (As ∠CBE = 90o)
E F
or 2 2 2 2
EC = h + l + b [From (i)] b
A l B
So, EC = l 2 + b2 + h2. Fig. 21.4
3 cm
4 cm
5 cm
Fig. 21.5
By actually counting the unit cubes, you can see that this cuboid is made up of 60 unit
cubes.
So, its volume = 60 cubic cm or 60cm3 (Because volume of 1 unit cube, in this case, is
1 cm3)
Also, you can observe that length × breadth × height = 5×4×3 cm3
= 60 cm3
You can form some more cuboids by joining different number of unit cubes and find their
volumes by counting the unit cubes and then by the product of length, breadth and height.
Everytime, you wll find that
Volume of a cuboid = length × breadth × height
or volume of a cuboid = lbh
Further, as cube is a special case of cuboid in which l = b = h, we have;
volume of a cube of side a = a×a×a× =a3.
Example 21.2: Find the volume of a cuboidal stone slab of length 3m, breadth 2m and
thickness 25cm.
Solution : Here, l = 3m, b = 2m and
25 1
h = 25cm = = m
100 4
(Note that here we have thickness as the third dimension in place of height)
So, required volume = lbh
1 3
= 3×2× m = 1.5m3
4
Example 21.3 : Volume of a cube is 2197 cm3. Find its surface area and the diagonal.
Solution: Let the edge of the cube be a cm.
So, its volume = a3 cm3
Therefore, from the question, we have :
a3 = 2197
or a3 = 13×13×13
So, a = 13
i.e., edge of the cube = 13 cm
Thus, surface area of the cube is 1014 cm2 and its diagonal is 13 3 cm .
Example 21.4 : The length and breadth of a cuboidal tank are 5m and 4m respectively. If
it is full of water and contains 60 m3 of water, find the depth of the water in the tank.
Solution: let the depth be d metres
So, volume of water in the tank
= l×b×h
= 5×4×d m3
Thus, according to the question,
5×4 ×d = 60
60
or d= m = 3m
5× 4
So, it can be said that capacity of the tank is 60 × 1000 litre = 60 kilolitres.
Example 21.5 : A wooden box 1.5m long, 1.25 m broad, 65 cm deep and open at the
top is to be made. Assuming the thickness of the wood negligible, find the cost of the wood
required for making the box at the rate of ` 200 per m2.
Solution : Surface area of the wood required
= lb + 2bh + 2hl (Because the box is open at the top)
65 65
= (1.5×1.25 + 2×1.25 × +2× ×1.5)m2
100 100
162.5 195
= (1.875 + + ) m2
100 100
4.5 × 1000
= metres per second
60 × 60
4500
= metres per second
3600
5
= metres per second
4
Threfore, volume of the water running into the sea per second = volume of the cuboid
= l × b× h
5
= × 100 × 10 m3
4
= 1250 m3
Example 21.7: A tank 30m long, 20m wide and 12 m deep is dug in a rectangular field of
length 588 m and breadth 50m. The earth so dug out is spread evenly on the remaining
part of the field. Find the height of the field raised by it.
Solution: Volume of the earth dug out = volume of a cuboid of
dimensions 30 m × 20 m × 12 m
= 30 × 20 × 12 m3 = 7200 m3
Area of the remaining part of the field
= Area of the field - Area of the top surface of the tank
= 588 × 50 m2 – 30 × 20 m2
= 29400 m2 – 600 m2
= 28800 m2
Example 21.8: Length, breadth and height of a room are 7m, 4m and 3m respectively. It
1 1
has a door and a window of dimensions 2 m ×1 m and 1 m × 1 m respectively. Find
2 2
the cost of white washing the walls and ceiling of the room at the rate of ` 4 per m2.
Solution: Shape of the room is that of a cuboid.
Area to be white washed = Area of four walls
+ Area of the ceiling
– Area of the door – Area of the window.
Area of the four walls = l×h+b×h+l×h+b×h
= 2(l+b)×h
= 2(7+4)×3 m2 = 66m2
Area of the ceiling = l×b
= 7×4 m2 = 28m2
1 2 1
So, area to be white washed = 66 m2 + 28 m2 – 2 × 1 m – 1 × 1m2
2 2
3 2
= 94 m2 – 3 m2 – m
2
(188 – 6 – 3) 2
= m
2
179 2
= m
2
Therefore, cost of white-washing at the rate of ` 4 per m2
179
=`4× = ` 358
2
Note: You can directly use the relation area of four walls = 2 (l + b) × h as a formula]
h h
h
(i) (ii)
Fig. 21.7
So, curved surface area of the cylinder
= area of the rectangle
= 2 π r × h = 2 π rh.
In case, the cylinder is closed at both the ends, then the total surface area of the cylinder
= 2 π rh + 2 π r2
= 2 π r (r + h)
Volume
In the case of a cuboid, we have seen that its volume = l × b × h
= area of the base × height
Extending this rule for a right circular cylinder (assuming it to be the sum of the infininte
number of small cuboids), we get : Volume of a right circular cylinder
= Area of the base × height
= π r2 × h
= π r2h
We now take some examples to illustrate the use of these formulae; (In all the problems in
this lesson, we shall take the value of π = 22/7, unless stated otherwise)
Example 21.9: The radius and height of a right circular cylinder are 7cm and 10cm
respectively. Find its
(i) curved surface area
22
= 2× × 7 × 10 cm 2 = 440 cm 2
7
(ii) total surface area = 2 π rh + 2 π r2
22 22
= (2 × × 7 × 10 + 2 × × 7 × 7) cm 2
7 7
= 440 cm2 + 308 cm2 = 748 cm2
(iii) volume = π r2h
22
= × 7 × 7 × 10cm 3
7
= 1540 cm3
Example 21.10: A hollow cylindrical metallic pipe is open at both the ends and its exter-
nal diameter is 12 cm. If the length of the pipe is 70 cm and the thickness of the metal used
is 1 cm, find the volume of the metal used for making the pipe.
Solution: Here, external radius of the pipe
12
= cm = 6cm
2
Therefore, internal radius = (6–1) = 5 cm (As thickness of metal = 1 cm)
Note that here virtually two cylinders have been formed and the volume of the metal used
in making the pipe.
= Volume of the external cylinder – Volume of the internal cylinder
= π r12h – π r22h (where r1 and r2 are the external and internal radii and h is the
length of each cylinder .
⎛ 22 22 ⎞
= ⎜ × 6 × 6 × 70 – × 5 × 5 × 70 ⎟cm 3
⎝ 7 7 ⎠
22
= 2πrh = 2 × × 35 × 100 cm2 (r = 35 cm, h = 1 m = 100 cm)
7
= 22000 cm2
22000
= m2
100 × 100
(since 100 cm = 1 m, so 100 cm × 100 cm = 1 m × 1 m)
= 2.2 m2
Therefore, area of the playground levelled in 200 revolutions = 2.2 × 200 m2 = 440 m2
Hence, cost of levelling at the rate of ` 3 per m2 = ` 3 × 440 = ` 1320.
Example 21.12: A metallic solid of volume 1 m3 is melted and drawn into the form of a
wire of diameter 3.5 mm. Find the length of the wire so drawn.
Solution: Let the length of the wire be x mm
You can observe that wire is of the shape of a right circular cylinder.
Its diameter = 3.5 mm
3.5 35 7
So, its radius = mm = = mm
2 20 4
Also, length of wire will be treated as the height of the cylinder.
So, volume of the cylinder = πr2h
22 7 7
= × × × x mm3
7 4 4
But the wire has been drawn from the metal of volume 1 m3
22 7 7 x
Therefore, × × × = 1 (since 1 m = 1000 mm)
7 4 4 1000000000
1 × 7 × 4 × 4 × 1000000000
or x = mm.
22 × 7 × 7
16000000000
=
154
Notes 16000000000
Thus, length of the wire = mm
154
16000000000
= m
154000
16000000
= m = 103896 m (approx)
154
A
21.4 RIGHT CIRCULAR CONE
Let us rotate a right triangle ABC right angled at B about one
of its side AB containing the right angle. The solid generated as
a result of this rotation is called a right circular cone (see
Fig. 21.8). In daily life, we come across many objects of this C D
shape, such as Joker’s cap, tent, ice cream cones, etc. B
Fig. 21.8
or l = r 2 + h 2 , where r, h and l are respectively the base radius, height and slant
height of the cone.
You can also observe that surface formed by the base of the cone is flat and the remaining
surface of the cone is curved.
A
Surface Area
Let us take a hollow right circular cone of radius r and height h l l
and cut it along its slant height. Now spread it on a piece of
paper. You obtain a sector of a circle of radius l and its arc
length is equal to 2πr (Fig. 21.9). B C
2πr
= × πl 2 = πrl
2πl
Clearly, curved surface of the cone = Area of the sector
= πrl
If the area of the base is added to the above, then it becomes the total surface area.
So, total surface area of the cone = πrl + πr2
= πr(l + r)
Volume
Take a right circular cylinder and a right circular cone of the same base radius and same
height. Now, fill the cone with sand (or water) and pour it in to the cylinder. Repeat the
process three times. You will observe that the cylinder is completely filled with the sand (or
water). It shows that volume of a cone with radius r and height h is one third the volume of
the cylinder with radius r and height h.
1
So, volume of a cone = volume of the cylinder
3
1
= πr2h
3
Notes Now, let us consider some examples to illustrate the use of these formulae.
Example 21.13: The base radius and height of a right circular cone is 7 cm and 24 cm.
Find its curved surface area, total surface area and volume.
Solution: Here, r = 7 cm and h = 24 cm.
= 7 × 7 + 24 × 24 cm
= 49 + 576 cm = 25 cm
Thus, curved surface area = πrl
22
= × 7 × 25 cm2 = 550 cm2
7
Total surface area = πrl + πr2
22
= (550 + × 49) cm2
7
= (550 + 154) cm2 = 704 cm2
1 2 1 22
Volume = πr h = × × 49 × 24 cm3
3 3 7
= 1232 cm3
Example 21.14: A conical tent is 6 m high and its base radius is 8 m. Find the cost of the
canvas required to make the tent at the rate of ` 120 per m2 (Use π = 3.14)
Solution: Let the slant height of the tent be x metres.
l = 36 + 64 = 100
or l = 10
Thus, slant height of the tent is 10 m.
So, its curved surface area = πrl
= 3.14 × 8 × 10 cm2 = 251.2 cm2
21.5 SPHERE
Let us rotate a semicircle about its diameter. The solid so P
generated with this rotation is called a sphere. It can also be r
defined as follows:
O
The locus of a point which moves in space in such a way that
its distance from a fixed point remains the same is called a
sphere. The fixed point is called the centre of the sphere and
the same distance is called the radius of the sphere (Fig. Fig. 21.10
21.10). A football, cricket ball, a marble etc. are examples
of spheres that we come across in daily life.
Hemisphere
If a sphere is cut into two equal parts by a plane passing
through its centre, then each part is called a hemisphere
(Fig. 21.11).
Fig. 21.11
Surface Areas of sphere and hemisphere
Let us take a spherical rubber (or wooden) ball and cut it into equal parts (hemisphere)
[See Fig. 21.12(i), Let the radius of the ball be r. Now, put a pin (or a nail) at the top of
the ball. starting from this point, wrap a string in a spiral form till the upper hemisphere is
Notes
(i) (ii)
(iii)
Fig. 21.12
Now draw a circle of radius r (i.e. the same radius as that of the ball and cover it with a
similar string starting from the centre of the circle [See Fig. 21.12 (iii)]. Measure the length
of the string used to cover the circle. What do you observe? You will observe that length
of the string used to cover the hemisphere is twice the length of the string used to
cover the circle.
Since the width of the two strings is the same, therefore
surface area of the hemisphere = 2 × area of the circle
= 2 πr2 (Area of the circle is πr2)
So, surface area of the sphere = 2 × 2πr2 = 4πr2
Thus, we have:
Surface area of a sphere = 4πr2
Curved surface area of a solid hemisphere = 2πr2 + πr2 = 3πr2
Where r is the radius of the sphere (hemisphere)
Volumes of Sphere and Hemisphere
Take a hollow hemisphere and a hollow right circular cone of the same base radius and
same height (say r). Now fill the cone with sand (or water) and pour it into the hemisphere.
Repeat the process two times. You will observe that hemisphere is completely filled with
the sand (or water). It shows that volume of a hemisphere of radius r is twice the volume
1 2
So, volume of the hemisphere = 2 × πr h
3
Notes
2
= × πr2 × r (Because h = r)
3
2
= × πr3
3
Therefore, volume of the sphere of radius r
2 3 4 3
=2× πr = πr
3 3
Thus, we have:
4 3
Volume of a sphere = πr
3
2 3
and volume of a hemisphere = πr ,
3
where r is the radius of the sphere (or hemisphere)
Let us illustrate the use of these formulae through some examples:
Example 21.15: Find the surface area and volume of a sphere of diameter 21 cm.
21
Solution: Radius of the sphere = cm
2
So, its surface area = 4πr2
22 21 21
=4× × × cm2
7 2 2
= 1386 cm2
4 3
Its volume = πr
3
4 22 21 21 21
= × × × × cm3 = 4851 cm3
3 7 2 2 2
2 22 3
or × r = 2425.5
3 7
21
So, r = , i.e. radius = 10.5 cm.
2
22 21 21
Now surface area of bowl = curved surface area = 2πr2 = 2 × × × cm2
7 2 2
= 693 cm2
Note: As the bowl (hemisphere) is open at the top, therefore area of the top, i.e., πr2 will
not be included in its surface area.
LET US SUM UP
• The objects or figures that do not wholly lie in a plane are called solid (or three
dimensional) objects or figures.
• Surface area of a cuboid = 2(lb + bh + hl) and volume of cuboid = lbh, where l, b
and h are respectively length, breadth and height of the cuboid.
4 3
• Surface area of sphere = 4πr2 and its volume = πr , where r is the radius of the
3
sphere.
• Curved surface area of a hemisphere of radius r = 2πr2 ; its total surface area = 3πr2
2 3
and its volume = πr
3
TERMINAL EXERCISE
1. Fill in the blanks:
(i) Surface area of a cuboid of length l, breadth b and height h = _________
(ii) Diagonal of the cuboid of length l, breadth b and height h = ___________
(iii) Volume of the cube of side a = __________
1
(C) times the volume of the cone (D) 3 times the volume of the cone.
3
3. If the surface area of a cube is 96 cm2, then find its volume.
4. Find the surface area and volume of a cuboid of length 3m, breadth 2.5 m and height
1.5 m.
5. Find the surface area and volume of a cube of edge 1.6 cm.
6. Find the length of the diagonal of a cuboid of dimensions 6 cm × 8 cm × 10 cm.
7. Find the length of the diagonal of a cube of edge 8 cm.
8. Areas of the three adjecent faces of cuboid are A, B and C square units respectively
and its volume is V cubic units. Prove that V2 = ABC.
9. Find the total surface area of a hollow cylindrical pipe open at the ends if its height is
10 cm, external diameter 10 cm and thickness 12 cm (use π = 3.14).
10. Find the slant height of a cone whose volume is 12936 cm3 and radius of the base is
21 cm. Also, find its total surface area.
11. A well of radius 5.6 m and depth 20 m is dug in a rectangular field of dimensions
150 m × 70 m and the earth dug out from it is evenly spread on the remaining part of
the field. Find the height by which the field is raised.
12. Find the radius and surface area of a sphere whose volume is 606.375 m3.
9. 61 m
21.2
1 2
1. 44 m2; 201 m ; 110 m3 2. 880 cm2
7
3. 80850 litres 4. 1386 cm3
5. 4 cm 6. 401.92 cm2
21.3
2
1. 2464 cm2; 11498 cm3 2. 21 cm, 5544 cm2
3
1
3. (i) 9928 cm2 (ii) 14892 cm2 (iii) 92661 cm3
3
4. 64
6. 10 2 cm 7. 8 3 cm 8. [Hint: A = l × h; B = b × h; and C = h × l]
Secondary Course
Mathematics
Notes
Practice Work-Mensuration
Instructions:
(A) 8 cm
(B) 4 cm
(C) 2 cm
(D) 16 cm
2. The sides of a triangle are in the ratio 3 : 5 : 7. If the perimeter of the triangle is 60 cm,
then the area of the triangle is 1
(A) 60 3 cm2
(B) 30 3 cm3
(A)5 cm
(B) 6 cm
(C) 8 cm
(D) 10 cm
4. A cuboid having surface areas of three adjacent faces as a, b, c has the volume 1
(A) 3
abc
(B) abc
(C) abc
(D) a3b3c3
(A) 38.5 m2
(B) 77 m2
(C) 115.5 m2
(D) 154 m2
6. The parallel sides of a trapezium are 20 metres and 16 metres and the distance between
them is 11m. Find its area. 2
7. A path 3 metres wide runs around a circular park whose radius is 9 metres. Find the
area of the path. 2
8. The radii of two right circular cylinders are in the ratio 4 : 5 and their heights are in the
ratio 5 : 3. Find the ratio of their volumes. 2
9. The circumference of the base of a 9 metre high wooden solid cone is 44 m. Find the
volume of the cone. 2
11. The radius and height of a right circular cone are in the ratio 5 : 12. If its volume is
314 m3, find its slant height. (Use π = 3.14) 4
Notes
12. A field is 200 m long and 75 m broad. A tank 40 m long, 20 m broad and 10 m deep
is dug in the field and the earth taken out of it, is spread evenly over the field. How
much is the level of field raised? 6
22 Notes
INTRODUCTION TO TRIGONOMETRY
Study of triangles occupies important place in Mathematics. Triangle being the bounded
figure with minimum number of sides serve the purpose of building blocks for study of any
figure bounded by straight lines. Right angled triangles get easy link with study of circles as
well.
In Geometry, we have studied triangles where most of the results about triangles are given
in the form of statements. Here in trigonometry, the approach is quite different, easy and
crisp. Most of the results, here, are the form of formulas. In Trigonometry, the main focus
is study of right angled triangle. Let us consider some situations, where we can observe the
formation of right triangles.
Have you seen a tall coconut tree? On seeing the tree, a question about its height comes to
the mind. Can you find out the height of the coconut tree without actually measuring it? If
you look up at the top of the tree, a right triangle can be imagined between your eye, the
top of the tree, a horizontal line passing through the point of your eye and a vertical line
from the top of the tree to the horizontal line.
Let us take another example.
Suppose you are flying a kite. When the kite is in the sky, can you find its height? Again a
right triangle can be imagined to form between the kite, your eye, a horizontal line passing
through the point of your eye, and a vertical line from the point on the kite to the horizontal
line.
(aeroplane) A (aeroplane) B
Let us consider another situation where a person
is standing on the bank of a river and observing a
temple on the other bank of the river. Can you
find the width of the river if the height of the temple
is given? In this case also you can imagine a right
triangle.
Finally suppose you are standing on the roof of
your house and suddenly you find an aeroplane in P Q
the sky. When you look at it, again a right triangle O (observer)
can be imagined. You find the aeroplane moving Fig. 22.1
OBJECTIVES
After studying this lesson, you will be able to
• write the trigonometric ratios of an acute angle of right triangle;
• find the sides and angles of a right triangle when some of its sides and
trigonometric ratios are known;
• write the relationships amongst trigonometric ratios;
• establish the trigonometric identities;
• solve problems based on trigonometric ratios and identities;
• find trigonometric ratios of complementary angles and solve problems based on
these.
Side opposite to ∠A
e
us
oten
p
Hy
A Side adjacent to ∠A B
Fig. 22.2
Again, if we consider acute ∠C, then side AB is side opposite to ∠C and side BC is
adjacent to ∠C.
C
Side adjacent to ∠C
e
us
oten
p
Hy
A Side opposite to ∠C B
Fig. 22.3
We now define certain ratios involving the sides of a right triangle, called trigonometric
ratios.
The trigonometric ratios of ∠A in right angled ΔABC are defined as:
side opposite to ∠A BC
(i) sine A = =
Hypotenuse AC
side adjacent to ∠A AB
(ii) cosine A = =
Hypotenuse AC
side opposite to ∠A BC
(iii) tangent A = =
side adjacent to ∠A AB
Notes Hypotenuse AC
(iv) cosecant A = =
side opposite to ∠A BC
Hypotenuse AC
(v) secant A = =
side adjacent to ∠A AB
side adjacent to ∠A AB
(vi) cotangent A = =
side opposite to ∠A BC
The above trigonometric ratios are abbreviated as sin A, cos A, tan A, cosec A, sec A and
cot A respectively. Trigonometric ratios are abbreviated as t-ratios.
If we write ∠A = θ, then the above results are
BC AB BC
sin θ = , cos θ = , tan θ =
AC AC AB
AC AC AB
cosec θ = , sec θ = and cot θ =
BC AB BC
Note: Observe here that sin θ and cosec θ are reciprocals of each other. Similarly cot θ
and sec θ are respectively reciprocals of tan θ and cos θ.
Remarks
C
Thus in right ΔABC,
α
AB = 4cm, BC = 3cm and cm
5 3 cm
AC = 5cm, then
θ
BC 3 A B
sin θ = = 4 cm
AC 5
Fig. 22.4
AB 4
cos θ = =
AC 5
BC 3
tan θ = =
AB 4
AC 5
cosec θ = =
BC 3
AC 5
sec θ = =
AB 4
AB 4 Notes
and cot θ = =
BC 3
In the above figure, if we take angle C = α, then
side opposite to ∠α AB 4
sin α = = =
Hypotenuse AC 5
side adjacent to ∠α BC 3
cos α = = =
Hypotenuse AC 5
side opposite to ∠α AB 4
tan α = = =
side adjacent to ∠α BC 3
Hypotenuse AC 5
cosec α = = =
side opposite to ∠α AB 4
Hypotenuse AC 5
sec α = = =
side adjacent to ∠α BC 3
side adjacent to ∠α BC 3
and cot α = = =
side opposite to ∠α AB 4
Remarks :
1. Sin A or sin θ is one symbol and sin cannot be separated from A or θ. It is not equal to
sin × θ. The same applies to other trigonometric ratios.
2. Every t-ratio is a real number.
3. For convenience, we use notations sin2θ, cos2θ, tan2θ for (sinθ)2, (cosθ)2, and (tanθ)2
respectively. We apply the similar notation for higher powers of trigonometric ratios.
4. We have restricted ourselves to t-ratios when A or θ is an acute angle.
Now the question arises: “Does the value of a t-ratio remains the same for the same
angle of different right triangles?.” To get the answer, let us consider a right triangle ABC,
right angled at B. Let P be any point on the hypotenuse AC.
Let PQ ⊥ AB
BC
sin A = ----(i)
Notes AC
and in right ΔAQP,
PQ
sin A = ----(ii) R
AP
Now in ΔAQP and ΔABC,
C
∠Q = ∠B ----(Each = 90°)
and ∠A = ∠A ----(Common) P
∴ ΔAQP ~ ΔABC
AP QP AQ
∴ = = Q B S
AC BC AB A
Fig. 22.5
BC PQ
or = ----(iii)
AC AP
From (i), (ii), and (iii), we find that sin A has the same value in both the triangles.
AB AQ BC PQ
Similarly, we have cos A = = and tan A = =
AC AP AB AQ
Hypotenuse AC 13
cosec C = = =
side opposite to ∠C AB 5
Hypotenuse AC 13 A B
and sec C = = = 5 cm
side adjacent to ∠C BC 12
Fig. 22.6
516 Mathematics Secondary Course
Introduction to Trigonometry MODULE - 5
Trigonometry
Example 22.2 : Find the value of sin θ, cot θ and Sec θ from Fig. 22.7.
A
θ
Notes
21 cm 29
cm
C
B 20 cm
Fig. 22.7
Solution:
side opposite to ∠θ BC 20
sin θ = = =
Hypotenuse AC 29
side adjacent t o ∠θ AB 21
cot θ = = =
side opposite to ∠θ BC 20
Hypotenuse AC 29
and sec θ = = =
side adjacent t o ∠θ AB 21
Solution:
40 cm
cm
side adjacent to ∠C BC 40
cos C = = =
41
Now
Hypotenuse AC 41
side adjacent to ∠C BC 40 A B
and cot C = = = 9 cm
side opposite to ∠C AB 9
Fig. 22.8
With reference to ∠A, side adjacent to A is AB and side opposite to A is BC.
side opposite to ∠A BC 40
∴ tan A = = =
side adjacent to ∠A AB 9
Hypotenuse AC 41
and cosec A = = =
side opposite to ∠A BC 40
Notes Example 22.4 : In Fig. 22.9, ΔABC is right angled at B, ∠A = ∠C, AC = 2 cm and
AB = 1 cm. Find the values of sin C, cos C and tan C.
Solution: In ΔABC, ∠A = ∠C A
∴ BC = AB = 1 cm (Given)
1 cm
side opposite to ∠C AB 1
∴ sin C = = =
Hypotenuse AC 2
C B
side adjacent to ∠C BC 1 Fig. 22.9
cos C = = =
Hypotenuse AC 2
side opposite to ∠C AB 1
and tan C = = = =1
side adjacent to ∠C BC 1
Remark: In the above example, we have ∠A = ∠C and ∠B = 90°
∴ ∠A = ∠C = 45°,
1
∴ We have sin 45° = cos 45° =
2
and tan 45o = 1
(ii) tan A =
b
b A C
(iii) cot A = b
a
Fig. 22.10
a
(iv) cot A =
b
side opposite to ∠A BC a
Solution: Here tan A = = =
side adjacent to ∠A AC b
side adjacent to ∠A b
and cot A = =
side opposite to ∠A a
b Notes
Hence the result (iii) i.e. cot A = is true.
a
1. In each of the following figures, ΔABC is a right triangle, right angled at B. Find all
the trigonometric ratios of θ .
B 4 cm
A C
θ
13
cm
3 cm
5 cm
m
5c
(i) θ (ii)
B 12 cm C
A
A
A
25 θ 10 cm
cm
24 cm
(iii) (iv)
6 cm
C
8 cm
θ C B
B
7 cm
Fig. 22.11
2. In ΔABC, ∠ B = 90°, BC = 5cm, AB = 4cm, and AC = 41 cm, find the value of
sin A, cos A, and tan A.
3. In ΔABC right angled at B, if AB = 40 cm, BC = 9 cm and AC = 41 cm, find the
values of sin C, cot C, cos, A and cot A.
4. In ΔABC, ∠ B = 90°. If AB = BC = 2cm and AC = 2 2 cm, find the value of
sec C, cosec C, and cot C. B
5. In Fig. 22.12, ΔABC is right angled at A. Which
of the following is true? cm
5 cm
13
13 12
(i) cot C = (ii) cot C =
12 13 C A
12 cm
5 12
(iii) cot C = (iv) cot C = Fig. 22.12
12 5
b
c b
(iii) cosec A = (iv) cosec A = .
b a B C
a
Fig. 22.13
5 cm
triangle, right angled at Q. If PQ = 5 cm and QR =
12 cm, find the values of sin R, cos R and tan R.
Solution: We shall find the third side by using R 12 cm Q
Pythagoras Theorem.
Fig. 22.14
Q ΔPQR is a right angled triangle at Q.
∴ PR = PQ 2 + QR 2 (Pythagoras Theorem)
= 52 + 12 2 cm
= 25 + 144 cm
= 169 or 13 cm
side opposite to ∠R PQ 5
sin R = = =
Hypotenuse PR 13
side adjacent to ∠R QR 12
cos R = = =
Hypotenuse PR 13
side opposite to ∠R 5
and tan R = =
side adjacent to ∠R 12
∴ QR = PR 2 − PQ 2
cm
25
7 cm
= 252 − 7 2 cm
= 625 − 49 cm θ
R Q
= 576 cm Fig. 22.15
= 24 cm
PQ 7
∴ tan θ = =
QR 24
PR 25
cosec θ = =
PQ 7
PR 25
and sec θ = =
QR 24
Example 22.8 : In ΔABC, ∠ B = 90°. If AB = 4 cm and BC = 3 cm, find the values of
sin C, cos C, cot C, tan A, sec A and cosec A. Comment on the values of tan A and cot C.
Also find the value of tan A – cot C.
A
Solution: By Pythagoras Theorem, in ΔABC,
AC = AB2 + BC2
4 cm
= 4 2 + 32 cm
= 25 cm
= 5 cm C 3 cm B
AB 4 Fig. 22.16
Now sin C = =
AC 5
BC 3
cos C = =
AC 5
Notes BC 3
cot C = =
AB 4
BC 3
tan A = =
AB 4
AC 5
sec A = =
AB 4
AC 5
and cosec A = =
BC 3
The value of tan A and cot C are equal P
∴ tan A – cot C = 0.
Example 22.9: In Fig. 22.17, PQR is right triangle at R. cm
If PQ = 13cm and QR = 5cm, which of the following is 13
true?
Q 5 cm R
17 17
(i) sin Q + cos Q = (ii) sin Q – cos Q =
13 13 Fig. 22.17
17 17
(iii) sin Q + sec Q = (iv) tan Q + cot Q =
13 13
PR 12 QR 5
∴ sin Q = = and cos Q = =
PQ 13 PQ 13
12 5 17
∴ sin Q + cos Q = + =
13 13 13
17
Hence statement (i) i.e. sin Q + cos Q = is true.
13
5
(i) sin F =
12
5m
12
(ii) sin F =
5
F 12 m E
5
(iii) sin F = Fig. 22.18
13
12
(iv) sin F =
13
12
Now sin θ = implies that sides AB and AC are in
13
the ratio 12 : 13. C θ
B
Fig. 22.19
= (13k )2 − (12k )2
= 169k 2 − 144k 2
= 25k 2 = 5 k
Now we can find all othe t-ratios.
BC 5k 5
cos θ = = =
AC 13k 13
AB 12k 12
tan θ = = =
BC 5k 5
AC 13k 13
cosec θ = = =
AB 12k 12
AC 13k 13
sec θ = = =
BC 5k 5
BC 5k 5
and cot θ = = =
AB 12k 12
The method discussed above gives the following steps for the solution.
Steps to be followed for finding the t-ratios when one t-ratio is given.
7
Example 22.10.: If cos θ = , find the values of sin θ and tan θ.
25
Solution : Draw a right-angled ΔABC in which ∠ B = 90° and ∠ C = θ.
adjacent side BC 7
cos θ = = = A
hypotenuse AC 25
Notes
Let BC = 7 k and AC = 25 k
Then by Pythagoras Theorem,
24 k
k
25
AB = AC2 − BC2
θ
= (25k ) − (7k )
2 2 C 7k B
Fig. 22.20
= 625k − 49k
2 2
= 576k 2 or 24 k
∴ In ΔABC,
AB 24k 24
sin θ = = =
AC 25k 25
AB 24k 24
and tan θ = = =
BC 7k 7
40 cos θ. sin θ
Example 22.11.: If cot θ = , find the value of .
9 sec θ
Solution. Let ABC be a right triangle, in which ∠ B = 90° and ∠ C = θ.
We know that
BC 40
cot θ = =
AB 9 A
9k
AC = BC2 + AB2
C θ
B
= (40k ) + (9k )
2 2 40 k
Fig. 22.21
= 1600k 2 + 81k 2
= 1681k 2 or 41 k
AB 9k 9
Notes Now sin θ = = =
AC 41k 41
BC 40k 40
cos θ = = =
AC 41k 41
AC 41k 41
and sec θ = = =
BC 40k 40
9 40
×
cos θ. sin θ 41 41
∴ = 41
sec θ
40
9 40 40
= × ×
41 41 41
14400
=
68921
1
Example 22.12.: In PQR, ∠ Q = 90° and tan R = . Then show that
3
sin P cos R + cos P sin R = 1
1
Solution: Let there be a right-triangle PQR, in which ∠ Q = 90° and tan R = .
3
We know that
P
PQ 1
tan R = =
QR 3
Let PQ = k and QR = 3k 2k 1k
R Q
Then, PR = PQ 2 + QR 2 3 k
= k2 + ( 3k )
2 Fig. 22.22
= k 2 + 3k 2
= 4k 2 or 2 k
Notes
side opposite to ∠P QR 3k 3
∴ sin P = = = =
Hypotenuse PR 2k 2
side adjacent t o ∠P 1k 1
cos P = = =
Hypotenuse 2k 2
side opposite to ∠R PQ 1k 1
sin R = = = =
Hypotenuse PR 2k 2
side adjacent to ∠R QR 3k 3
and cos R = = = =
Hypotenuse PR 2k 2
3 3 1 1
∴ sin P cos R + cos P sin R = . + .
2 2 2 2
3 1 4
= + =
4 4 4
=1
Example 22.13.: In ΔABC, ∠ B is right-angle. If AB = c, BC = a and AC = b, which of
the following is true?
2b
(i) cos C + sin A =
a A
b a
(ii) cos C + sin A = +
a b b
c
2a
(iii) cos C + sin A =
b C B
a
a c Fig. 22.23
(iv) cos C + sin A = +
b b
BC a
Solution: Here cos C = =
AC b
Notes BC a
and sin A = =
AC b
a a 2a
∴ cos C + sin A = + =
b b b
2a
∴ Statement (iii), i.e., cos C + sin A = is true.
b
20
1. If sin θ = , find the values of cos θ and tan θ.
29
24
2. If tan θ = , find the values of sin θ and cos θ.
7
7
3. If cos A = , find the values of sin A and tan A.
25
m
4. If cos θ = , find the values of cot θ and cosec θ.
n
4 cos θ . cot θ
5. If cos θ = , evaluate .
5 1 − sec 2θ
2
6. If cosec θ = , find the value of sin2 θ cos θ + tan2 θ.
3
5
7. If cot B = , then show that cosec2 B = 1 + cot2 B.
4
3
8. ΔABC is a right triangle with ∠C = 90o. If tan A = , find the values of sin B and
2
tan B.
1
9. If tan A = and tan B = 3 , then show that cos A cos B – sin A sin B = 0.
3
12 Notes
10. If cot A = , show that tan2A – sin2A = sin4A sec2A.
5
[Hint: Find the vlaues of tan A, sin A and sec A and substitute]
11. In Fig. 22.24, ΔABC is right-angled at vertex B. If AB = c, BC = a and CA = b,
which of the following is true?
A
b+c
(i) sin A + cos A =
a
b
a+c
c
(ii) sin A + cos A =
b
C B
a+b a
(iii) sin A + cos A =
c Fig. 22.24
a+b+c
(iv) sin A + cos A =
b
AB AB BC Fig. 22.25
Rewriting, tan θ = = ÷
BC AC AC
AB
AC sin θ
= BC =
cos θ
AC
sin θ
Thus, we see that tan θ =
cos θ
3 4 3
∴ sin θ = , cos θ = and tan θ =
5 5 4
3
sin θ 5 = 3
Now = 4 4 = tan θ.
cos θ
5
AB
Again sin θ = gives us
AC
1 1 AC
= = = cosec θ
sin θ AB AB
AC
1
Thus cosec θ = or cosec θ . sin θ = 1
sin θ
We say cosec θ is the reciprocal of sin θ.
BC
Again, cos θ = gives us
AC
1 1 AC
= = = sec θ
cos θ BC BC
AC
1
Thus sec θ = or sec θ . cos θ = 1
cos θ
We say that sec θ is reciprocal of cos θ.
AB
Finally, tan θ = gives us
BC
1 1 BC
= = = cot θ
tan θ AB AB
BC
Notes
1
Thus, cot θ = or tan θ . cot θ = 1
tan θ
1 cos θ
=
Also cot θ = sin θ sin θ
cos θ
We say that cot θ is reciprocal of tan θ.
Thus, we have cosec θ, sec θ and cot θ are reciprocal of sin θ, cos θ and tan θ respectively.
We have, therefore, established the following results:
sin θ
(i) tan θ =
cos θ
1
(ii) cosec θ =
sin θ
1
(iii) sec θ =
cos θ
1 cos θ
(iv) cot θ = =
tan θ sin θ
Now we can make use of the above results in finding the values of different trigonometric
ratios.
1 3
Example 22.14: If cos θ = and sin θ = , find the values of cosec θ, sec θ and
2 2
tan θ.
Solution: We know that
1 1 2
cosec θ = = 3 =
sin θ 3
2
1 1
sec θ = = =2
cos θ 1
2
Mathematics Secondary Course 531
MODULE - 5 Introduction to Trigonometry
Trigonometry
3
sin θ 3 2
and tan θ = = 2 = × = 3
cos θ 1 2 1
Notes 2
Example 22.15: For a right angled triangle ABC, right angled at C, tan A = 1. Find the
value of cos B.
Solution: Let us construct a right angled ΔABC in which ∠C = 90o.
We have tan A = 1 (Given)
We know that A
BC
tan A = =1
AC
∴ BC and AC are equal.
Let BC = AC = k
B C
Then AB = BC + AC
2 2
Fig. 22.26
= k2 + k2
= 2k
BC k
Now cos B = =
AB 2k
1
=
2
1
Hence cos B =
2
1 3
1. If sin θ = and cos θ = , find the values of cot θ and sec θ .
2 2
3
2. If sin θ = and tan θ = 3 , find the value of cos2 θ + sin θ cot θ.
2
22.5 IDENTITY
We have studied about equations in algebra in our earlier classes. Recall that when two
expressions are connected by ‘=’ (equal to) sign, we get an equation. In this section, we
now introduce the concept of an identity. We get an identity when two expressions are
connected by the equality sign. When we say that two expressions when connected by ‘=’
give rise to an equation as well as identity, then what is the difference between the two.
The major difference between the two is that an equation involving a variable is true for
some values only whereas the equation involving a variable is true for all values of the
variable, is called an identity.
Thus x2 – 2x + 1 = 0 is an equation as it is true for x = 1.
x2 – 5x + 6 = 0 is an equation as it is true for x = 2 and x = 3.
If we consider x2 – 5x + 6 = (x – 2) (x – 3), it becomes an identity as it is true for x = 2,
x = 3 and say x = 0, x = 10 etc. i.e. it is true for all values of x. In the next section, we shall
consider some identities in trigonometry.
PM
sin θ =
OP
Notes
OM
and cos θ =
OP
Squaring and adding, we get
2 2
⎛ PM ⎞ ⎛ OM ⎞
sin θ + cos θ = ⎜
2 2 ⎟ +⎜ ⎟
⎝ OP ⎠ ⎝ OP ⎠
PM 2 + OM 2 OP 2
= =
OP 2 OP 2
=1
Hence, sin2 θ + cos2 θ = 1 ...(1)
Also we know that
OP
sec θ =
OM
PM
and tan θ =
OM
Squaring and subtracting, we get
2 2
⎛ OP ⎞ ⎛ PM ⎞
sec θ – tan θ = ⎜
2 2 ⎟ −⎜ ⎟
⎝ OM ⎠ ⎝ OM ⎠
OP 2 − PM 2
=
OM 2
OM 2
= [By Pythagoras Theorm, OP2 – PM2 = OM2]
OM 2
=1
Hence, sec2 θ – tan2 θ = 1 ...(2)
OP
Again, cosec θ =
PM
OM
and cot θ =
PM
sin 2θ + cos 2θ 1
= = (Q sin2 θ + cos2 θ = 1)
sin θ cos θ sin θ cos θ
= R.H.S.
1
Hence, tan θ + cot θ =
sin θ cos θ
Exampe 22.17: Prove that
sin A 1 + cos A
+ = 2 cosec A
1 + cos A sin A
sin A 1 + cos A
Solution: L.H.S = +
1 + cos A sin A
sin 2 A + (1 + cos A )
2
=
sin A (1 + cos A )
Notes
sin 2 A + 1 + cos 2 A + 2 cos A
=
sin A (1 + cos A )
=
sin A (1 + cos A )
1 + 1 + 2 cos A
sin A (1 + cos A )
=
2 + 2 cos A
sin A (1 + cos A )
=
2 (1 + cos A )
sin A (1 + cos A )
=
2
=
sin A
= 2 cosec A
= R.H.S.
sin A 1 + cos A
Hence, + = 2 cosec A
1 + cos A sin A
Example 22.18: Prove that:
1 − sin A
= (sec A − tan A )
2
1 + sin A
2
⎛ 1 sin A ⎞
=⎜ − ⎟
⎝ cos A cos A ⎠
2
⎛ 1 − sin A ⎞
=⎜ ⎟
⎝ cos A ⎠
(1− sin A ) 2
=
cos 2 A
(1 − sin A )2 Notes
=
(1 – sin A )(1 + sin A )
1 − sin A
=
1 + sin A
= L.H.S.
1 − sin A
= (sec A − tan A )
2
Hence,
1 + sin A
Alternative method
We can prove the identity by starting from L.H.S. in the following way:
1 − sin A
L.H.S. =
1 + sin A
1 − sin A 1− sin A
= ×
1 + sin A 1 – sin A
(1 − sin A )2
=
1 – sin 2 A
(1 − sin A )2
=
cos 2 A
2
⎛ 1 − sin A ⎞
=⎜ ⎟
⎝ cos A ⎠
2
⎛ 1 sin A ⎞
=⎜ − ⎟
⎝ cos A cos A ⎠
= (sec A − tan A )2
= R.H.S.
Remark: From the above examples, we get the following method for solving questions
on Trigonometric identities.
1 − sinθ cosθ
=
1 + sinθ 1 + sinθ
1 − sinθ
Solution: L.H.S. =
1 + sinθ
1 − sinθ 1 + sinθ
= ×
1 + sinθ 1 + sinθ
1 − sin 2 θ
=
(1 + sinθ )
cos 2θ
=
1 + sinθ
(Q1 − sin θ = cos θ )
2 2
cosθ
= = R.H.S.
1 + sinθ
1 − sinθ cosθ
Hence, =
1 + sinθ 1 + sinθ
Example 22.20: Prove that
cos4 A – sin4A = cos2 A – sin2 A = 1 – 2 sin2A
Solution: L.H.S. = cos4 A – sin4A
= (cos2 A)2 – (sin2 A)2
= (cos2 A + sin2 A) (cos2 A – sin2 A)
=
1
(1 − sin A)⎛⎜ 1 + sin A ⎞⎟
cos A ⎝ cos A cos A ⎠
1 − sin 2 A
=
cos 2 A
cos 2 A
=
cos 2 A
= 1 = R.H.S.
Hence, sec A (1 – sin A) (sec A + tan A) = 1
Example 22.22: Prove that
tan θ + sec θ − 1
Solution: L.H.S. =
tan θ − sec θ + 1
1 + sinθ
=
cosθ
= R.H.S.
1 − sin 2 θ
=
cosθ (1 − sinθ )
cos 2 θ
=
cosθ (1 − sinθ )
cosθ
=
1 − sinθ
= R.H.S.
Example 22.23: If cos θ – sin θ = 2 sin θ, then show that cos θ + sin θ = 2 cos θ.
or cos θ = ( 2 + 1) sin θ
cosθ
or = sinθ
2 +1
sinθ =
cosθ
×
( )
2 −1 Notes
or
2 +1 ( )
2 −1
2 cos θ − cos θ
or sin θ =
2 −1
1
or 1 + tan 2 θ = = cot 2 θ
tan θ
2
1 cos 2 θ
or =
cos 2 θ sin 2 θ
or sin2 θ = cos4 θ
or 1 – cos2 θ = cos4 θ (sin2 θ = 1 – cos2 θ)
or cos4 θ + cos2 θ = 1
sinA sinA
5. + = 2cosecA
1 + cosA 1 − cosA
1 − cosA
= (cosecA − cotA )
2
13.
1 + cosA
tanA cotA
14. + = 1 + secAcosecA
1 − cotA 1 − tanA
cos2 θ + cos4 θ = 1
Select the correct alternative from the four given in each of the following questions (17 - 20):
17. (sin A + cos A)2 – 2 sin A cos A is equal to
(i) 0 (ii) 2 (iii) 1 (iv) sin2A – cos2A
18. sin4A – cos4A is equal to:
(i) 1 (ii) sin2A – cos2A (iii) 0 (iv) tan2A
PM OM PM
sin θ = , cos θ = and tan θ =
OP OP OM
OP OP OM
cosec θ = , sec θ = and cot θ =
PM OM PM
For reference angle (90o –θ), we have in right ∠d ΔOPM,
(
sin 90 o − θ =) OM
OP
= cos θ
Notes
(
cos 90 o − θ = ) PM
OP
= sin θ
(
tan 90 o − θ =) OM
PM
= cot θ
(
cot 90 o − θ =) PM
OM
= tan θ
(
cosec 90 o − θ = ) OP
OM
= sec θ
and (
sec 90 o − θ =) OP
PM
= cosec θ
The above six results are known as trigonometric ratios of complementary angles. For
example,
sin (90o – 20o) = cos 20o i.e. sin 70o = cos 20o
tan (90o – 40o) = cot 40o i.e. tan 50o = cot 40o and so on.
Let us take some examples to illustrate the use of above results.
Example 22.25: Prove that tan 13o = cot 77o
Solution: R.H.S. = cot 77o
= cot (90o – 13o)
= tan 13o ....[Q cot (90o – θ) = tan θ]
= L.H.S.
Thus, tan 13o = cot 77o
Example 22.26: Evaluate sin2 40o – cos2 50o
Solution: cos 50o = cos (90o – 40o)
= sin 40o ....[Q cos (90o – θ) = tan θ]
∴ sin2 40o – cos2 50o = sin2 40o – sin2 40o = 0
1 1
o
= 3 sin 17 . o
+ 2 tan 20 o.
sin 17 tan 20 o
=3+2=5
Example 22.29: Show that tan 7o tan 23o tan 67o tan 83o = 1
Solution: tan 67o = tan (90o – 23o) = cot 23o
and tan 83o = tan (90o – 7o) = cot 7o
Now. L.H.S. = tan 7o tan 23o tan 67o tan 83o
= tan 7o tan 23o cot 23o cot 7o
= (tan 7o cot 7o) (tan 23o cot 23o)
= 1 .1 = 1
= R.H.S.
Hence, tan 7o tan 23o tan 67o tan 83o = 1
Example 22.30: If tan A = cot B, prove that A + B = 90o.
Solution: We are given
tan A = cot B
or tan A = tan (90o – B) ... [Q cot θ = tan (90o – θ)]
∴ A = 90o – B
or A + B = 90o
⎛ B+C⎞ ⎛A⎞
Example 22.31: For a ΔABC, show that sin ⎜ ⎟ = cos⎜ ⎟ , where A, B and C are
⎝ 2 ⎠ ⎝2⎠
interior angles of ΔABC.
Notes
Solution: We know that sum of angles of triangle is 180o.
∴ A + B + C = 180o
or B + C = 180o – A
B+C A
or = 90 o −
2 2
⎛ B+C⎞ ⎛ o A⎞
∴ sin ⎜ ⎟ = sin ⎜ 90 − ⎟
⎝ 2 ⎠ ⎝ 2⎠
⎛ B+C⎞ ⎛A⎞
or sin ⎜ ⎟ = cos⎜ ⎟
⎝ 2 ⎠ ⎝2⎠
cosθ sinθ
+ = 2.
Example 22.32: Prove that
(
sin 90 − θ cos 90 o − θ
o
) ( )
cosθ sinθ
+
Solution: L.H.S. =
( )
sin 90 − θ cos 90 o − θ
o
( )
cos θ sin θ
= + ... [Q sin (90o – θ) = cos θ and cos (90o – θ) = sin θ]
cos θ sin θ
=1+1=2
= R.H.S.
cosθ sinθ
+ =2
Hence,
( ) (
sin 90 − θ cos 90 o − θ
o
)
sin 90o − θ
+
((
cos 90 o − θ
=1
) )
Example 22.33: Show that
( ) (
cosec 90o − θ sec 90o − θ )
sin (90 − θ ) cos (90 − θ )
o o
+
cosec (90 − θ ) sec(90 − θ )
Solution: L.H.S. = o o
cos θ sin θ
= + ...[Q sin (90o – θ) = cos θ, cos (90o – θ) = sin θ,
secθ cosec θ
cosec (90o – θ) = sec θ and sec (90o – θ) = cosec θ]
cos θ sin θ
= + = cos2 θ + sin2 θ = 1
cos θ sin θ
= R.H.S.
Notes
(
sin 90 − θ
+
)
cos 90 − θ
o
=1
( o
)
Hence,
(
cosec 90 − θ sec 90o − θ
o
) ( )
Example 22.34: Simplify:
( ) (
cos 90 o − θ sec 90 o − θ tanθ ) +
tan 90 o − θ( )
( ) ( ) (
cosec 90 o − θ sin 90 o − θ cot 90 o − θ )
cotθ
( ) (
cos 90 o − θ sec 90 o − θ tanθ
+
)
tan 90 o − θ ( )
=
( ) (
cosec 90 o − θ sin 90 o − θ cot 90 o − θ ) ( cotθ )
sin θinθ.co θ.tanθ cot θ
= + ...[Q sin θ . cos θ = 1 and sec θ . cos θ = 1]
sec θecθ.cotan θ cot θ
=1+1
=2
Example 22.35: Express tan 68o + sec 68o in terms of angles between 0o and 45o.
Remark: While using notion of complementary angles, usually we change that angle which
is > 45o to its complement.
Example 22.36: If tan 2A = cot (A – 18o) where 2A is an acute angle, find the value of A.
or cot (90o – 2A) = cot (A – 18o) ...[Q cot (90o – 2A = tan 2A]
or 3A = 90o + 18o
Notes or 3A = 108o
or A = 36o
(
cos 90 o − θ
+
)
1 + sin 90 o − θ (
= 2cosecθ
)
(iii)
(
1 + sin 90 o − θ )
cos 90 o − θ ( )
Notes
( ) ( ) (
tan 90 − θ o
)
(iv) sin 90 − θ .cos 90 − θ =
o o
(
1 + tan 2 90 o − θ )
(v) tan 45o tan 13o tan 77o tan 85o = 1
(vi) 2 tan 15o tan 25o tan 65o tan 75o = 2
(vii) sin 20o sin 70o – cos 20o cos 70o = 0
5. Show that sin (50o + θ) – cos (40o – θ) = 0
6. If sin A = cos B where A and B are acute angles, prove that A + B = 90o.
7. In a ΔABC, prove that
⎛ B+C⎞ ⎛A⎞
(i) tan ⎜ ⎟ = cot ⎜ ⎟
⎝ 2 ⎠ ⎝2⎠
⎛ A+B⎞ ⎛C⎞
(ii) cos⎜ ⎟ = sin ⎜ ⎟
⎝ 2 ⎠ ⎝2⎠
8. Express tan 59o + cosec 85o in terms of trigonometric ratios of angles between 0o and
45o.
9. Express sec 46o – cos 87o in terms of trigonometric ratios of angles between 0o and
45o.
10. Express sec2 62o + sec2 69o in terms of trigonometric ratios of angles between 0o and
45o.
Select the correct alternative for each of the following questions (11-12):
1 1
(i) – 1 (ii) (iii) − (iv) 1
6 6
12. If sin (θ + 36o) = cos θ, where θ + 36o is an acute angle, then θ is
(i) 54o (ii) 18o (iii) 21o (iv) 27o
LET US SUM UP
Hypotenuse AC
cosec θ = =
side opposite to angle θ AB
sin θ cos θ
(i) tan θ = (ii) cot θ =
cos θ sin θ
1 1
(iii) sec θ = (iv) cosec θ =
cos θ sin θ
1
(v) cot θ =
tan θ
• The trigonometric identities are:
(i) sin2 θ + cos2 θ = 1
(ii) sec2 θ – tan2 θ = 1
(iii) cosec2 θ – cot2 θ = 1
• Two angles, whose sum is 90o, are called complementary angles.
TERMINAL EXERCISE
4
1. If sin A = , find the values of cos A and tan A.
5
20
2. If tan A = , find the values of cosec A and sec A.
21
3
3. If cot θ = , find the value of sin θ + cos θ.
4
m
4. If sec θ = , find the values of sin θ and tan θ.
n
3
5. If cos θ = , find the value of
5
sin θ tan θ − 1
2 tan 2 θ
5 tan θ
6. If sec θ = , find the value of
4 1 + tan θ
7. If tan A = 1 and tan B = 3 , find the value of cos A cos B – sin A sin B.
1 + cos A
13. = cosec A + cot A
1 − cos A
cosec A + 1 cos A
14. =
cosec A −1 1 − sin A
15. sin3A – cos3A = (sin A – cos A) (1 + sin A cos A)
cos A sin A
16. + = cos A + sin A
1 − tan A 1 − cot A
sec A − 1 sec A + 1
17. + = 2cosecA
sec A + 1 sec A − 1
1
18. (cosecA − sin A )(sec A − cos A ) =
tan A + cot A
19. (1 + cot θ – cosec θ) (1 + tan θ + sec θ) = 2
20. 2(sin6 θ + cos6 θ ) – 3(sin4 θ + cos4 θ ) + 1 = 0
p2 −1
21. If sec θ + tan θ = p, show that sin θ =
p2 +1
(
cos 90o − A
+
) (
1 + sin 90o − A )
= 2sec 90o − A ( )
22. Prove that
1 + sin 90 − A
o
( ) (
cos 90 − A
o
)
3
24. If tan θ = and θ + α = 90o, find the value of cot α.
4
25. If cos (2θ + 54o) = sin θ and (2θ + 54o) is an acute angle, find the value of θ.
26. If sec Q = cosec P and P and Q are acute angles, show that P + Q = 90o.
3 4 3
(ii) sin θ = , cos θ = , tan θ =
5 5 4
5 5 4
cosec θ = , sec θ = and cot θ =
3 4 3
24 7 24
(iii) sin θ = , cos θ = , tan θ =
25 25 7
25 25 7
cosec θ = , sec θ = and cot θ =
24 7 24
4 3 4
(iv) sin θ = , cos θ = , tan θ =
5 5 3
5 5 3
cosec θ = , sec θ = and cot θ =
4 3 4
5 4 5
2. sin A = , cos A = and tan A =
41 41 4
40 9 40 40
3. sin C = , cot C = , cos A = and cot A =
41 40 41 9
3 4 3
1. sin C = , cos C = and tan C =
5 5 4
24 25 7
2. sin A = , cosec A = and cot A =
25 24 24
2 1 2
4. tan R = 3 , cosec R = 3 , sin P = 2 and sec P = 3
24 7 25 7
5. cot θ = , sin θ = , sec θ = , and tan θ =
7 25 24 24
2 6 5 5 2 6
6. sin P = , cos P = , sin R = and cos R = , sin P – cos R = 0
7 7 7 7
7. (iii)
22.3
21 20
1. cos θ = and tan θ =
29 21
24 7
2. sin θ = and cos θ =
25 25
24 24
3. sin A = and tan A =
25 7
m n
4. cot θ = and cosec θ =
n 2 − m2 n 2 − m2
256
5. −
135
27
6.
8
2 2
7. sin B = and tan B =
13 3
11. (ii)
22.4
2
1. cot θ = 3 and sec θ = 3
3
2.
4
3 Notes
3.
2
1 1
4. sin A = and tan A =
2 3
14
5. −
3
22.5
17. (iii)
18. (ii)
19. (i)
20. (iii)
22.6
1 1
1. (i) 3 (ii) (iii) (iv) 0
2 3
(v) 5 (vi) 0 (vii) 2 (viii) 1
1
3. (i) 2 (ii)
3
8. cot 31o + sec 5o
9. cosec 44o – sin 3o
10. cosec2 28o + cosec2 21o
11. (ii)
12. (iv)
3 4
1. cos A = and tan A =
5 3
29 29
2. cosec A = and sec A =
20 21
Notes 7
3.
5
m2 − n 2 m2 − n 2
4. sin θ = and tan θ =
m n
3
5.
160
3
6.
7
1− 3
7.
2 2
3
24.
4
25. 12o
23 Notes
In the last lesson, we have defined trigonometric ratios for acute angles in a right triangle
and also developed some relationship between them. In this lesson we shall find the values
of trigonometric ratios of angles of 30o, 45o and 60o by using our knowledge of geometry.
We shall also write the values of trigonometric ratios of 0o and 90o and we shall observe
that some trigonometric ratios of 0o and 90o are not defined. We shall also use the knowledge
of trigonometry to solve simple problems on heights and distances from day to day life.
OBJECTIVES
After studying this lesson, you will be able to
• find the values of trigonometric ratios of angles of 30o, 45o and 60o;
• write the values of trigonometric ratios of 0o and 90o;
• tell, which trigonometric ratios of 0o and 90oare not defined;
• solve daily life problems of heights and distances;
Notes 1 1 1
cosec C = , sec C = and cot C =
sin C cos C tan C
• sin (90o – θ) = cos θ, cos (90o – θ) = sin θ
tan (90o – θ) = cot θ, cot (90o – θ) = tan θ
• sec (90o – θ) = cosec θ and cosec (90o – θ) = sec θ
∴ OM = PM
Let OM = a units, then PM = a units.
In right triangle PMO, Y’
= a2 + a2
= 2 a2
∴ OP = 2 a units
PM a 1
Now sin 45o = = =
OP 2a 2
OM a 1
cos 45o = = =
OP 2a 2
PM a
tan 45o = = =1
OM a
1 1 Notes
cosec 45o = o
= = 2
sin 45 1 / 2
1 1
sec 45o = o
= = 2
cos 45 1 / 2
1 1
and cot 45o = o
= =1
tan 45 1
Notes or OM = 3 a units
PM a 1
∴ sin 30o = = =
OP 2a 2
OM 3a 3
cos 30o = = =
OP 2a 2
PM a 1
tan 30o = = =
OM 3a 3
1 1
cosec 30o = o
= =2
sin 30 1/ 2
1 1 2
sec 30o = o
= =
cos 30 3/2 3
1 1
and cot 30o = o
= = 3
tan 30 1/ 3
∴ PM = 3 a units
PM 3a 3
∴ sin 60o = = =
OP 2a 2
OM a 1
cos 60o = = =
OP 2a 2
PM 3a
tan 60o = = = 3
OM a
1 1 2
cosec 60o = o
= =
sin 60 3 3
2
1 1
sec 60o = o
= =2
cos 60 1/ 2
1 1
and cot 60o = 0
=
tan 60 3
Notes (v) cos 90o = 0 and therefore sec 90o is not defined.
(vi) cot 90o = 0 and therefore tan 90o is not defined.
The values of trignometric ratios for 0o, 30o, 45o, 60o and 90o can be put in a tabular form
which makes their use simple. The following table also works as an aid to memory.
θ 0o 30o 45o 60o 90o
Trig. ratio
0 1 1 2 1 3 3 4
sin θ =0 = = = =1
4 4 2 4 2 4 2 4
4 3 3 2 1 1 1 0
cos θ =1 = = = =0
4 4 2 4 2 4 2 4
0 1 1 2 3
tan θ =0 = =1 = 3 Not defined
4−0 4 −1 3 4−2 4−3
3 2 1 1 0
cot θ Not defined = 3 =1 = =0
4−3 4−2 4 −1 3 4−0
4 4 4 2 4
cosec θ Not defined =2 = 2 = =1
1 2 3 3 4
4 4 2 4 4
sec θ =1 = = 2 =2 Not defined
4 3 3 2 1
Let us, now take some examples to illustrate the use of these trigonometric ratios.
Example 23.1: Find the value of tan260o – sin230o.
1
Solution: We know that tan 60o = 3 and sin 30o =
2
tan 60 – sin 30 = ( 3 ) − ⎜ ⎟
2
⎛1⎞ 2
∴ 2 o 2 o
2 ⎝ ⎠
1 11
= 3− =
4 4
= ( 3 ) ( 2 ) + ( 2 ) . 12
2 2 2
1
=3×2+2×
2
=6+1
=7
Example 23.3: Evaluate : 2(cos2 45o + tan260o) – 6(sin245o – tan230o)
Solution: 2(cos2 45o + tan260o) – 6(sin245o – tan230o)
⎡⎛ 1 ⎞ 2
( 3 ) ⎤⎥ − 6⎡⎢⎛⎜ 1 ⎞ ⎛ 1 ⎞ ⎤
2 2
2
= ⎢⎜
2 ⎟ + ⎟ − ⎜ ⎟ ⎥
⎣⎢⎝ 2 ⎠ ⎦⎥ ⎢⎣⎝ 2 ⎠ ⎝ 3 ⎠ ⎥⎦
⎛1 ⎞ ⎛1 1⎞
= 2⎜ + 3 ⎟ − 6⎜ − ⎟
⎝2 ⎠ ⎝ 2 3⎠
=1+6–3+2
=6
Example 23.4: Verify that
1 2 5 ×1
= + −
2 1 2 ×1
1 5
= + 2 − = 0 = R.H.S.
2 2
4 3 10
cot 2 30 o + 3 sin 2 60 o − 2 cosec 2 60 o − tan 2 30 o =
3 4 3
4 3
Solution: L.H.S. = cot 2 30 o + 3 sin 2 60 o − 2 cosec 2 60 o − tan 2 30 o
3 4
2
⎛ 3⎞
( )
2 2
⎟ − 2⎛⎜
4 2 2 ⎞ 3⎛ 1 ⎞
= × 3 + 3⎜⎜ ⎟ ⎟ − ⎜ ⎟
3 ⎝ 2 ⎠ ⎝ 3 ⎠ 4 ⎝ 3 ⎠
4 3 4 3 1
= × 3 + 3× − 2 × − ×
3 4 3 4 3
9 8 1
= 4+ − −
4 3 4
48 + 27 − 32 − 3
=
12
40 10
= =
12 3
= R.H.S.
4 3 10
Hence, cot 2 30 o + 3 sin 2 60 o − 2 cosec 2 60 o − tan 2 30 o =
3 4 3
Example 23.6 : Verify that
1 4 1
4× + − 2×
3 3 2
= 3 1
+
4 2 Notes
8 5
−1
3 = 3
= 5 5
4 4
5 4 4
= × =
3 5 3
= R.H.S.
2 tan θ
tan 2θ =
1 − tan 2 θ
Solution: For θ = 30o
L.H.S. = tan 2θ
= tan (2 × 30o)
= tan 60o
= 3
2 tan θ
and R.H.S. =
1 − tan 2 θ
2 tan 30 o
=
1 − tan 2 30 o
⎛ 1 ⎞
2.⎜ ⎟
⎝ 3⎠
= 2
⎛ 1 ⎞
1− ⎜ ⎟
⎝ 3⎠
2 2
3 = 3
=
1 2
Notes 1−
3 3
2 3
= × = 3
3 2
∴ L.H.S. = R.H.S.
2 tan θ
Hence, tan 2θ =
1 − tan 2 θ
Example 23.8: Let A = 30o. Verify that
sin 3A = 3 sin A – 4 sin3 A
Solution: For A = 30o,
L.H.S. = sin 3A
= sin (3 × 30o)
= sin 90o
=1
and R.H.S. = 3 sin A – 4 sin3 A
= 3 sin 30o – 4 sin3 30o
3
1 ⎛1⎞
= 3× − 4 × ⎜ ⎟
2 ⎝2⎠
3 4
= −
2 8
3 1
= −
2 2
=1
∴ L.H.S. = R.H.S.
Hence, sin 3A = 3 sin A – 4 sin3 A
Example 23.9: Using the formula sin (A – B) = sin A cos B – cos A sin B, find the value
of sin 15o.
1 3 1 1
or sin 15o = × − ×
2 2 2 2
3 −1
=
2 2
3 −1
Hence, sin 15o = .
2 2
Remark: In the above examples we can also take A = 60o and B = 45o.
Example 23.10: If sin (A + B) = 1 and cos (A – B) = 1, 0o < A + B ≤ 90o, A ≥ B, find A
and B.
Solution: Q sin (A + B) = 1 = sin 90o
∴ A + B = 90o ...(i)
Again cos (A – B) = 1 = cos 0o
∴ A – B = 0o ...(ii)
Adding (i) and (ii), we get
2A = 90o or A = 45o
From (ii), we get
B = A = 45o
Hence, A = 45o and B = 45o
Example 23.11: If cos (20o + x) = sin 30o, find x.
⎛ 1⎞
Q cos 60 o = ⎟
Solution: cos (20o + x) = sin 30o = = cos 60o ... ⎜⎝ 2⎠
∴ 20o + x = 60o
or x = 60o – 20o = 40o
Hence, x = 40o
5
or sin 30o =
AC
1 5
or =
2 AC
C 5 cm B
∴ AC = 2 × 5 or 10 cm
Fig. 23.4
By Pythagoras Theorem,
AB = AC2 − BC2
= (10)2 − 52 cm
= 75 cm
= 5 3 cm
AC 8 cm
Now sin B = 4 cm
AB
4 1 B C
= or Fig. 23.5
8 2
⎡ 1⎤
... ⎢Q sin 30 = ⎥
o
∴ B = 30o
⎣ 2⎦
Now ∠A = 90o – ∠B [
.... Q ∠A + ∠B = 90o ]
1 1
= + =1
2 2
(ii) sin A sin B + cos A cos B
= sin 45o sin 90o + cos 45o cos 90o
1 1
= ×1 + ×0
2 2
1
=
2
tan 2x – 3 =0
o
or tan 2x = 3 = tan 60
(i) cosec3 30o × cos 60o × tan3 45o × sin2 90o × sec2 45o × cot 30o = 8 3
1 2 o 1 7
(ii) tan 30 + sin 45 + cos 30 + cot 60 =
2 o 2 o 2 o
2 3 6
(iii) cos260o – sin260o = – cos 60o
(iv) 4(sin430o + cos460o) – 3(cos245o – sin290o) = 2
tan 60 o − tan 30 o
(v) = tan 30o
1 + tan 60 o tan 30o
3. If ∠A = 30o, verify each of the following:
2 tan A
(i) sin 2A =
1 + tan 2 A
1 − tan 2 A
(ii) cos 2A =
1 + tan 2 A
(iii) cos 3 A = 4 cos3A – 3 cos A
tan A − tan B
(ii) tan (A − B) = Notes
1 + tan A tan B
5. Taking 2A = 60o, find sin 30o and cos 30o, using cos 2A = 2 cos2A – 1.
6. Using the formula cos (A + B) = cos A cos B – sin A sin B, evaluate cos 75o.
1 1
7. If sin (A – B) = , cos (A + B) = , 0o < A + B < 90o, A > B, find A and B.
2 2
3
8. If sin (A + 2B) = and cos (A + 4 B) = 0, find A and B.
2
11. In ΔABC, ∠B = 90o. If A = 30o, find the value of sin A cos B + cos A sin B.
3 2
(A) 2 (B) (C) (D) 2
2 3
14. If sin 2A = 2 sin A, then A is
(A) 30o (B) 0o (C) 60o (D) 90o
2 tan 60 o
15. is equal to
1 + tan 2 60 o
(A) sin 60o (B) sin 30o (C) cos 60o (D) tan 60o
When the observer is looking at an object (P) which is at a greater height than the observer
(A), he has to lift his eyes to see the object and an angle of elevation is formed between the
line of sight joining the observer’s eye to the object and the borizontal line. In Fig. 23.6, A
is the observer, P is the object, AP is the line of sight and AB is the horizontal line, then ∠θ
is the angle of elevation.
. P Object
t
s igh
e of
Lin
A
Observer
. θ
Horizontal line
B
Fig. 23.6
BC
= cos 60 o
AC
60o
C B
Fig. 23.8
BC 1
or =
6 2
1 Notes
or BC = × 6 or 3 m
2
Hence, the foot of the ladder is 3 m away from the wall.
1
Example 23.17: The shadow of a vertical pole is of its height. Show that the sun’s
3
elevation is 60o.
Solution: Let AB be vertical pole of height h units and BC be its shadow.
1 A
Then BC = h × units
3
Let θ be the sun’s elevation.
Then in right ΔABC, h units
AB h
tan θ = = = 3
BC h/ 3
θ B
C
or tan θ = tan 60o h
units
∴ θ = 60o 3
Example 23.18: A tower stands vertically on the ground. The angle of elevation from a
point on the ground, which is 30 m away from the foot of the tower is 30o. Find the height
of the tower. (Take 3 = 1.73)
Solution: Let AB be the tower h metres high.
Let C be a point on the ground, 30 m away A
from B, the foot of the tower
∴ BC = 30 m hm
Then by question, ∠ACB = 30o
30o
Now in right ΔABC, C 30 m B
Fig. 23.10
AB
= tan 30o
BC
h 1
or =
30 3
Notes 30
∴ h= m
3
30 3
= × m
3 3
= 10 3 m
= 10 × 1.73 m
= 17.3 m
Hence, height of the tower is 17.3 m.
Example 23.19: A balloon is connected to a meterological ground station by a cable of
length 100 m inclined at 60o to the horizontal. Find the height of the balloon from the
ground assuming that there is no slack in the cable.
Solution: Let A be the position of the balloon, attached to the cable AC of length 100 m.
AC makes an angle of 60o with the level ground BC.
Let AB, the height of the balloon be h metres
A
Now in right ΔABC,
AB
= sin 60o
AC
100 m
hm
h 3
or =
100 2
60o
or h = 50 3 B
C
= 50 × 1.732 Fig. 23.11
= 86.6
Hence, the balloon is at a height of 86.6 metres.
Example 23.20: The upper part of a tree is broken by the strong wind. The top of the tree
makes an angle of 30o with the horizontal ground. The distance between the base of the
tree and the point where it touches the ground is 10 m. Find the height of the tree.
Solution: Let AB be the tree, which was broken at C, by the wind and the top A of the
tree touches the ground at D, making an angle of 30o with BD and BD= 10 m.
Let BC = x metres . A
x 1
. C
or =
10 3 xm
or x=
10
3
m ...(i) D
30o
10 m
. B
Fig. 23.12
Again in right ΔCBD,
BC
= sin 30 o
DC
x 1
or =
DC 2
or DC = 2x
20
= m ...[By (i)]
3
20
∴ AC = DC = ...(ii)
3
Now height of the tree = BC + AC
⎛ 10 20 ⎞
=⎜ + ⎟
⎝ 3 3⎠
30
= or 10 3 m
3
= 17.32 m
Hence height of the tree = 17.32 m
Example 23.21: The shadown of a tower, when the angle of elevation of the sun is 45o is
found to be 10 metres longer than when it was 60o. Find the height of the tower.
AB
= tan 45o hm
BC
h
or =1
x + 10
∴ x = (h – 10) m ...(i) 45o 60o
C D B
10 m xm
Again in rt ∠d ΔABD,
Fig. 23.13
AB
= tan 60 o
BD
h
or = 3
x
or h= 3x ...(ii)
h= 3 (h – 10)
or h= 3 h – 10 3
or ( 3 –1)h = 10 3
10 3
∴ h=
3 −1
10 3 ( )
3 + 1 10 3 3 + 1
=
( )
=
3 −1
×
( )
3 +1 2
AQ
= tan 45o = 1
AO
60o 45o
or AQ = AO ...(i) A O
PA
= tan 60 o = 3
AO
3000 3000
∴ = 3 or AO = ...(ii)
AO 3
From (i) and (ii), we get
3000 3
AQ = × = 1000 3 = 1732 m
3 3
50 P
or = tan 60 o ...(ii)
BQ
AB hm
= tan 60 o
BC
h
or = 3
x
30o 60o
or h= 3x ...(i) D C B
40 m xm
Again in right ΔABD,
Fig. 23.16
AB
= tan 30 o
BD
h 1
or = ...(ii)
x + 40 3
From (i) and (ii), we get,
3x 1
=
x + 40 3
h= 3 × 20 = 20 × 1.732
= 34.64
Hence, width of the river is 20 m and height of the tree is 34.64 metres.
Example 23.25: Standing on the top of a tower 100 m high, Swati observes two cars on
the opposite sides of the tower. If their angles of depression are 45o and 60o, find the
distance between the two cars.
Solution: Let PM be the tower 100 m high. Let A and B be the positions of the two cars.
Let the angle of depression of car at A be 60o and of the car at B be 45o as shown in
Fig. 23.17.
Now ∠QPA = 60o = ∠PAB
Q P R
and ∠RPB = 45o = ∠PBA o
60 45o
In right ΔPMB,
PM
= tan 45o 100 m
MB
PM
= tan 60 o
MA
100
or = 3
MA
100
∴ MA =
3
100 3
=
3
100 × 1.732
=
3
Notes = 57.74
∴ MA = 57.74 m ...(ii)
Hence, the distance between the two cars
= MA + MB
= (57.74 + 100) m [By (i) and (ii)]
= 157.74 m
Example 23.26: Two pillars of equal heights are on either side of a road, which is 100 m
wide. At a point on the road between the pillars, the angles of elevation of the top of the
pillars are 60o and 30o respectively. Find the position of the point between the pillars and
the height of each pillar.
Solution: Let AB and CD be two pillars each of height h metres. Let O be a point on the
road. Let BO = x metres, then
OD = (100 – x) m
By question, ∠AOB = 60o and ∠COD = 30o
In right ΔABO,
AB A C
= tan 60 o
BO
h
or = 3
x hm
hm
or h= 3 x ...(i)
60o 30o
In right ΔCDO, D
B O
xm (100–x) m
CD
= tan 30 o
OD Fig. 23.18
h 1
or = ...(ii)
100 − x 3
From (i) and (ii), we get
3x 1
=
100 − x 3
AC
= tan 45o
OC 3000 m
3000
or =1 45o 30
o
OC O C D
or OC = 3000 m ...(i)
Fig. 23.19
In rt ∠d ΔBDO,
BD
= tan 30 o
OD
3000 1
or =
OC + CD 3
or CD = 3000 ( 3 – 1)
= 3000 × 0.732
= 2196
∴ Distance covered by the aeroplane in 15 seconds = AB = CD = 2196 m
⎛ 2196 60 × 60 ⎞
∴ Speed of the plane = ⎜ × ⎟ km/h
⎝ 15 1000 ⎠
Notes = 527.04 km/h
Example 23.28: The angles of elevation of the top of a tower from two points P and Q at
distanes of a and b respectively from the base and in the same straight line with it are
complementary. Prove that the height of the tower is ab .
Solution: Let AB be the tower of height h, P and Q be the given points such taht PB = a
and QB = b.
Let ∠APB = α and ∠AQB = 90o – α A
Now in rt ∠d ΔABQ,
AB
QB
(
= tan 90o − α )
h
h
or = cot α ...(i)
b
and in rt ∠d ΔABP, α 90o–α
P
Q B
AB b
= tan α
PB a
h Fig. 23.20
or = tan α ...(ii)
a
Multiplying (i) and (ii), we get
h h
× = cot α . tan α = 1
b a
or h2 = ab
or h= ab
Hence, height of the tower is ab .
6. Find the angle of elevation of the top of a tower which is 100 3 m high, from a point
at a distance of 100 m from the foot of the tower on a horizontal plane.
7. A tree 12 m high is broken by the wind in such a way that its tip touches the ground
and makes an angle of 60o with the ground. At what height from the ground, the tree is
broken by the wind?
8. A tree is broken by the storm in such way that its tip touches the ground at a horizontal
distance of 10 m from the tree and makes an angle of 45o with the ground. Find the
height of the tree.
9. The angle of elevation of a tower at a point is 45o. After going 40 m towards the foot
of the tower, the angle of elevation becomes 60o. Find the height of the tower.
10. Two men are on either side of a cliff which is 80 m high. They observe the angles of
elevation of the top of the cliff to be 30o and 60o respectively. Find the distance between
the two men.
11. From the top of a building 60 m high, the angles of depression of the top and bottom
of a tower are observed to be 45o and 60o respectively. Find the height of the tower
and its distance from the building.
12. A ladder of length 4 m makes an angle of 30o with the level ground while leaning
against a window of a room. The foot of the ladder is kept fixed on the same point of
the level ground. It is made to lean against a window of another room on its opposite
side, making an angle of 60o with the level ground. Find the distance between these
rooms.
13. At a point on the ground distant 15 m from its foot, the angle of elevation of the top of
the first storey is 30o. How high the second storey will be, if the angle of elevation of
the top of the second storey at the same point is 45o?
14. An aeroplane flying horizontal 1 km above the ground is observed at an elevation of
60o. After 10 seconds its elevation is observed to be 30o. Find the speed of the
aeroplane.
LET US SUM UP
• Table of values of Trigonometric Ratios
θ 0o 30o 45o 60o 90o
Trig. ratio
1 1 3
sin θ 0 1
2 2 2
3 1 1
cos θ 1 0
2 2 2
1
tan θ 0 1 3 Not defined
3
1
cot θ Not defined 3 1 0
3
2
cosec θ Not defined 2 2 1
3
2
sec θ 1 2 2 Not defined
3
Supportive website:
• http://www.wikipedia.org
• http://mathworld:wolfram.com
TERMINAL EXERCISE
1. Find the value of each of the following:
(i) 4 cos2 60o + 4 sin2 45o – sin2 30o
(ii) sin245o – tan2 45o + 3(sin290o + tan2 30o)
3 5
(i) 2 cot230o – 2 cos260o – sin245o – 4 sec2 30o = −
4 24
(ii) 2 sin230o + 2 tan2 60o – 5 cos245o = 4
(iii) cos 60o cos 45o + sin 60o sin 45o = sin 45o cos 30o + cos 45o sin 30o
2 tan θ
(iii) tan 2θ =
1 − tan 2 θ
4. If A = 60o and B = 30o, verify that
(i) sin (A + B) ≠ sin A + sin B
(ii) sin (A + B) = sin A cos B + cos A sin B
(iii) cos (A – B) = cos A cos B + sin A sin B
(iv) cos (A + B) = cos A cos B – sin A sin B
1 − cos 2 A
(v) tan A =
cos A
5. Using the formula cos (A – B) = cos A cos B + sin A sin B, find the value of cos 15o.
3 o
6. If sin (A + B) = 1 and cos (A – B) = , 0 < A + B ≤ 90o, A > B, find A and B.
2
7. An observer standing 40 m from a building observes that the angle of elevation of the
top and bottom of a flagstaff, which is surmounted on the building are 60o and 45o
respectively. Find the height of the tower and the length of the flagstaff.
1 3
5. sin 30o = , cos 30o =
2 2
3 −1
6.
2 2
9. QR = 5 3 and PR = 10 cm
10. ∠A = 60o and ∠C = 30o Notes
3
11.
2
12. x = 10o
13. C
14. B
15. A
23.2
1. 6 m 2. 86.6 m 3. 86.6 m
4. 86.6 m 5. 115.46 m 6. 60o
7. 5.57 m 8. 24.14 m 9. 94.64 m
10. 184.75 m 11. 25.35 m 12. 5.46 m
13. 6.34 m 14. 415.66 km/h 15. 16.67 m
11 7 40 3
1. (i)
4
(ii)
2
(iii)
121
(iv)
( )
2 3 +1
3 +1
5. 6. A = 60o and B = 30o 7. 40m , 29.28 m
2 2
Secondary Course
Mathematics
Notes
Practice Work-Trignometry
Instructions:
5
(A) cm
5 cm
13 13
12
(B) C B
13 12 cm
5
(C)
12
13
(D)
12
sinA − cosA
2. If 4 cot A = 3, then value of is 1
sinA + cosA
1 6
(A) (B)
7 7
5 3 Notes
(C) (D)
6 4
3
(A) 2 (B)
2
2
(C) (D) 2
3
1
(B)
6
1
(C) –
6
(D) 1
1
6. If sin A = , show that 2
2
3 cos A – 4 cos3 A = 0
7. Using the formula sin (A – B) = sin A cos B – cos A sin B, find the value of sin 15o. 2
8. Find the value of
tan 15o tan 25o tan 60o tan 65o tan 75o 2
1 + sinA
9. Show that = secA + tanA 2
1 − sinA
Notes
10. If sin2θ + sin θ = 1, then show that 2
cos θ + cos θ = 1
2 4
24 Notes
Statistics is a special and an important branch of mathematics which deals mainly with data
and their representations. In this lesson, we shall make a beginning of this study of this
branch of mathematics with collection, classification, presentation and analysis of data.
We shall study how to classify the given data into ungrouped as well as grouped frequency
distributions. We shall also learn about cumulative frequency of a class and cumulative
frequency table.
Further we shall learn graphical representation of data in the form of bar charts, histograms
and frequency polygons.
OBJECTIVES
After studying this lesson, you will be able to
• know meaning of ‘statistics’ in singular and plural form;
• differentiate between primary and secondary data;
• understand the meaning of a class, class mark, class limits, discrete and
continuous data, frequency of a class, class size or class width through examples;
• condense and represent data into a frequency table;
• form a cumulative frequency table of a frequency distribution;
• draw a bar chart or bar graph of a frequency distribution;
• draw a bar chart or bar graph for the given data;
• draw a histogram and frequency polygon for a given continuous data;
• read and interpret given bar graphs, histograms.
(a) Statistics, in singular sense, means the subject which deals with _______, _____,
analysis of data as well as drawing of meaningful _______ from the data.
(c) The data are said to be __________ if the investigator himself is responsible for
its collection.
(d) Data taken from governmental or private agencies in the form of published reports
are called __________ data.
(e) Statistics is the science which deals with collection, organisation, analysis and
interpretation of the ____________.
2. Javed wanted to know the size of shoes worn by the maximum number of persons in
a locality. So, he goes to each and every house and notes down the information on a
sheet. The data so collected is an example of ___________ data.
Notes
3. To find the number of absentees in each day of each class from I to XII, you collect the
information from the school records. The data so collected is an example of _______
data.
60
we decide to have 9 classes. Than the size of each class is ≈7.
9
Step 4: Next, we set up the class limits using the size of the interval determined in
Notes Step 3. We make sure that we have a class to include the minimum as well as a
class to include the maximum value occurring in the data. The classes should be
non-overlapping, no gaps between the classes, and classes should be of the
same size.
Step 5: We take each item (observation) from the data, one at a time, and put a tally
mark (|) against the class to which it belongs. For the sake of convenience, we
record the tally marks in bunches of five, the fifth one crossing the other four
diagonally as ||||.
Step 6: By counting tally marks in each class, we get the frequency of that class. (obviously,
the total of all frequencies should be equal to the total number of observations in
the data)
Step 7: The frequency table should be given a proper title so as to convey exactly what
the table is about.
Using the above steps, we obtain the following table for the marks obtained by 20 students.
Frequency Table of the marks obtained by 20 students in a mathematics test
Class Interval Tally Marks Frequency
(Marks out of 100)
28-34 ||| 3
35-41 | 1
42-48 | 1
49-55 – 0
56-62 |||| ||| 8
63-69 || 2
70-76 |||| 4
77-83 – 0
84-90 | 1
Total 20
The above table is called a frequency distribution table for grouped data or briefly, a
grouped frequency table. The data in the above form are called grouped data.
In the above table, the class 28-34 includes the observations 28, 29, 30, 31, 32, 33 and
34; class 35-41 includes 35, 36, 37, 38, 39, 40 and 41 and so on. So, there is no
overlapping.
For the class 28-34, 28 is called the lower class limit and 34, the upper class limit, and
so on.
From this type of presentation, we can draw better conclusions about the data. Some of
these are. Notes
Suppose two students of weights 35.5 kg and 50.54 kg are admitted in this class. In which
class (interval) will we include them? Can we include 35.5 in class 31-35? In class 36-40?
No! The class 31-35 includes numbers upto 35 and the class 36-40, includes numbers
from 36 onwards. So, there are gaps in between the upper and lower limits of two
consecutive classes. To overcome this difficulty, we divide the intervals in such a way that
the upper and lower limits of consecutive classes are the same. For this, we find the
difference between the upper limit of a class and the lower limit of its succeeding class. We
than add half of this difference to each of the upper limits and subtract the same from each
of the lower limits. For example
Consider the classes 31-35 and 36-40
The lower limit of 36-40 is 36
The upper limit of 31-35 is 35
The difference = 36 – 35 = 1
1
So, half the difference = = 0.5
2
So, the new class interval formed from 31-35 is (31 – 0.5) – (35 + 0.5), i.e., 30.5 – 35.5.
Similarly, class 36-40 will be (36 – 0.5) – (40 + 0.5), i.e., 35.5 – 40.5 and so on.
This way, the new classes will be
30.5-35.5, 35.5-40.5, 40.5-45.5, 45.5-50.5, 50.5-55.5, 55.5-60.5, 60.5-65.5,
65.5-70.5 and 70.5-75.5. These are now continuous classes.
Note that the width of the class is again the same, i.e., 5. These changed limits are called
30.5-35.5 10
40.5-45.5 15
45.5-50.5 4
60.5-65.5 4
65.5-70.5 3
70.5-75.5 2
Total 52
Note: Here, in the above case, we could have also taken the classes as 30-35, 35-40,
40-45, ..., 65-70 and 70-75.
Example 24.1: Construct a frequency table for the following data which give the daily
wages (in rupees) of 32 persons. Use class intervals of size 10.
Earlier, we have discussed presentation of data by tables. There is another way to present Notes
the data called graphical representation which is more convenient for the purpose of
comparison among the individual items. Bar chart (graph) is one of the graphical
representation of numerical data. For example Fig 24.1 represents the data given in the
table regarding blood groups.
Blood groups of 35 students in a class
Blood Group Number of students
A 13
B 9
AB 6
O 7
Total 35
We can represent this data by Fig. 24.1
14
13
12
11
10
Number of Students
9
8
7
6
5
4
3
2
1
0
A B AB O
B l oo d G r ou p
Fig. 24.1
This is called a bar chart or bar graph.
Bars (rectangles) of unifoirm width are drawn with equal spaces in between them, on the
horizontal axis-called x-axis. The heights of the rectangles are shown along the vertical
axis-called y-axis and are proportional to their respective frequencies (number of students).
O
Blood Group
AB
0 1 2 3 4 5 6 7 8 9 10 11 12 13 14
Number of Students
Fig. 24.2
There is not much difference between the bar graphs in Fig. 24.1 and Fig. 24.2 except that
it depends upon the person’s liking to represent data with vertical bars or with horizontal
bars. Generally vertical bar graphs are preferred.
Example 24.4: Given below (Fig. 24.3) is the bar graph of the number of students in
Class IX during academic years 2001-02 to 2005-06. Read the bar graph and answer the
following questions:
(i) What is the information given by the bar graph?
(ii) In which year is the number of students in the class, 250?
(iii) State whether true or false:
The enrolment during 2002-03 is twice that of 2001-02.
350
300
Notes
Number of Students
250
200
150
100
50
0
2001-02 2002-03 2003-04 2004-05 2005-06
Academic Year
Fig. 24.3
Solution:
(i) The bar graph represents the number of students in class IX of a school during academic
year 2001-02 to 2005-06.
(ii) In 2003-04, the number of students in the class was 250.
(iii) Enrolment in 2002-03 = 200
Enrolment in 2001-02 = 150
200 4 1
= =1 < 2
150 3 3
Therefore, the given statement is false.
Example 24.5: The bar graph given in Fig. 24.4 represents the circulation of newspapers
in six languages in a town (the figures are in hundreds). Read the bar graph and answer the
following questions:
(i) Find the total number of newspapers read in Hindi, English and Punjabi.
(ii) Find the excess of the number of newspapers read in Hindi over those of Urdu, Marathi
and Tamil together.
(iii) In which language is the number of newspapers read the least?
(iv) Write, in increasing order, the number of newspapers read in different languages.
800
700
Notes
600
Students
hundred’s)
500
(inof
400
Number
Newspapers
300
200
100
0
English Hindi Punjabi Urdu Marathi Tamil
Language
Fig. 24.4
Solution:
(i) Number of newspapers (in hundreds) read in Hindi, English and
Punjabi = 800 + 700 + 400 = 1900
(ii) Number of newspapers (in hundreds) read in Hindi = 800
Number of newspapers (in hundreds) in Urdu,
Marathi and Tamil = 200 + 300 +100 = 600
So, difference (in hundreds) = 800 – 600) = 200
(iii) In Tamil, the number of newspapers read is the least.
(iv) Tamil, Urdu, Marathi, Punjabi, English, Hindi
Construction of Bar Graphs
Solution:
Step 1: Take a graph paper and draw two perpendicular lines and call them horizontal Notes
and vertical axes (Fig. 24.5)
Step 2: Along the horizontal axis, represent the information ‘years’ and along the vertical
axis, represent the corresponding ‘loans (in crores of rupees)’.
Step 3: Along the horizontal axis, choose a uniform (equal) width of bars and a uniform
gap between them, according to the space available.
Step 4: Choose a suitable scale along the vertical axis in view of the data given to us.
Step 5: Calculate the heights of the bars for different years as given below:
1
2000 : × 25 = 2.5 units
10
1
2001 : × 30 = 3 units
10
1
2002 : × 40 = 4 units
10
1
2003 : × 55 = 5.5 units
10
1
2004 : × 60 = 6 units
10
Step 6: Draw five bars of equal width and heights obtained in Step 5 above, the
corresponding years marked on the horizontal axis, with equal spacing between
them as shown in Fig. 24.5.
70
Notes
60
40
30
20
10
0
2000 2001 2002 2003 2004
Years
Fig. 24.5
Thus, Fig. 24.5 gives the required bar graph.
Example 24.7: The data below shows the number of students present in different classes
on a particular day.
Class VI VII VIII IX X
Number of students present 40 45 35 40 50
Represent the above data by a bar graph.
Solution: The bar graph for the above data is shown in Fig. 24.6.
60
Number of students present
50
40
30
20
10
0
VI VII VIII IX X
Classes
Fig. 24.6
60
55
50
45
Number of students
40
35
30
25
20
15
10
5
0
Playing Reading story Watching T V List ening to Paint ing
books music
Pre ferred activity
Fig. 24.7
6
Notes
5
0
Bus Car Bicycle Foot
Mode of transport
Fig. 24.8
Study the bar graph and answer the following questions:
(i) How many members of staff come to school on bicycle?
(ii) How many member of staff come to school by bus?
(iii) What is the most common mode of transfport of the members of staff?
3. The bar graph given below shows the number of players in each team of 4 given
games:
Volleyball
T able T ennis
Games
Football
Basket ball
0 1 2 3 4 5 6 7 8 9 10 11 12
Number of players
Fig. 24.9
1600
1400
1200
Number of trees planted
1000
800
600
400
200
0
2003 2004 2005 2006 2007 2008
Years
Fig. 24. 10
Study the above bar graph and answer the following questions:
(i) What is the total number of trees planted by the agency from 2003 to 2008?
(ii) In which year is the number of trees planted the maximum?
(iii) In which year is the number of trees planted the minimum?
(iv) In which year, the number of trees planted is less than the number of trees planted in
the year preceding it?
5. The expenditure of a company under different heads (in lakh of rupees) for a year is
given below:
Head Expenditure (in lakhs of rupees)
Salary of employees 200
Travelling allowances 100
Electricity and water 50
Rent 125
Others 150
Construct a bar chart to represent this data.
20 30 40 50 80 70 80
Fig. 24.11
Fig. 24.11 shows the histogram for the frequency distribution of marks obtained by 20
students in a class test.
Example 24.10: Draw a histogram for the following data:
Notes
Height 125-130 130-135 135-140 140-145 145-150 150-155 155-160
(in cm)
Number of 1 2 3 5 4 3 2
students
Solution: Following the steps as suggested in the above example, the histogram representing
the given data is given below:
Fig. 24.12
Frequency Polygon
There is yet another way of representing a grouped frequency distribution graphically. This
is called frequency polygen. To see what we mean, consider the histogram in Fig. 24.13.
E
B
A
20 30 40 50 80 70 80
H
Fig. 24.13
D G
C H
A I
Fig. 24.14
Example 24.12: Marks (out of 50) obtained by 30 students of Class IX in a mathematics
test are given in the following table:
Marks 0-10 10-20 20-30 30-40 40-50
Number of students 5 8 6 7 4
Draw a frequency polygon for this data.
Solution: Let us first draw a histogram for this data (Fig. 24.15)
Mark the mid points B, C, D, E and F of the tops of the rectangles as shown in Fig. 24.15.
Here, the first class is 0-10. So, to find the class preceding 0-10, we extend the horizontal
axis in the negative direction and find the mid point of the imaginary class (–10)-0. Let us
C
E
D
Notes
B F
A
I
H G
O 10 20 30 40 50 60
Fig. 24.15
join B to the mid point of the class (015010)-0. Let A be the mid point where this line
segment meets the vertial axis. Let G be the mid point of the class 50-60 (succeeding the
last class). Let the line segment FG intersects the length of the last rectangle at I (Fig.
24.15). Then OABCDEFIH is the required frequency polygen representing the given
data.
Note: Why have we not taken the points before O and G? This is so because marks
obtained by the students cannot go below 0 and beyond maximum marks 50. In the figure,
extreme line segments are only partly drawn and then brought down vertically to 0 and 50.
Frequency polygon can also be drawn independently without drawing histogram. We will
illustrate it through the following example.
Example 24.13: Draw a frequency polygon for the data given in Example 24.9, without
drawing a histogram for the data.
Solution: To draw a frequency polygon without drawing a histogram, we go through the
following steps.
Step 1: Draw two lines perpendicualar to each other.
Step 2: Find the class marks of the classes.
20 + 30 30 + 40 40 + 50 50 + 60 60 + 70 70 + 80
Here they are: , , , , and
2 2 2 2 2 2
i.e. the class marks are 25, 35, 45, 55, 65 and 75 respectively.
Step 3: Plot the points B (25, 1), C(35, 3), D(45, 1), E(55, 6), F(65, 4) and G(75, 5),
i.e., (class mark, frequency)
Step 4: Join the points B, C, D, E, F and G by line segments and complete the polygon
as explained earlier.
E (55, 6)
Notes
G (75, 5)
F (65, 4)
Frequency
C (35, 3)
B (25, 1) D (45, 1)
A
20 30 40 50 60 70 80
H
Fig. 24.16
Reading a Histogram
Consider the following example:
Example 24.14: Study the histogram given below and answer the following questions:
20 25 30 35 40 45 50 55
Fig. 24.17
(i) What is the number of teachers in the oldest and the youngest group in the school?
(ii) In which age group is the number of teachers maximum?
(v) How many students have the height more than or equal to 140 but less than 155?
Notes
Fig. 24.18
LET US SUM UP
• Statistics is that branch of mathematics which deals with collection, organisation, analysis
and interpretation of data.
• Statistics is used in both plural and singular sense.
• The data collected from the respondents “as it is” is called raw data.
• Data are said to be primary if the investigator himself collects it through his/her own
designed tools.
• Data taken from other sources such as printed reports, and not collected by the
experimenter himself, is called secondary data.
• The raw data arranged in ascending or decending order is called “arrayed data”.
• When the arrayed data are arranged with frequencies, they are said to form a frequency
table for ungrouped data or a ungrouped frequency distribution table.
• When the data are divided into groups/classes, they are called grouped data.
• The difference between the maximum and minimum observations occuring in the data
is called the range of the raw data.
• The number of classes have to be decided according to the range of the data and size
of class.
TERMINAL EXERCISE
1. Fill in the blanks by appropriate words/phrases to make each of the following statements
true:
(i) When the data are condensed in classes of equal size with frequencies, they are
called ________ data and the table is called _______ table.
(ii) When the class limits are adjusted to make them continuous, the class limits are
renamed as ________.
(iii) The number of observations falling in a particular class is called its _______.
(iv) The difference between the upper limit and lower limit of a class is called
_________.
(v) The sum of frequencies of a class and all classes prior to that class is called
________ frequency of that class.
(vi) Class size = Difference between ________ and _____ of the class.
(vii) The raw data arranged in ascending or descending order is called an _______
data.
Notes
(viii) The difference between the maximum and minimum observations occuring in the
data is called the _________ of the raw data.
2. The number of TV sets in each of 30 households are given below:
1, 2, 2, 4, 2, 1, 1, 1, 2, 1, 3, 1, 1, 1, 3
1, 2, 2, 1, 2, 0, 3, 3, 1, 2, 1, ,1 0, 1, 1
Construct a frequency table for the data.
3. The number of vehicles owned by each of 50 families are listed below:
2, 1, 2, 1, 1, 1, 2, 1, 2, 1, 0, 1, 1, 2, 3, 1, 1, 1,
2, 2, 1, 1, 3, 1, 1, 2, 1, 0, 1, 2, 1, 2, 1, 1, 4, 1
3, 1, 1, 1, 2, 2, 2, 2, 1, 1, 3, 2, 1, 2
Construct a frequency distribution table for the data.
4. The weight (in grams) of 40 New Year’s cards were found as:
10.4 6.3 8.7 7.3 8.8 9.1 6.7 11.1 14.0 12.2
11.3 9.4 8.6 7.1 8.4 10.0 9.1 8.8 10.3 10.2
7.3 8.6 9.7 10.9 13.6 9.8 8.9 9.2 10.8 9.4
6.2 8.8 9.4 9.9 10.1 11.4 11.8 11.2 10.1 8.3
Prepare a grouped frequency distribution using the class 5.5-7.5, 7.5-9.5 etc.
5. The lengths, in centimetres, to the nearest centimeter of 30 carrots are given below:
15 21 20 10 18 18 16 18 20 20
18 16 13 15 15 16 13 14 14 16
12 15 17 12 14 15 13 11 14 17
Construct a frequency table for the data using equal class sizes and taking one class as
10-12 (12 excluded).
6. The following is the distribution of weights (in kg) of 40 persons:
Fig. 24.19
(i) What is the information given by the bar graph?
(ii) On which day is number of students born the maximum?
(iii) How many more students were born on Thursday than that on Tuesday.
(iv) What is the total number of students in the class?
10 15 20 25 30 35 40
Fig. 24.20
Study the above histogram and answer the following questions:
(i) What was the total number of literate females in the town in the age group 10 to 40?
(ii) In which age group, the number of literate females was the highest?
(iii) In which two age groups was the number of literate females the same?
Notes
25
MEASURES OF CENTRAL TENDENCY
In the previous lesson, we have learnt that the data could be summarised to some extent
by presenting it in the form of a frequency table. We have also seen how data were
represented graphically through bar graphs, histograms and frequency polygons to get
some broad idea about the nature of the data.
Some aspects of the data can be described quantitatively to represent certain features of
the data. An average is one of such representative measures. As average is a number of
indicating the representative or central value of the data, it lies somewhere in between the
two extremes. For this reason, average is called a measure of central tendency.
In this lesson, we will study some common measures of central tendency, viz.
OBJECTIVES
After studying this lesson, you will be able to
• define mean of raw/ungrouped and grouped data;
• calculate mean of raw/ungrouped data and also of grouped data by ordinary
and short-cut-methods;
• define median and mode of raw/ungrouped data;
• calculate median and mode of raw/ungrouped data.
x1 + x 2 + ... + x n
n
It is generally denoted by x . so
x1 + x 2 + ... + x n
x =
n
∑x
i =1
i
= (I)
n
where the symbol “Σ” is the capital letter ‘SIGMA’ of the Greek alphabet and is used to
denote summation.
To economise the space required in writing such lengthy expression, we use the symbol Σ,
read as sigma.
n
In ∑x
i =1
i , i is called the index of summation.
Example 25.1: The weight of four bags of wheat (in kg) are 103, 105, 102, 104. Find the
mean weight.
103 + 105 + 102 + 104
Solution: Mean weight ( x ) = kg
4
414
= kg = 103.5 kg
4
Example 25.2: The enrolment in a school in last five years was 605, 710, 745, 835 and
910. What was the average enrolment per year?
Solution: Average enrolment (or mean enrolment)
30
∑x i
40 + 73 + .... + 28 1455
Mean = ( x ) = i =1
= =
n 30 30
= 48.5
Example 25.4: Refer to Example 25.1. Show that the sum of x1– x , x2– x , x3– x and
x4– x is 0, where xi’s are the weights of the four bags and x is their mean.
Solution: x1– x = 103 – 103.5 = – 0.5, x2– x = 105 – 103.5 = 1.5
x3– x = 102 – 103.5 = – 1.5, x4– x = 104 – 103.5 = 0.5
So, (x1– x ) + (x2– x ) + (x3– x ) + (x4– x ) = – 0.5 + 1.5 + (–1.5) + 0.5 = 0
Example 25.5: The mean of marks obtained by 30 students of Section A of Class X is
48, that of 35 students of Section B is 50. Find the mean marks obtained by 65 students
in Class X.
Solution: Mean marks of 30 students of Section A = 48
So, total marks obtained by 30 students of Section A = 30 × 48 = 1440
Similarly, total marks obtained by 35 students of Section B = 35 × 50 = 1750
Total marks obtained by both sections = 1440 + 1750 = 3190
3190
Mean of marks obtained by 65 students = = 49.1 approx.
65
Example 25.6: The mean of 6 observations was found to be 40. Later on, it was detected
that one observation 82 was misread as 28. Find the correct mean.
294
Hence, correct mean = = 49
6
i.e.,
∑x i
. But this process will be time consuming.
n
We can also find the mean of this data by first making a frequency table of the data and
then applying the formula:
n
∑fx i i
mean = x = i =1
n
(II)
∑f
i =1
i
∑fx i i
=
146
= 7.3
Mean =
∑f i 20
Example 25.7: The following data represents the weekly wages (in rupees) of the Notes
employees:
Weekly wages 900 1000 1100 1200 1300 1400 1500
(in `)
Number of 12 13 14 13 14 11 5
employees
Find the mean weekly wages of the employees.
Solution: In the following table, entries in the first column are xi’s and entries in second
columen are fi’s, i.e., corresponding frequencies. Recall that to find mean, we require the
product of each xi with corresponding frequency fi. So, let us put them in a column as
shown in the following table:
Weekly wages (in `) Number of employees f ix i
(xi) (fi)
900 12 10800
1000 13 13000
1100 14 15400
1200 13 15600
1300 12 15600
1400 11 15400
1500 5 7500
Σfi = 80 Σfi xi = 93300
∑fx i i 93300
Mean weekly wages = =`
∑f i 80
= ` 1166.25
Sometimes when the numerical values of xi and fi are large, finding the product fi and xi
becomes tedius and time consuming.
We wish to find a short-cut method. Here, we choose an arbitrary constant a, also called
the assumed mean and subtract it from each of the values xi. The reduced value,
di = xi – a is called the deviation of xi from a.
Thus, xi = – a + di
Notes
∑ f x =∑ af + ∑ f d
i =1
i i
i =1
i
i =1
i i [Summing both sides over i from i to r]
1
Hence x = ∑ f i + ∑ fi di , where Σfi = N
N
1
x =a+
N
∑ f i di (III)
[since Σfi = N]
This meghod of calcualtion of mean is known as Assumed Mean Method.
In Example 25.7, the values xi were very large. So the product fixi became tedious and
time consuming. Let us find mean by Assumed Mean Method. Let us take assumed
mean a = 1200
Weekly wages Number of Deviations f id i
(in `) (xi) employees (fi) di = xi – 1200
900 12 – 300 – 3600
1000 13 – 200 – 2600
1100 14 – 100 – 1400
1200 13 0 0
1300 12 100 + 1200
1400 11 200 + 2200
1500 5 300 + 1500
Σfi = 80 Σfi di = – 2700
1
Mean = a +
N
∑ f i di
1
= 1200 + (– 2700)
80
= 1200 – 33.75 = 1166.25
So, the mean weekly wages = ` 1166.25
Observe that the mean is the same whether it is calculated by Direct Method or by Assumed
Mean Method.
Example 25.8: If the mean of the following data is 20.2, find the value of k
xi 10 15 20 25 30
fi 6 8 20 k 6
Notes
∑fx i i
=
60 + 120 + 400 + 25k + 180
Solution: Mean =
∑f i 40 + k
760 + 25k
=
40 + k
760 + 25k
So, = 20.2 (Given)
40 + k
or 760 +25k = 20.2 (40 + k)
or 7600 + 250k = 8080 + 202k
or k = 10
What we can infer from this table is that there are 5 workers earning daily somewhere
from ` 150 to ` 160 (not included 160). We donot know what exactly the earnings of
each of these 5 workers are
Therefore, to find mean of the grasped frequency distribution, we make the following
assumptions:
Frequency in any class is centred at its class mark or mid point
150 + 160
Now, we can say that there are 5 workers earning a daily wage of ` =
2
160 + 170
` 155 each, 8 workers earning a daily wage of ` = ` 165, 15 workers aerning
2
170 + 160
a daily wage of ` = ` 175 and so on. Now we can calculate mean of the given
2
data as follows, using the Formula (II)
∑fx i i
=
6960
= 174
Mean =
∑f i 40
So, the mean daily wage = ` 174 Notes
This method of calculate of the mean of grouped data is Direct Method.
We can also find the mean of grouped data by using Formula III, i.e., by Assumed Mean
Method as follows:
We take assumed mean a = 175
Daily wages Number of Class marks Deviations f id i
(in `) workers (fi) (xi) di = xi–175
150-160 5 155 – 20 – 100
160-170 8 165 – 10 – 80
170-180 15 175 0 0
180-190 10 185 + 10 100
190-200 2 195 + 20 40
Σfi = 40 Σfidi = – 40
So, using Formula III,
1
Mean = a +
N
∑ f i di
1
= 175 + (–40)
40
= 175 – 1 = 174
Thus, the mean daily wage = ` 174.
Example 25.9: Find the mean for the following frequency distribution by (i) Direct Method,
(ii) Assumed Mean Method.
Class Frequency
20-40 9
40-60 11
60-80 14
80-100 6
100-120 8
120-140 15
140-160 12
Total 75
∑fx i i
=
6970
= 92.93
So, mean =
∑f i 75
1 220
Mean = a +
N
∑ f i di = 90 +
75
= 92.93
Note that mean comes out to be the same in both the methods.
In the table above, observe that the values in column 4 are all multiples of 20. So, if we
divide these value by 20, we would get smaller numbers to multiply with fi.
Note that, 20 is also the class size of each class.
xi − a
So, let ui = , where a is the assumed mean and h is the class size.
h
⎛ ∑ f iU i ⎞
Mean = x = a + ⎜⎜ ⎟⎟ × h (IV) Notes
⎝ ∑ f i ⎠
⎛ ∑ f i ui ⎞
Mean = x = a + ⎜⎜ ⎟⎟ × h = 90 + 11 × 20
⎝ ∑ fi ⎠ 75
220
= 90 + = 92.93
75
Calculating mean by using Formula (IV) is known as Step-deviation Method.
Note that mean comes out to be the same by using Direct Method, Assumed Method or
Step Deviation Method.
Example 25.10: Calcualte the mean daily wage from the following distribution by using
Step deviation method.
Daily wages (in `) 150-160 160-70 170-180 180-190 190-200
Numbr of workers 5 8 15 10 2
⎛ ∑ f i ui ⎞
Mean daily wages = a + ⎜⎜ ⎟⎟ × h = 175 + − 4 × 10 = ` 174
⎝ ∑ fi ⎠ 40
Note: Here again note that the mean is the same whether it is calculated using the Direct
Method, Assumed mean Method or Step deviation Method.
Calculate mean weekly cost of living index by using Step deviation Method.
4. Find the mean of the following data by using (i) Assumed Mean Method and (ii) Step
deviation Method.
Class 150-200 200-250 250-300 300-350 350-400
Frequency 48 32 35 20 10
25.2 MEDIAN
In an office there are 5 employees: a superviosor and 4 workers. The workers draw a
salary of ` 5000, ` 6500, ` 7500 and ` 8000 per month while the supervisor gets
` 20000 per month.
5000 + 6500 + 7500 + 8000 + 20000
In this case mean (salary) = `
5
47000
=` = ` 9400
5
Note that 4 out of 5 employees have their salaries much less than ` 9400. The mean salary
` 9400 does not given even an approximate estimate of any one of their salaries.
This is a weakness of the mean. It is affected by the extreme values of the observations in
the data.
This weekness of mean drives us to look for another average which is unaffected by a few
extreme values. Median is one such a measure of central tendency.
Median is a measure of central tendency which gives the value of the middle-
most observation in the data when the data is arranged in ascending (or descending)
order.
⎛ n +1⎞
(ii) When the number of observations (n) is odd, the median is the value of ⎜ ⎟ th
⎝ 2 ⎠
observation.
⎛n⎞
(iii) When the number of observations (n) is even, the median is the mean of the ⎜ ⎟ th
⎝2⎠
Notes ⎛n ⎞
and ⎜ +1⎟ th observations.
⎝2 ⎠
Let us illustrate this with the help of some examples.
Example 25.11: The weights (in kg) of 15 dogs are as follows:
9, 26, 10 , 22, 36, 13, 20, 20, 10, 21, 25, 16, 12, 14, 19
Find the median weight.
Solution: Let us arrange the data in the ascending (or descending) order:
9, 10, 10, 12, 13, 14, 16, 19, 20, 20, 21, 22, 25, 36
Median
Here, number of observations = 15
⎛ n +1⎞ ⎛ 15 + 1 ⎞
So, the median will be ⎜ ⎟ th, i.e., ⎜ ⎟ th, i.e., 8th observation which is 19 kg.
⎝ 2 ⎠ ⎝ 2 ⎠
Remark: The median weight 19 kg conveys the information that 50% dogs have weights
less than 19 kg and another 50% have weights more then 19 kg.
Example 25.12: The points scored by a basket ball team in a series of matches are as
follows:
16, 1, 6, 26, 14, 4, 13, 8, 9, 23, 47, 9, 7, 8, 17, 28
Find the median of the data.
Solution: Here number of observations = 16
⎛ 16 ⎞ ⎛ 16 ⎞
So, the median will be the mean of ⎜ ⎟ th and ⎜ +1⎟ th, i.e., mean of 6th and 9th
⎝2⎠ ⎝ 2 ⎠
observations, when the data is arranged in ascending (or descending) order as:
1, 4, 6, 7, 8, 8, 9, 9, 13, 14, 16, 17, 23, 26, 28, 47
8th term 9th term
9 + 13
So, the median = = 11
2
Remark: Here again the median 11 conveys the information that the values of 50% of the
observations are less than 11 and the values of 50% of the observations are more than 11.
⎛ n +1⎞ ⎛ 35 + 1 ⎞
Here n = 35, which is odd. So, the median will be ⎜ ⎟ th, i.e., ⎜ ⎟ th, i.e., 18th
⎝ 2 ⎠ ⎝ 2 ⎠
observation.
To find value of 18th observation, we prepare cumulative frequency table as follows:
Marks obtained Number of students Cumulative frequency
3 4 4
5 6 10
6 5 15
7 3 18
10 1 19
11 7 26
12 3 29
13 2 31
14 3 34
15 1 35
From the table above, we see that 18th observation is 7
So, Median = 7
Example 25.14: Find the median of the following data:
Weight (in kg) 40 41 42 43 44 45 46 48
Number of 2 5 7 8 13 26 6 3
students
⎛n⎞ ⎛n ⎞
Since n is even, so the median will be the mean of ⎜ ⎟ th and ⎜ +1⎟ th observations,
⎝2⎠ ⎝2 ⎠
i.e., 35th and 36th observations. From the table, we see that
35 the observation is 44
and 36th observation is 45
44 + 45
So, Median = = 44.5
2
(b) xi 5 10 15 20 25 30 35 40
fi 3 7 12 20 28 31 28 26
25.3 MODE
Look at the following example:
A company produces readymade shirts of different sizes. The company kept record of its
sale for one week which is given below:
From the table, we see that the sales of shirts of size 105 cm is maximum. So, the company
will go ahead producing this size in the largest number. Here, 105 is nothing but the mode
of the data. Mode is also one of the measures of central tendency.
The observation that occurs most frequently in the data is called mode of the
data.
In other words, the observation with maximum frequency is called mode of the data.
The readymade garments and shoe industries etc, make use of this measure of central
tendency. Based on mode of the demand data, these industries decide which size of the
product should be produced in large numbers to meet the market demand.
In case of raw data, it is easy to pick up mode by just looking at the data. Let us consider
the following example:
LET US SUM UP
• Mean, median and mode are the measures of central tendency.
∑x
i =1
i
• Mean (Arithmetic average) of raw data is givne by x =
n
∑fx ∑fx i i
i i
• Mean of ungrouped data is given by x= i =1
=
n N
∑f
i =1
i
1
• Mean of ungrouped data can also be found by using the formula x = a + ∑ f i di
N
where di = xi – a, a being the assumed mean
Mean of grouped data
(i) To find mean of the grouped frequency distribution, we take the assumption:
Frequency in any class is centred at its class mark or mid point.
∑fx i i
Notes x= i =1
n
∑f
i =1
i
where xi’s are the class marks and fi are the corresponding freqeucies of xi’s.
(iii) Assumed Mean Method
∑fd i i
x =a+ i =1
N
⎛ n ⎞
⎜ ∑ f i ui ⎟
x = a + ⎜ i =1n ⎟× h
⎜ ⎟
⎜ ∑ fi ⎟
⎝ i =1 ⎠
xi − a
where a is the assumed mean, ui = and h is the class size.
h
• Median is a measure of central tendency which gives the value of the middle most
obseration in the data, when the data is arranged in ascending (or descending) order.
• Median of raw data
⎛ n +1⎞
(i) When the number of observations (n) is odd, the median is the value of ⎜ ⎟ th
⎝ 2 ⎠
observation.
⎛n⎞
(ii) When the number of observations (n) is even, the median is the mean of the ⎜ ⎟ th
⎝2⎠
⎛n ⎞
and ⎜ +1⎟ th observations.
⎝2 ⎠
• Median of ungrouped data
Median of ungrouped data can be found from the cumulative frequency table (arranging
data in increasing or decreasing order) using (i) and (ii) above.
• The value of observation with maximum frequency is called the mode of the data.
TERMINAL EXERCISE
1. Find the mean of first five prime numbers. Notes
2. If the mean of 5, 7, 9, x, 11 and 12 is 9, find the value of x.
3. Following are the marks obtained by 9 students in a class
51, 36, 63, 46, 38, 43, 52, 42 and 43
(i) Find the mean marks of the students.
(ii) What will be the mean marks if a student scoring 75 marks is also included in the
class.
4. The mean marks of 10 students in a class is 70. The students are divided into two
groups of 6 and 4 respectively. If the mean marks of the first group is 60, what will be
the mean marks of the second group?
n
6. There are 50 numbers. Each number is subtracted from 53 and the mean of the numbers
so obtained is found to be – 3.5. Determine the mean of the given numbers.
7. Find the mean of the following data:
(a) xi 5 9 13 17 22 25
fi 3 5 12 8 7 5
(b) xi 16 18 28 22 24 26
fi 1 3 5 7 5 4
8. Find the mean of the following data
(a) Classes 10-20 20-30 30-40 40-50 50-60 60-70
Frequencies 2 3 5 7 5 3
(b) xi 35 36 37 38 39 40 41 42
fi 2 3 5 4 7 6 4 2
12. Find the mode of the following data:
(a) 8, 5, 2, 5, 3, 5, 3, 1
(b) 19, 18, 17, 16, 17, 15, 14, 15, 17, 9
13. Find the mode of the following data which gives life time (in hours) of 80 bulbs selected
at random from a lot.
Life time (in hours) 300 500 700 900 1100
Number of bulbs 10 12 20 27 11
14. In the mean of the following data is 7, find the value of p:
xi 4 p 6 7 9 11
fi 2 4 6 10 6 2
15. For a selected group of people, an insurance company recorded the following data:
Age (in years) 0-10 10-20 20-30 30-40 40-50 50-60 60-70 70-80
Number of deaths 2 12 55 95 71 42 16 7
Determine the mean of the data.
16. If the mean of the observations: x + 1, x + 4, x + 5, x + 8, x + 11 is 10, the mean of
the last three observations is
(A) 12.5 (B) 12.2 (C) 13.5 (D) 14.2
1. ∑ x /n
i =1
i 2. 5.5 3. 86.33 kg
4. 142.8 cm 5. 25.68oC 7. 42
8. 29.17
25.2
1. 5.84 2. (i) 18.99 (ii) 6.57
3. 11.68 4. 10
25.3
1. 28.80 2. 55.19 3. 167.9 4. 244.66
25.4
1. 3 2. 50 3. 4
4. (a) 4 (b) 30 (c) 5.8
25.5
1. 2 2. 1 3. 6 4. 7
Notes
26
INTRODUCTION TO PROBABILITY
OBJECTIVES
After studying this lesson, you will be able to
• understand the meaning of a random experiment;
• differentiate between outcomes and events of a random experiment;
• define probability P(E) of occurrence of an event E;
• determine P( E ) if P(E) is given;
• state that for the probability P(E), 0 ≤ P(E) ≤ 1;
• apply the concept of probability in solving problems based on tossing a coin
throwing a die, drawing a card from a well shuffled deck of playing cards, etc.
In the above situations, tossing a coin, throwing a die, planting seeds and observing the
germinated seeds, each is an example of a random experiment
In (1), the possible outcomes of the random experiment of tossing a coin are: Head and
Tail.
In (2), the possible outcomes of the experiment are: 1, 2, 3, 4, 5, 6
In (3), the possible outcomes are: 0, 1, 2, 3, 4.
A random experiment always has more than one possible outcomes. When the experiment
is performed only one outcome out of all possible outcomes comes out. Moreover, we
can not predict any particular outcome before the experiment is performed. Repeating the
experiment may lead to different outcomes.
Some more examples of random experiments are:
1
=
2
Similarly, if F is the event “getting a tail”, then
1
P(F) =
2
Example 26.2: A die is thrown once. What is the probability of getting a number 3?
Solution: Let E be the event “getting a number 3”.
Possible outcomes of the experiment are: 1, 2, 3, 4, 5, 6
5
So, P(F) =
6
Note that event F in Example 26.3 is the same as event ‘not E’ in Example 26.2.
Example 26.4: A ball is drawn at random from a bag containing 2 red balls, 3 blue balls
and 4 black balls. What is the probability of this ball being of (i) red colour (ii) blue colour
(iii) black colour (iv) not blue colour?
Solution:
(i) Let E be the event that the drawn ball is of red colour
Number of possible outcomes of the experiment = 2 + 3 + 4=9
(Red) (Blue) (black)
Number of outcomes favourable to E = 2
2
So, P(Red ball) = P(E) =
9
(ii) Let F be the event that the ball drawn is of blue colour
3 1
So, P(Blue ball) = P(F) = =
9 3
(iii) Let G be the event that the ball drawn is of black colour
4
So P (Black ball) = P(G) =
9
26 1
Thus, P(E) = =
52 2
(ii) Let F be the event that the card drawn is of black colour. Number of cards of black
colour = 13 + 13 = 26
26 1
So P(F) = =
52 2
Example 26.6: A die is thrown once. What is the probability of getting a number (i) less
than 7? (ii) greater than 7?
Solution: (i) Let E be the event “number is less than 7”.
Number of favourable outcomes to E = 6 (since every face of a die is marked with
a number less than 7)
6
So, P(E) = =1
6
(ii) Let F be the event “number is more than 7”
Number of outcomes favourable to F = 0 (since no face of a die is marked with a
number more than 7)
0
So, P(F) = =0
6
1 1 1 1 1 1
P(1) + P (2) + P(3) + P(4) + P(5) + P(6) = + + + + + =1
6 6 6 6 6 6 Notes
Observe that the sum of the probabilities of all the elementary events of an
experiment is one.
Observation 3: From Examples 26.2 and 26.3,
1 5
Probability of getting 3 + Probability of getting a number other than 3 = + =1
6 6
i.e. P(3) + P(not 3) = 1
or P(E) + P(not E) = 1 ...(1)
Similarly, in Example 26.1
1
P(getting a head) = P(E) =
2
1
P(getting a tail) = P(F) =
2
1 1
So, P(E) + P(F) = + =1
2 2
So, P(E) + P(not E) = 1 [getting a tail means getting no head] ...(2)
From (1) and (2), we see that for any event E,
P(E) + P(not E) = 1
P(E) + P( E ) = 1
2
Example 26.7: If P(E) = , what is the probability of ‘not E’?
7
Solution: P(E) + P(not E) = 1
2 5
So, P(not E) = 1 – P(E) = 1 – =
7 7
Example 26.8: What is the probability that the number 5 will not come up in single throw
Notes
of a die?
Solution: Let E be the event “number 5 comes up on the die”
1
Now P(E) =
6
1 5
So, P( E ) == 1 – =
6 6
Example 26.9: A card is drawn at random from a well-shuffled deck of 52 cards. Find
the probability that this card is a face card.
Solution: Number of all possible outcomes = 52
Number of outcomes favourable to the Event E “a face card” = 3 × 4 = 12
[Kings, queens, and jacks are face cards]
12 3
So, P(a face card) = =
52 13
Example 26.10: A coin is tossed two times. What is the probability of getting a head each
time?
Solution: Let us write H for Head and T for Tail.
In this expreiment, the possible outcomes will be: HH, HT, TH, TT
HH means Head on both the tosses
HT means Head on 1st toss and Tail on 2nd toss.
TH means Tail on 1st toss and Head on 2nd toss.
TT means Tail on both the tosses.
So, the number of possible outcomes = 4
Let E be the event “getting head each time”. This means getting head in both the
tosses, i.e. HH.
1
Therefore, P(HH) =
4
100 10
So, P(E) = =
110 11
Example 26.12: Two dice, one of black colour and other of blue colour, are thrown at
the same time. Write down all the possible outcomes. What is the probability that same
number appear on both the dice?
Solution: All the possible outcomes are as given below, where the first number in the
bracket is the number appearing on black coloured die and the other number is on blue
die. 2
Blue coloured die 3
1 2 3 4 5 6
1 (1, 1) (1, 2) (1, 3) (1, 4) (1, 5) (1, 6)
Black
coloured 2 (2, 1) (2, 2) (2, 3) (2, 4) (2, 5) (2, 6)
die 3 (3, 1) (3, 2) (3, 3) (3, 4) (3, 5) (3, 6)
1 4 (4, 1) (4, 2) (4, 3) (4, 4) (4, 5) (4, 6)
2 3
5 (5, 1) (5, 2) (5, 3) (5, 4) (5, 5) (5, 6)
6 (6, 1) (6, 2) (6, 3) (6, 4) (6, 5) (6, 6)
So, the number of possible outcomes = 6 × 6 = 36
The outcomes favourable to the event E : “Same number appears on both dice”. are
(1, 1), (2, 2), (3, 3), (4, 4), (5, 5) and (6, 6).
So, the number of outcomes favourable to E = 6.
6 1
Hence, P(E) = =
36 6
6. If P( E ) = 0, find P(E).
7. A card is drawn from a well shuffled deck of 52 playing cards. Find the probability
that this card will be
(i) a red card (ii) a black card
(iii) a red queen (iv) an ace of black colour
(v) a jack of spade (vi) a king of club
(vii) not a face card (viii) not a jack of diamonds
8. A bag contains 15 white balls and 10 blue balls. A ball is drawn at random from the
bag. What is the probability of drawing
(i) a ball of not blue colour (ii) a ball not of white colour
9. In a bag there are 3 red, 4 green and 2 blue marbles. If a marble is picked up at
random what is the probability that it is
(i) not green? (ii) not red? (iii) not blue?
LET US SUM UP
• A random experiment is one which has more than one outcomes and whose outcome
is not exactly predictable in advance before performig the experiment.
• One or more outcomes of an experiment constitute an event.
• An event having only one outcome of the experiment is called an elementary event.
• Probability of an event E, P(E), is defined as
TERMINAL EXERCISE
1. Which of the following statements are True (T) and which are False (F):
(i) Probability of an event can be 1.01